Sunteți pe pagina 1din 65

Aznar vs. Garcia, G.R. No. L-16749, Jan.

3, 1963 Renvoi doctrine is not applicable because


there is no conflict as to the nationality
FACTS: and domicile of Bellis. He is both a citizen
and a resident of Texas. So even if
Edward Christensen, who at his death was a US assuming the law of Texas applies the
citizen but domiciled in the Philippines, left a domiciliary rule, it is still Texas law that
will, devising unto Maria Helen a certain governs because his domicile is Texas.
amount of money and giving the rest of his
estate to Maria Lucy. Helen opposed the Gibbs v. Government of Philippine Islands
partition on the ground that she is deprived of
her legitime. Her contention is that the law of Facts:
California directs that the law of the domicile
(Philippines) should govern the will. Gibbs and his wife were American nationals,
domiciled in California. They acquired lands in
ISSUE: Whether or not the national law or the the Philippines. The wife died in California.
domiciliary law should apply Gibbs was appointed administrator of the
intestate proceedings instituted in Manila.
HELD: Gibbs asked the court to adjudicate to him
lands acquired in the Philippines not under our
The intrinsic validity of wills is governed by the laws on succession but because in accordance
national law of the decedent. In the present with the law of California, the community
case, the national law of Edward is the laws of property of spouses who are citizens of
California. However, there were two conflicting California, upon the death of the wife previous
California laws regarding succession. One is to that of the husband, belongs absolutely to
enunciated in In Re Kaufman (which does the surviving husband without administration.
not provide for legitimes) and another is CFI granted such to Gibbs upon proof of
Art. 946 of the California Civil Code California law. The register of deeds refused to
(which provides that the law of the transfer such properties on the ground of non-
domicile applies). SC held that the payment of inheritance tax. Gibbs argued that
national law is Art. 946, which is the the conjugal right of a California wife in a
conflict of laws rule of California. The community property is a personal right and
reason is that In Re Kaufman applies only to even if this was a case of succession, California
residents while Art. 946 is specific to non- law would still apply.
residents. Thus, since Art. 946 contains a refer-
back to Philippine laws (the law of the Issue: is Gibbs exempt from inheritance tax?
domicile), then Maria Helen is entitled to her
legitime. Held:

Bellis vs. Bellis, G.R. No. L-23678, June 6, 1967 The appellee contends that the law of
California should determine the nature
FACTS: and extent of the title, if any, that was
vested in Eva Johnson Gibbs citing article
Amos Bellis, a US citizen, died a resident of 9 of the Civil Code. But that, even if the
Texas. He left two wills -- one devising a nature and extent of her title under said
certain amount of money to his first wife and certificates be governed by the law of the
three illegitimate children and another, leaving Philippine Islands, the laws of California
the rest of his estate to his seven legitimate govern the succession to such title, citing
children. Before partition, the illegitimate the second paragraph of article 10 of the
children who are Filipinos opposed on the Civil Code. It is argued that the conjugal right
ground that they are deprived of their of the California wife in community real estate
legitimes. in the Philippine Islands is a personal right and
must, therefore, be settled by the law
ISSUE: Whether the applicable law is Texas law
governing her personal status, that is, the law
or Philippine laws
of California. But our attention has not been
HELD: called to any law of California that
incapacitates a married woman from acquiring
Applying the nationality rule, the law of Texas or holding land in a foreign jurisdiction in
should govern the intrinsic validity of the will accordance with the lex rei sitae.
and therefore answer the question on
entitlement to legitimes. But since the law of The trial court found that under the law of
Texas was never proven, the doctrine of California, upon the death of the wife, the
processual presumption was applied. entire community property without
Hence, SC assumed that Texas law is the administration belongs to the surviving
same as Philippine laws, which upholds husband; that he is the absolute owner of
the nationality rule. all the community property from the
moment of the death of his wife, not by

Private International Law Page 1


virtue of succession or by virtue of her
death, but by virtue of the fact that when The petitioner is a special administrator of
the death of the wife precedes that of the the estate of Edward Hix. He alleged that
the latter’s will was executed in Elkins,
husband he acquires the community
West Virginia on November 3, 1925 by Hix
property, not as an heir or as the who had his residence in that jurisdiction,
beneficiary of his deceased wife, but and that the laws of that state govern.
because she never had more than an
inchoate interest or expentancy which is To this end, the petitioner submitted a
extinguished upon her death. However, the copy of Section 3868 of Acts 1882, c.84
argument of the appellee apparently leads to as found in West Virginia Code, annotated
by Hogg, Charles E., vol.2 1914, p. 1690
this dilemma: If he takes nothing by succession
and as certified to by the Director of
from his deceased wife, how can the second National Library.
paragraph of article 10 be invoked? Can the
appellee be heard to say that there is a The Judge of the First Instance however
legal succession under the law of the denied the probate of the will on the
Philippine Islands and no legal succession grounds that the will did not show the
under the law of California? It seems clear following:
that the second paragraph of article 10 applies • acknowledgment by Hix in the
presence of 2 competent witnesses
only when a legal or testamentary succession
• Witnesses subscribed to will in
has taken place in the Philippines and in presence of the testator, and of each
accordance with the law of the Philippine other
Islands; and the foreign law is consulted only in
regard to the order of succession or the extent Hence, this appeal.
of the successional rights; in other words, the
second paragraph of article 10 can be invoked Issue:
only when the deceased was vested with a
Is it necessary to prove in this jurisdiction
descendible interest in property within the the existence of such law in West Virginia
jurisdiction of the Philippine Islands. as a prerequisite to the allowance and
recording of said will?
However, the court held that it is
principle firmly established that to the Held:
law of the state in which the land is
situated we must look for the rules which Yes. The laws of the foreign jurisdiction do
not prove themselves in our courts. The
govern its descent, alienation, and
courts of the Philippine Islands are not
transfer, and for the effect and authorized to take judicial notice of the laws
construction of wills and other of the various states of the American
conveyances. This fundamental principle is Union. Such laws must be proved as
stated in the first paragraph of article 10 of our facts.
Civil Code as follows: "Personal property is
subject to the laws of the nation of the Here the requirements of the law were not
met.
owner thereof; real property to the laws
of the country in which it is situated. • There was no showing that the book
from which an extract was taken was
Under the provisions of the Civil Code and printed or published under the
the jurisprudence prevailing here, the authority of the state of West Virginia,
wife, upon the acquisition of any conjugal as provided in Sec 30 of the Code of
property, becomes immediately vested Civil Procedure.
with an interest and title therein equal to • Nor was the extract from the law
that of her husband, subject to the power attested by the certificate of the officer
of management and disposition which the having charge of the original, under the
seal of the State of West Virginia as
law vests in the husband. It results that the
provided in Sec 301.
wife of the appellee was, by the law of the
• No evidence was introduced showing
Philippine Islands, vested of a descendible that the extract from the laws of West
interest, equal to that of her husband and the Virginia was in force at the time the
descendible interest of Eva Johnson Gibbs alleged will was executed.
in the lands aforesaid was transmitted to • Due execution of the will was not
her heirs by virtue of inheritance and this established: only showed testimony of the
transmission plainly falls within the petitioner
language of section 1536 of Article XI of
The court therefore did not err in denying
Chapter 40 of the Administrative Code the probate of the will. The existence of
which levies a tax on inheritances. such law in West
Virginia must be proved.
Fleumer vs. Hix
54 Phil 610
Miciano v. Brimo
Facts:

Private International Law Page 2


Decedent is a Turkish citizen. He left a will Impossible conditions and those
stating that he wishes to dispose of his contrary to law or good morals
properties in accordance with Philippine Laws. shall be considered as not imposed
and shall not prejudice the heir or
His brother, Andre Brimo opposed the partition.
legatee in any manner whatsoever,
The appellant's opposition is based on the fact even should the testator otherwise
that the partition in question puts into effect provide.
the provisions of Joseph G. Brimo's will which
are not in accordance with the laws of his And said condition is contrary to law
Turkish nationality, for which reason they are because it expressly ignores the
void as being in violation or article 10 of the testator's national law when, according to
Civil Code which states that testamentary article 10 of the civil Code above quoted,
successions shall be governed by the national such national law of the testator is the
one to govern his testamentary
law of the deceased. He was therefore
dispositions.
excluded from the inheritance pursuant to a
clause in the decedent’s will that anyone who
PCIB VS. ESCOLIN
would oppose the decedent’s wish to dispose
56 SCRA 266
his estate under Philippine law shall have his
share annul or cancelled.
FACTS:
Issue:
Linnie Jane Hodges died giving her
Whether or not the declaration that Turkish testamentary provisions to her husband. At the
laws are impertinent to this case; time of her
death, she was citizen of Texas but, was,
Whether or not the appellant’s exclusion from
however domiciled in the Philippines. To see
the will is valid?
whether
Held: the testamentary provisions are valid, it is
apparent and necessary to know what law
1st issue should be
applied.
The oppositor did not prove that said
testamentary dispositions are not in ISSUE:
accordance with the Turkish laws, inasmuch as
he did not present any evidence showing Whether or not laws of Texas is applicable.
what the Turkish laws are on the matter,
and in the absence of evidence on such RULING:
laws, they are presumed to be the same
as those of the Philippines.
Prior evidence already presented to prove the
The refusal to give the oppositor another existence of Texas Law.
opportunity to prove such laws does not
constitute an error. It is discretionary with the It is necessary that the Texas law be
trial court, and, taking into consideration that ascertained. Here it must be proven whether a
the oppositor was granted ample opportunity renvoi will
to introduce competent evidence, the Court happen or whether Texas law makes the
finds no abuse of discretion on the part of the testamentary provisions valid. In line with
lower court in this particular. There is, Texas law,
therefore, no evidence in the record that that which should be proven is the law
the national law of the testator Joseph G. enforced during the death of Hodges and not in
Brimo was violated in the testamentary
any other
dispositions in question which, not being
time.
contrary to our laws in force, must be
complied with and executed.
The Supreme Court held that for what the
2nd issue Texas law is on the matter, is a question
of fact to be resolved by the evidence
The institution of legatees in this will is that would be presented in the probate
conditional, and the condition is that the court. Texas law at the time of her death
instituted legatees must respect the testator's (and not said law at any other time).
will to distribute his property, not in
accordance with the laws of his nationality, but
in accordance with the laws of the Philippines. Article 16 of the Civil Code provides that
“the national law of the person whose
succession is under consideration,
The fact is, however, that the said condition is whatever may be the nature of the
void, being contrary to law, for article 792 of property and regardless of the country
the civil Code provides the following: wherein said property may be found”,

Private International Law Page 3


shall prevail. However, the Conflict of Law of Section 1 of the rule provides:
Texas, which is the “national law” of the
testatrix, Linnie Jane Hodges, provide that the Wills proved and allowed in a
domiciliary law (Philippine law — see foreign country, according to the
paragraph 2, supra) should govern the laws of such country, may be
testamentary dispositions and successional allowed, filed, and recorded by the
rights over movables (personal properties), and proper Court of First Instance in
the law of the situs of the property (also the Philippines.
Philippine law as to properties located in the
Philippines) with regards immovable (real
properties). Thus applying the “Renvoi Section 2 provides:
Doctrine”, as approved and applied by our
Supreme Court in the case of “In The Matter Of When a copy of such will and the
The Testate Estate of Eduard E. Christensen”, allowance thereof, duly
G.R. No. L-16749, promulgated January 31, authenticated, is filed with a petition
1963, Philippine law should apply to the for allowance in the Philippines, by the
Will of Linnie Jane Hodges and to the executor or other person interested, in
successional rights to her estate insofar the court having jurisdiction, such court
as her movable and immovable assets in shall fix a time and place for the
the Philippines are concerned. We shall hearing, and cause notice thereof to be
not, at this stage, discuss what law should given as in case of an original will
govern the assets of Linnie Jane Hodges presented for allowance.
located in Oklahoma and Texas, because the
only assets in issue in this motion are those Section 3 provides:
within the jurisdiction of this motion Court in
the two above-captioned Special Proceedings.
If it appears at the hearing that
the will should be allowed in the
Nota bene: When can foreign law be given Philippines, the court shall so
judicial notice allow it, and a certificate of its
allowance, signed by the Judge,
1. If the court of the forum is familiar of and attested by the seal of the
the law courts, to which shall be attached a
2. Or it is within the actual knowledge of copy of the will, shall be filed and
the court recorded by the clerk, and the will shall
have the same effect as if originally
Suntay v. Suntay proved and allowed in such court.

Jose B. Suntay died intestate leaving properties The fact that the municipal district court
in the Philippines and a house in China. He is of Amoy, China, is a probate court must
survived by children from the 1st marriage and be proved. The law of China on procedure in
a child and his widow from the 2nd. Intestate the probate or allowance of wills must also be
proceedings were instituted. Thereafter the proved. The legal requirements for the
widow filed a petition for a probate of a will but execution of a valid will in China in 1931 should
was later denied when the will was lost after also be established by competent evidence.
the filing of said petition. On appeal, the There is no proof on these points.
petition was granted since there was
sufficiency to prove the loss of the will. In spite The unverified answers to the questions
of the fact that a commission from the probate propounded by counsel for the appellant
court was issued on 24 April 1937 for the to the Consul General of the Republic of
taking of the deposition of Go Toh, an attesting China objected to by counsel for the
witness to the will, on 7 February 1938 the appellee, are inadmissible, because apart
probate court denied a motion for continuance from the fact that the office of Consul
of the hearing sent by cablegram from China General does not qualify and make the
by the surviving widow and dismissed the person who holds it an expert on the
petition. In the meantime the Pacific War Chinese law on procedure in probate
supervened. After liberation, Silvino claimed to matters, if the same be admitted, the
have found a will by his father which was filed, adverse party would be deprived of his
recorded and probated in the Amoy district right to confront and cross-examine the
court, Province of Fookien, China and thus filed witness. Consuls are appointed to attend to
a petition in the intestate proceedings praying trade matters.
for the probate of the will.
The order of the municipal district court
Issue: of Amoy, China, does not purport to
probate or allow the will which was the
May a will filed, recorded, and probated in subject of the proceedings. In view thereof,
China be reprobated in the Philippines? the will and the alleged probate thereof cannot
be said to have been done in accordance with
the accepted basic and fundamental concepts
Held: and principles followed in the probate and
allowance of wills. Consequently, the
As to the will claimed to have been executed authenticated transcript of proceedings held in
on 4 January 1931 in Amoy, China, the law on the municipal district court of Amoy, China,
the point in Rule 78. cannot be deemed and accepted as

Private International Law Page 4


proceedings leading to the probate or State of Illinois and the will was not in
allowance of a will and, therefore, the will conformity with the laws of that State.
referred to therein cannot be allowed, filed and
recorded by a competent court of this country. Held:

Likewise, the proceedings had in the Emil Johnson was a citizen of the State of
municipal district court of Amoy were for Illinois. In the absence of clear proof to the
the purpose of taking the testimony of contrary it should be presumed that a person
two attesting witnesses to the will and
naturalized in a court of a certain State thereby
that the order of the municipal district
court of Amoy does not purport to becomes a citizen of that State as well as of
probate the will. In the absence of proof that the United States.
the municipal district court of Amoy is a
probate court and on the Chinese law of In Section 625 of the Code of Civil
procedure in probate matters, it may be Procedure it is declared that "the
presumed that the proceedings in the matter of allowance by the court of a will of real or
probating or allowing a will in the Chinese personal property shall be conclusive as
courts are the a deposition or to a perpetuation to its due execution."
of testimony, and even if it were so it does not
measure same as those provided for in our
The due execution of a will involves conditions
laws on the subject. It is a proceedings in
relating to a number of matters, such as the
rem and for the validity of such
age and mental capacity of the testator, the
proceedings personal notice or by
signing of the document by the testator, or by
publication or both to all interested
someone in his behalf, and the
parties must be made. The interested
acknowledgment of the instrument by him in
parties in the case were known to reside
the presence of the required number of
in the Philippines. The evidence shows
witnesses who affix their signatures to the will
that no such notice was received by the
to attest the act. The proof of all these
interested parties residing in the
requisites is involved in the probate; and
Philippines. The proceedings had in the
as to each and all of them the probate is
municipal district court of Amoy, China, may be
conclusive. Our reported cases do not contain
likened to or come up to the standard of such
the slightest intimation that a will which has
proceedings in the Philippines for lack of notice
been probated according to law, and without
to all interested parties and the proceedings
fraud, can be annulled, in any other proceeding
were held at the back of such interested
whatever, on account of any supposed
parties.
irregularity or defect in the execution of the will
or on account of any error in the action of the
The decree appealed from is affirmed, without court upon the proof adduced before it.
pronouncement as to costs.
We are not unmindful of the fact that when a
In re: Johnson citizen of one State leaves it and takes up his
abode in another State with no intention of
Emil H. Johnson, a native of Sweden and a returning, he immediately acquires citizenship
naturalized citizen of the United States, died in in the State of his new domicile. This is in
the city of Manila, leaving a holographic will accordance with that provision of the
Fourteenth Amendment to the Constitution of
and is signed by himself and two witnesses
the United States which says that every citizen
only, instead of three witnesses required by of the United States is a citizen of the State
section 618 of the Code of Civil Procedure. where in he resides. The effect of this
However, a petition was presented in the Court provision necessarily is that a person
of First Instance of the city of Manila for the transferring his domicile from one State
probate of this will, on the ground that Johnson to another loses his citizenship in the
State of his original above upon acquiring
was at the time of his death a citizen of the
citizenship in the State of his new abode.
State of Illinois, United States of America; that The acquisition of the new State citizenship
the will was duly executed in accordance with extinguishes the old. That situation, in our
the laws of that State; and hence could opinion, has no analogy to that which
properly be probated here. Petitioner contends arises when a citizen of an American
that the decedent is not a citizen of Illinois and State comes to reside in the Philippine
prays to annul the decree of probate and put Islands. Here he cannot acquire a new
citizenship; nor by the mere change of
the estate into intestate administration, thus
domicile does he lose that which he
preparing the way for the establishment of the brought with him.
claim of the petitioner as the sole legitimate
heir of her father. Proper rule in taking judicial notice: The
proper rule is to require proof of the statutes of
Issue: the States of the American Union whenever
their provisions are determinative of the issues
Whether or not judgment from which the in any action litigated in the Philippine courts.
petitioner seeks relief should be set aside
because the testator was not a resident of the Collector vs. Fisher

Private International Law Page 5


Facts: Internal Revenue Code which the ancillary
administrator averred was allowable by
This case relates to the determination and way of the reciprocity granted by Section
settlement of the hereditary estate left by the 122 of the National Internal Revenue
deceased Walter G. Stevenson, and the laws Code, as then held by the Board of Tax
applicable thereto. Walter G. Stevenson (born Appeals in case No. 71 entitled "Housman vs.
in the Philippines on August 9, 1874 of British Collector," August 14, 1952; and (2)
parents and married in the City of Manila on exemption from the imposition of estate
January 23, 1909 to Beatrice Mauricia and inheritance taxes on the 210,000
Stevenson another British subject) died on shares of stock in the Mindanao Mother
February 22, 1951 in San Francisco, Lode Mines, Inc. also pursuant to the
California, U.S.A. whereto he and his wife reciprocity proviso of Section 122 of the
moved and established their permanent National Internal Revenue Code. In this last
residence since May 10, 1945. In his will return, the estate claimed that it was liable
executed in San Francisco on May 22, 1947, only for the amount of P525.34 for estate tax
and which was duly probated in the Superior and P238.06 for inheritance tax and that, as a
Court of California on April 11, 1951, consequence, it had overpaid the
Stevenson instituted his wife Beatrice as government. The refund of the amount of
his sole heiress to the following real and P15,259.83, allegedly overpaid, was
personal properties acquired by the accordingly requested by the estate. The
spouses while residing in the Philippines. Collector denied the claim. For this reason,
Ancillary administration proceedings were action was commenced in the Court of First
instituted in the Court of First Instance of Instance of Manila by respondents, as
Manila for the settlement of the estate in the assignees of Beatrice Mauricia Stevenson, for
Philippines. In due time Stevenson's will was the recovery of said amount. Pursuant to
duly admitted to probate by our court and Ian Republic Act No. 1125, the case was forwarded
Murray Statt was appointed ancillary to the Court of Tax Appeals which court, after
administrator of the estate, filed a preliminary hearing, rendered decision :
estate and inheritance tax return with the that: (a) the one-half (½) share of the
reservation of having the properties declared surviving spouse in the conjugal partnership
therein finally appraised at their values six property as diminished by the obligations
months after the death of Stevenson. properly chargeable to such property should
Preliminary return was made by the ancillary be deducted from the net estate of the
administrator in order to secure the waiver of deceased Walter G. Stevenson, pursuant to
the Collector of Internal Revenue on the Section 89-C of the National Internal Revenue
inheritance tax due on the 210,000 shares of Code; (b) the intangible personal property
stock in the Mindanao Mother Lode Mines Inc. belonging to the estate of said Stevenson
which the estate then desired to dispose in the is exempt from inheritance tax, pursuant to
United States. Acting upon said return, the the provision of section 122 of the National
Collector of Internal Revenue accepted the Internal Revenue Code in relation to the
valuation of the personal properties declared California Inheritance Tax Law but decedent's
therein, but increased the appraisal of the two estate is not entitled to an exemption of
parcels of land located in Baguio City by fixing P4,000.00 in the computation of the estate tax;
their fair market value. After allowing the (c) for purposes of estate and inheritance
deductions claimed by the ancillary taxation the Baguio real estate of the spouses
administrator for funeral expenses in the should be valued at P52,200.00, and 210,000
amount of P2,000.00 and for judicial and shares of stock in the Mindanao Mother Lode
administration expenses in the sum of Mines, Inc. should be appraised at P0.38 per
P5,500.00, the Collector assessed the state the share; and (d) the estate shall be entitled to a
amount of P5,147.98 for estate tax and deduction of P2,000.00 for funeral expenses
P10,875,26 or inheritance tax, or a total of and judicial expenses of P8,604.39.
P16,023.23. Both of these assessments were
paid by the estate. Issue: whether or not foreign law needs to be
The ancillary administrator filed in proved in our jurisdiction?
amended estate and inheritance tax
return in pursuance of his reservation Ruling:
made at the time of filing of the
preliminary return and for the purpose of It is well-settled that foreign laws do not
availing of the right granted by section 91 prove themselves in our jurisdiction and
of the National Internal Revenue Code. our courts are not authorized to take
Beatrice Mauricia Stevenson assigned all her judicial notice of them. Like any other fact,
rights and interests in the estate to the they must be alleged and proved. Section 41,
spouses, Douglas and Bettina Fisher, Rule 123 of our Rules of Court prescribes the
respondents herein. manner of proving foreign laws before our
The ancillary administrator filed a second tribunals. However, although we believe it
amended estate and inheritance tax return. desirable that these laws be proved in
This return declared the same assets of the accordance with said rule, we held in the case
estate stated in the amended return of of Willamette Iron and Steel Works v.
September 22, 1952, except that it contained Muzzal, 61 Phil. 471, that "a reading of sections
new claims for additional exemption and 300 and 301 of our Code of Civil Procedure
deduction to wit: (1) deduction in the (now section 41, Rule 123) will convince one
amount of P4,000.00 from the gross that these sections do not exclude the
estate of the decedent as provided for in presentation of other competent evidence to
Section 861 (4) of the U.S. Federal prove the existence of a foreign law." In that

Private International Law Page 6


case, we considered the testimony of an the death of the testator. Since no right to
attorney-at-law of San Francisco, share in the inheritance in favor of a
California who quoted verbatim a section divorced wife exists in the State of
of California Civil Code and who stated Nevada and since the court below had already
that the same was in force at the time the found that there was no conjugal property
obligations were contracted, as sufficient between the testator and Magdalena C.
evidence to establish the existence of Bohanan, the latter can now have no longer
said law. In line with this view, we find no claim to pay portion of the estate left by the
error, therefore, on the part of the Tax Court in testator.
considering the pertinent California law as
proved by respondents' witness. Issue:

Phil Trust Company vs. Bohanan whether the estementary dispositions,


especially those for the children which are
Facts: short of the legitime given them by the Civil
Code of the Philippines, are valid?
Appeal against an order of the Court of First
Instance of Manila, Hon. Ramon San Jose, Ruling:
presiding, dismissing the objections filed by
Magdalena C. Bohanan, Mary Bohanan and The old Civil Code, which is applicable to
Edward Bohanan to the project of partition this case because the testator died in
submitted by the executor and approving the 1944, expressly provides that
said project. successional rights to personal property
The Court of First Instance of Manila, Hon. are to be earned by the national law of
Rafael Amparo, presiding, admitted to probate the person whose succession is in question.
a last will and testament of C. O. Bohanan, Says the law on this point:
executed by him in Manila. In the said order, Nevertheless, legal and testamentary
the court made the following findings: successions, in respect to the order of
According to the evidence of the opponents the succession as well as to the extent of the
testator was born in Nebraska and successional rights and the intrinsic validity of
therefore a citizen of that state, or at their provisions, shall be regulated by the
least a citizen of California where some of national law of the person whose succession is
his properties are located. This contention in question, whatever may be the nature of the
is untenable. Notwithstanding the long property and the country in which it is found.
residence of the decedent in the Philippines, (par. 2, Art. 10, old Civil Code, which is the
his stay here was merely temporary, and he same as par. 2 Art. 16, new Civil Code.)
continued and remained to be a citizen of the In the proceedings for the probate of the will, it
United States and of the state of his pertinent was found out and it was decided that the
residence to spend the rest of his days in that testator was a citizen of the State of Nevada
state. His permanent residence or domicile in because he had selected this as his domicile
the United States depended upon his personal and his permanent residence. It is not disputed
intent or desire, and he selected Nevada as his that the laws of Nevada allow a testator to
homicide and therefore at the time of his dispose of all his properties by will. It does not
death, he was a citizen of that state. Nobody appear that at time of the hearing of the
can choose his domicile or permanent project of partition, the above-quoted provision
residence for him. That is his exclusive was introduced in evidence, as it was the
personal right. executor's duly to do. The law of Nevada,
Wherefore, the court finds that the testator C. being a foreign law can only be proved in
O. Bohanan was at the time of his death a our courts in the form and manner
citizen of the United States and of the State of provided for by our Rules, which are as
Nevada and declares that his will and follows:
testament, is fully in accordance with the laws SEC. 41. Proof of public or official record.
of the state of Nevada and admits the same to — An official record or an entry therein,
probate. Accordingly, the Philippine Trust when admissible for any purpose, may be
Company, named as the executor of the will, is evidenced by an official publication
hereby appointed to such executor and upon thereof or by a copy attested by the
the filing of a bond in the sum of P10,000.00. officer having the legal custody of the
The executor filed a project of partition dated record, or by his deputy, and accompanied, if
January 24, 1956, making adjudications, in the record is not kept in the Philippines, with a
accordance with the provisions of the will. certificate that such officer has the
The wife Magadalena C. Bohanan and her custody. . . . (Rule 123).
two children question the validity of the
testamentary provisions disposing of the We have, however, consulted the records of
estate in the manner above indicated, the case in the court below and we have found
claiming that they have been deprived of that during the hearing on October 4, 1954 of
the legitime that the laws of the forum the motion of Magdalena C. Bohanan for
concede to them. withdrawal of P20,000 as her share, the foreign
Moreover, the court below had found that the law, especially Section 9905, Compiled
testator and Magdalena C. Bohanan were Nevada Laws was introduced in evidence
married on January 30, 1909, and that by appellant's counsel. Again said laws
divorce was granted to him on May 20, 1922; presented by the counsel for the executor
that sometime in 1925, Magdalena C. and admitted by the Court during the
Bohanan married Carl Aaron and this hearing of the case on before Judge
marriage was subsisting at the time of Rafael Amparo.

Private International Law Page 7


In addition, the other appellants, children of Makati. As aforesaid, the lower court
the testator, do not dispute the above-quoted ruled in favor of petitioners.
provision of the laws of the State of Nevada. On appeal, the respondent Court of Appeals
Under all the above circumstances, we are held that moral damages are recoverable in a
constrained to hold that the pertinent law of damage suit predicated upon a breach of
Nevada, especially Section 9905 of the contract of carriage only where there is fraud
Compiled Nevada Laws of 1925, can be or bad faith. Since it is a matter of record
taken judicial notice of by us, without that overbooking of flights is a common
proof of such law having been offered at and accepted practice of airlines in the
the hearing of the project of partition. United States and is specifically allowed
As in accordance with Article 10 of the old Civil under the Code of Federal Regulations by
Code, the validity of testamentary dispositions the Civil Aeronautics Board, no fraud nor
are to be governed by the national law of the bad faith could be imputed on respondent
testator, and as it has been decided and it is TransWorld Airlines.
not disputed that the national law of the
testator is that of the State of Nevada, already Ruling:
indicated above, which allows a testator to
dispose of all his property according to his will, That there was fraud or bad faith on the
as in the case at bar, the order of the court part of respondent airline when it did not
approving the project of partition made in allow petitioners to board their flight for
accordance with the testamentary provisions, Los Angeles in spite of confirmed tickets
must be, as it is hereby affirmed, with costs cannot be disputed. The U.S. law or
against appellants. regulation allegedly authorizing overbooking
has never been proved. Foreign laws do not
Zalamea vs. CA prove themselves nor can the courts take
judicial notice of them. Like any other fact,
Facts: they must be alleged and proved. Written
law may be evidenced by an official
Petitioners-spouses Cesar C. Zalamea and publication thereof or by a copy attested
Suthira Zalamea, and their daughter, Liana by the officer having the legal custody of
Zalamea, purchased three (3) airline tickets the record, or by his deputy, and
from the Manila agent of respondent accompanied with a certificate that such officer
TransWorld Airlines, Inc. for a flight to New has custody. The certificate may be made
York to Los Angeles. by a secretary of an embassy or legation,
The tickets of petitioners-spouses were consul general, consul, vice-consul, or
purchased at a discount of 75% while that of consular agent or by any officer in the
their daughter was a full fare ticket. All three foreign service of the Philippines
tickets represented confirmed reservations. stationed in the foreign country in which
Petitioners received notice of the the record is kept, and authenticated by
reconfirmation of their reservations for said the seal of his office. 7
flight. On the appointed date, however, Respondent TWA relied solely on the statement
petitioners checked in at 10:00 a.m., an hour of Ms. Gwendolyn Lather, its customer service
earlier than the scheduled flight at 11:00 a.m. agent, in her deposition dated January 27,
but were placed on the wait-list because the 1986 that the Code of Federal Regulations of
number of passengers who had checked in the Civil Aeronautics Board allows overbooking.
before them had already taken all the seats Aside from said statement, no official
available on the flight. Liana Zalamea publication of said code was presented as
appeared as the No. 13 on the wait-list while evidence. Thus, respondent court's finding that
the two other Zalameas were listed as "No. 34, overbooking is specifically allowed by the US
showing a party of two." Out of the 42 names Code of Federal Regulations has no basis in
on the wait list, the first 22 names were fact.
eventually allowed to board the flight to Los Even if the claimed U.S. Code of Federal
Angeles, including petitioner Cesar Zalamea. Regulations does exist, the same is not
The two others, on the other hand, at No. 34, applicable to the case at bar in
being ranked lower than 22, were not able to accordance with the principle of lex loci
fly. As it were, those holding full-fare tickets contractus which require that the law of
were given first priority among the wait-listed the place where the airline ticket was
passengers. Mr. Zalamea, who was holding the issued should be applied by the court
full-fare ticket of his daughter, was allowed to where the passengers are residents and
board the plane; while his wife and daughter, nationals of the forum and the ticket is
who presented the discounted tickets were issued in such State by the defendant
denied boarding. airline. 8 Since the tickets were sold and
Even in the next TWA flight to Los Angeles Mrs. issued in the Philippines, the applicable law in
Zalamea and her daughter, could not be this case would be Philippine law.
accommodated because it was also fully Existing jurisprudence explicitly states that
booked. Thus, they were constrained to book in overbooking amounts to bad faith, entitling the
another flight and purchased two tickets from passengers concerned to an award of moral
American Airlines at a cost of Nine Hundred damages. In Alitalia Airways v. Court of
Eighteen ($918.00) Dollars. Appeals, 9 where passengers with confirmed
Upon their arrival in the Philippines, bookings were refused carriage on the last
petitioners filed an action for damages minute, this Court held that when an airline
based on breach of contract of air issues a ticket to a passenger confirmed on a
carriage before the Regional Trial Court of particular flight, on a certain date, a contract of
carriage arises, and the passenger has every

Private International Law Page 8


right to expect that he would fly on that flight credit to the genuineness of a document in a
and on that date. If he does not, then the foreign country.
carrier opens itself to a suit for breach of With respect to proof of written laws, parol
contract of carriage. Where an airline had proof is objectionable, for the written law itself
deliberately overbooked, it took the risk of is the best evidence. According to the weight of
having to deprive some passengers of their authority, when a foreign statute is involved,
seats in case all of them would show up for the the best evidence rule requires that it be
check in. For the indignity and inconvenience proved by a duly authenticated copy of the
of being refused a confirmed seat on the last statute.
minute, said passenger is entitled to an award At this juncture, we have to point out that the
of moral damages. Venezuelan law was not pleaded before the
lower court.
Wild Valley Shipping Co. Vs. CA A foreign law is considered to be pleaded
if there is an allegation in the pleading
Facts: about the existence of the foreign law, its
import and legal consequence on the
The Philippine Roxas, a vessel owned by event or transaction in issue.
Philippine President Lines, Inc., private A review of the Complaint revealed that it was
respondent herein, arrived in Puerto Ordaz, never alleged or invoked despite the fact that
Venezuela, to load iron ore. the grounding of the M/V Philippine Roxas
Upon the completion of the loading and when occurred within the territorial jurisdiction of
the vessel was ready to leave port, an official Venezuela.
pilot of Venezuela, was designated by the We reiterate that under the rules of private
harbour authorities in Puerto Ordaz to navigate international law, a foreign law must be
the Philippine Roxas through the Orinoco River. properly pleaded and proved as a fact. In the
The Philippine Roxas experienced some absence of pleading and proof, the laws of a
vibrations when it entered the San Roque foreign country, or state, will be
Channel. The vessel proceeded on its way, with presumed to be the same as our own local
the pilot assuring the watch officer that the or domestic law and this is known as
vibration was a result of the shallowness of the processual presumption.
channel.
The master (captain) checked the position of Board of Commissioners v. Dela Rosa
the vessel and verified that it was in the centre
of the channel. Facts:
The Philippine Roxas ran around in the Orinoco
River, thus obstructing the ingress and egress On July 12, 1960, Santiago Gatchalian,
of vessels. grandfather of William Gatchalian, was
As a result of the blockage, the Malandrinon, a recognized by the Bureau of Immigration as a
vessel owned by herein petitioner Wildvalley native born Filipino citizen following the
Shipping Company, Ltd., was unable to sail out citizenship of natural mother Mariana
of Puerto Ordaz on that day. Gatchalian. On June 27, 1961, Willian, then
Subsequently, Wildvalley Shipping Company, twelve years old, arrives in Manila from
Ltd. filed a suit with the Regional Trial Court of Hongkong together with a daughter and a son
Manila, Branch III against Philippine President of Santiago. They had with them certificate of
Lines, Inc. and Pioneer Insurance Company (the registration and identity issued by the
underwriter/insurer of Philippine Roxas) for Philippine consulate in Hongkong based on a
damages in the form of unearned profits, and cablegram bearing the signature of the
interest thereon amounting to US $400,000.00 secretary of foreign affairs, Felixberto Serrano,
plus attorney's fees, costs, and expenses of and sought admission as Filipino citizens.
litigation.
On July 6, 1961, the board of special inquiry
Issue: whether or not Venezuelan law is admitted the Gatchalians as Filipino citizens
applicable to the case at bar? and issued an identification certificate to
William. The board of commissioners was
Ruling: directed by the Secretary of Justice to Review
all cases where entry was granted on the
It is well-settled that foreign laws do not ground that the entrant was a Filipino citizen
prove themselves in our jurisdiction and such included the case of William. As a result of
our courts are not authorized to take the decision of the board of special inquiry
judicial notice of them. Like any other fact, which recommended for the reversal of the
they must be alleged and proved. decision of the Board of Commissioners. Acting
For a copy of a foreign public document to be commissioner issued an order affirming the
admissible, the following requisites are decision of the Board of Special Inquiry.
mandatory: (1) It must be attested by the
officer having legal custody of the records On August 15, 1990, the Commission on
or by his deputy; and (2) It must be Immigration and Deportatiion ordered the
accompanied by a certificate by a arrest of William and was released upon
secretary of the embassy or legation, posting P 200,000 cash bond. Thus on the 29th
consul general, consul, vice consular or of the same month, he filed a petition for
consular agent or foreign service officer, certiorari and prohibition before the RTC of
and with the seal of his office. The latter Manila. A motion to dismiss was filed but
requirement is not a mere technicality but is denied.
intended to justify the giving of full faith and
Issue:

Private International Law Page 9


the law of the country where it
Whether or not William Gatchalian is to be is stipulated.
declared as a Filipino citizen
However, intestate and
Held: testamentary successions, both
with respect to the order of
William Gatchalian is declared as a Filipino succession and to the amount
Citizen. Having declared the assailed marriage of successional rights and to
as valid, respondent William Gatchalian follows the intrinsic validity of
the citizenship of his father, a Filipino as testamentary provisions, shall
legitimate child. Respondent belongs to a class be regulated by the national
of Filipinos who are citizens of the Philippines law of the person whose
at the time of the adoption of the constitution. succession is under
consideration, whatever may
In Moy Ya Lim vs. Commissioner of Immigration be the nature of the property
(41 SCRA 292 [1971]) and in Lee vs. and regardless of the country
Commissioner of Immigration (supra), this wherein said property may be
Court declared that: found. (10a)
(e)verytime the citizenship of a person is
material or indispensable in a judicial or Art. 17. The forms and solemnities of
administrative case, whatever the contracts, wills, and other
corresponding court or administrative authority public instruments shall be
decides therein as to such citizenship is governed by the laws of the
generally not considered as res adjudicata, country in which they are
hence it has to be threshed out again and executed.
again as the occasion may demand.
When the acts referred to are
An exception to the above rule was laid by this executed before the diplomatic
Court in Burca vs. Republic (51 SCRA 248 or consular officials of the
[1973]), viz: Republic of the Philippines in a
foreign country, the solemnities
We declare it to be a sound rule that where the established by Philippine laws
citizenship of a party in a case is definitely shall be observed in their
resolved by a court or by an administrative execution.
agency, as a material issue in the controversy,
after a full-blown hearing with the active Prohibitive laws concerning
participation of the Solicitor General or his persons, their acts or property,
authorized representative, and this finding or and those which have, for their
the citizenship of the party is affirmed by this object, public order, public
Court, the decision on the matter shall policy and good customs shall
constitute conclusive proof of such party's not be rendered ineffective by
citizenship in any other case or proceeding. But laws or judgments
it is made clear that in no instance will a promulgated, or by
decision on the question of citizenship in such determinations or conventions
cases be considered conclusive or binding in agreed upon in a foreign
any other case or proceeding, unless obtained country. (11a)
in accordance with the procedure herein
stated.
Art. 71. All marriages performed
Thus, in order that the doctrine of res judicata outside the Philippines in
may be applied in cases of citizenship, the accordance with the laws in
following must be present: 1) a person's force in the country where they
citizenship must be raised as a material issue were performed, and valid
in a controversy where said person is a party; there as such, shall also be
2) the Solicitor General or his authorized
valid in this country, except
representative took active part in the
resolution thereof, and 3) the finding or bigamous, polygamous, or
citizenship is affirmed by this Court. incestuous marriages as
determined by Philippine law.
Doctrine of processual presumption (19a)

Civil Code Art. 124. If the marriage is between a


citizen of the Philippines and a
Art. 15. Laws relating to family rights foreigner, whether celebrated
and duties, or to the status, in the Philippines or abroad,
condition and legal capacity of the following rules shall prevail:
persons are binding upon
citizens of the Philippines, even
though living abroad. (9a) 1. If the husband is a citizen
of the Philippines while the
wife is a foreigner, the
Art. 16. Real property as well as provisions of this Code
personal property is subject to shall govern their relations;

Private International Law Page 10


2. If the husband is a attestation must be under the official seal of
foreigner and the wife is a the attesting officer, if there be any, or if he be
citizen of the Philippines, the clerk of a court having a seal, under the
the laws of the husband's
seal of such court. (26a)
country shall be followed,
without prejudice to the
Rule 130
provisions of this Code with
regard to immovable
property. (1325a) Section 45. Commercial lists and the like. —
Evidence of statements of matters of interest
Art. 815. When a Filipino is in a foreign to persons engaged in an occupation contained
country, he is authorized to in a list, register, periodical, or other published
make a will in any of the forms compilation is admissible as tending to prove
established by the law of the the truth of any relevant matter so stated if
country in which he may be. that compilation is published for use by
Such will may be probated in persons engaged in that occupation and is
the Philippines. (n)
generally used and relied upon by them
therein. (39)
Art. 816. The will of an alien who is
abroad produces effect in the Section 46. Learned treatises. — A published
Philippines if made with the
treatise, periodical or pamphlet on a subject of
formalities prescribed by the
law of the place in which he history, law, science, or art is admissible as
resides, or according to the tending to prove the truth of a matter stated
formalities observed in his therein if the court takes judicial notice, or a
country, or in conformity with witness expert in the subject testifies, that the
those which this Code writer of the statement in the treatise,
prescribes. (n)
periodical or pamphlet is recognized in his
profession or calling as expert in the subject.
Art. 818. Two or more persons cannot (40a)
make a will jointly, or in the
same instrument, either for
How to acquire nationality
their reciprocal benefit or for
1. by birth
the benefit of a third person.
2. by repatriation
(669)
3. by naturalization
4. by subrogation and cession
Art. 819. Wills, prohibited by the
preceding article, executed by how to lose a nationality
Filipinos in a foreign country 1. release
shall not be valid in the 2. deprivation
Philippines, even though 3. expiration
authorized by the laws of the 4. renunciation
country where they may have
been executed. (733a) domicile
1. intent to stay
Art. 829. A revocation done outside the 2. physical presence
Philippines, by a person who 3. conduct indicative of such intention
does not have his domicile in
this country, is valid when it is Rep. Act No. 9225
done according to the law of signed into law by President Gloria M. Arroyo
the place where the will was on August 29, 2003
made, or according to the law
of the place in which the SECTION 1. Short Title.-This Act shall be known
testator had his domicile at the as the "Citizenship Retention and Reacquisition
time; and if the revocation Act of 2003."
takes place in this country,
when it is in accordance with SEC. 2. Declaration of Policy.-It is hereby
the provisions of this Code. (n) declared the policy of the State that all
Philippine citizens who become citizens of
Art. 1039. Capacity to succeed is another country shall be deemed not to
governed by the law of the
have lost their Philippine citizenship
nation of the decedent. (n)
under the conditions of this Act.
Rule 132 Sec. 25 SEC. 3. Retention of Philippine Citizenship.-Any
provision of law to the contrary
What attestation of copy must state. —
notwithstanding, natural-born citizens of the
Whenever a copy of a document or record is
Philippines who have lost their Philippine
attested for the purpose of evidence, the
citizenship by reason of their
attestation must state, in substance, that the
naturalization as citizens of a foreign
copy is a correct copy of the original, or a
country are hereby deemed to have
specific part thereof, as the case may be. The

Private International Law Page 11


reacquired Philippine citizenship upon
taking the following oath of allegiance to
the Republic: 4. Those intending to practice their
profession in the Philippines shall apply
"I ___________________________, solemnly swear with the proper authority for a license or
(or affirm) that I will support and defend the permit to engage in such practice; and
Constitution of the Republic of the Philippines
and obey the laws and legal orders 5. That right to vote or be elected or
promulgated by the duly constituted appointed to any public office in the
authorities of the Philippines; and I hereby Philippines cannot be exercised by, or
declare that I recognize and accept the extended to, those who:
supreme authority of the Philippines and will
maintain true faith and allegiance thereto; and a. are candidates for or are occupying
that I impose this obligation upon myself any public office in the country of
voluntarily without mental reservation or which they are naturalized citizens;
purpose of evasion." and/or

Natural-born citizens of the Philippines b. are in the active service as


who, after the effectivity of this Act, commissioned or noncommissioned
become citizens of a foreign country shall officers in the armed forces of the
retain their Philippine citizenship upon country which they are naturalized
taking the aforesaid oath. citizens.

SEC. 4. Derivative Citizenship. - The SEC. 6. Separability Clause. - If any section or


unmarried child, whether legitimate, provision of this Act is held unconstitutional or
illegitimate or adopted, below eighteen invalid, any other section or provision not
(18) years of age, of those who reacquire affected thereby shall remain valid and
Philippine citizenship upon effectivity of effective.
this Act shall be deemed citizens of the
SEC. 7. Repealing Clause. - All laws, decrees,
Philippines.
orders, rules and regulations inconsistent with
SEC. 5. Civil and Political Rights and Liabilities. the provisions of this Act are hereby repealed
- Those who retain or reacquire Philippine or modified accordingly.
citizenship under this Act shall enjoy full civil
SEC. 8. Effectivity Clause. - This Act shall take
and political rights and be subject to all
effect after fifteen (15) days following its
attendant liabilities and responsibilities under
publication in the Official Gazette or two (2)
existing laws of the Philippines and the
newspapers of general circulation.
following conditions:
ARTICLE IV
1. Those intending to exercise their right
of suffrage must meet the requirements CITIZENSHIP
under Section 1, Article V of the
Constitution, Republic Act No. 9189, Section 1. The following are citizens of the
otherwise known as "The Overseas Philippines:
Absentee Voting Act of 2003" and other
existing laws; 1. Those who are
citizens of the
2. Those seeking elective public office in Philippines at the time
the Philippines shall meet the qualifications of the adoption of this
for holding such public office as required Constitution;
by the Constitution and existing laws and,
at the time of the filing of the certificate of 2. Those whose
candidacy, make a personal and sworn fathers or mothers are
renunciation of any and all foreign citizens of the
citizenship before any public officer Philippines;
authorized to administer an oath;
3. Those born
3. Those appointed to any public office before January 17,
1973, of Filipino
shall subscribe and swear to an oath
mothers, who elect
of allegiance to the Republic of the
Philippine citizenship
Philippines and its duly constituted
upon reaching the age
authorities prior to their assumption
of majority; and
of office: Provided, That they renounce
their oath of allegiance to the country
where they took that oath;

Private International Law Page 12


4. Those who are themselves with a certificate of
naturalized in legal capacity to contract
accordance with law. marriage, to be issued by their
respective diplomatic or
Section 2. Natural-born citizens are those consular officials. (13a)
who are citizens of the
Philippines from birth Art. 1039. Capacity to succeed is
without having to perform governed by the law of the
any act to acquire or nation of the decedent. (n)
perfect their Philippine
citizenship. Those who elect Art. 50. For the exercise of civil rights
Philippine citizenship in and the fulfillment of civil
accordance with paragraph (3), obligations, the domicile of
Section 1 hereof shall be natural persons is the place
deemed natural-born citizens. of their habitual residence.
(40a)
Section 3. Philippine citizenship may be
lost or reacquired in the manner provided by Art. 51. When the law creating or
law. recognizing them, or any other
provision does not fix the
Section 4. Citizens of the Philippines who domicile of juridical persons,
marry aliens shall retain the same shall be understood
their citizenship, unless by to be the place where their
their act or omission, they are legal representation is
deemed, under the law, to established or where they
have renounced it. exercise their principal
functions. (41a)
Section 5. Dual allegiance of citizens is
inimical to the national Art. 99. No person shall be entitled to a
interest and shall be dealt legal separation who has not
with by law. resided in the Philippines for
one year prior to the filing of
NCC Art. 15. Laws relating to family rights the petition, unless the cause
and duties, or to the status, for the legal separation has
condition and legal capacity of taken place within the territory
persons are binding upon of this Republic. (Sec. 2a, Act
citizens of the Philippines, even No. 2710)
though living abroad. (9a)
Art. 1251. Payment shall be made in the
Art. 16. Real property as well as place designated in the
personal property is subject to obligation.
the law of the country where it
is stipulated. There being no express
stipulation and if the
However, intestate and undertaking is to deliver a
testamentary successions, both determinate thing, the
with respect to the order of payment shall be made
succession and to the amount wherever the thing might be at
of successional rights and to the moment the obligation was
the intrinsic validity of constituted.
testamentary provisions, shall
be regulated by the national In any other case the place of
law of the person whose payment shall be the domicile
succession is under of the debtor.
consideration, whatever may
be the nature of the property If the debtor changes his
and regardless of the country domicile in bad faith or after he
wherein said property may be has incurred in delay, the
found. (10a) additional expenses shall be
borne by him.
Art. 66. When either or both of the
contracting parties are citizens These provisions are without
or subjects of a foreign prejudice to venue under the
country, it shall be necessary, Rules of Court. (1171a)
before a marriage license can
be obtained, to provide

Private International Law Page 13


Art. 58. Save marriages of an taking the oath recognizes and accepts
exceptional character
the supreme authority of the Philippines
authorized in Chapter 2 of this
Title, but not those under is an unmistakable and categorical
Article 75, no marriage shall be affirmation of his undivided loyalty to the
solemnized without a license Republic.
first being issued by the local
civil registrar of the Issue:
municipality where either
contracting party habitually Whether R.A. 9225 is unconstitutional and
resides. (7a) whether the court

Art. 829. A revocation done outside the Held:


Philippines, by a person who
does not have his domicile R.A. 9225 is constitutional and that the Court
in this country, is valid has no jurisdiction yet to pass upon the issue of
when it is done according
dual allegiance. The court held that that the
to the law of the place
where the will was made, or intent of the legislature in drafting Rep.
according to the law of the Act No. 9225 is to do away with the
place in which the testator provision in Commonwealth Act No. 635
had his domicile at the time; which takes away Philippine citizenship
and if the revocation takes
from natural-born Filipinos who become
place in this country, when it is
in accordance with the naturalized citizens of other countries.
provisions of this Code. (n) What Rep. Act No. 9225 does is allow dual
citizenship to natural-born Filipino citizens who
Cases:
have lost Philippine citizenship by reason of
AASJS vs. Datumanong their naturalization as citizens of a foreign
country. On its face, it does not recognize
G.R. No. 160869, May 11, 2007
dual allegiance. By swearing to the
Sec. 3, RA 9225 stayed clear of the problem of supreme authority of the Republic, the
dual allegiance and shifted the burden of person implicitly renounces his foreign
confronting the issue of whether or not there is citizenship. Plainly, from Section 3, Rep.
dual allegiance to the concerned foreign
Act No. 9225 stayed clear out of the
country.
problem of dual allegiance and shifted
FACTS: the burden of confronting the issue of
whether or not there is dual allegiance to
Petitioner filed the instant petition against the concerned foreign country. What
respondent, then Secretary of Justice Simeon happens to the other citizenship was not made
Datumanong, the official tasked to implement a concern of Rep. Act No. 9225.
laws governing citizenship in order to prevent
the Justice Secretary from implementing R. A. For its part, the OSG counters that pursuant to
9225. Petitioner argues that RA 9225 is Section 5, Article IV of the 1987 Constitution,
unconstitutional as it violates Sec. 5, Article VI dual allegiance shall be dealt with by law.
of the Constitution which states that dual Thus, until a law on dual allegiance is enacted
allegiance of citizens is inimical to national by Congress, the Supreme Court is without any
interest and shall be dealt with by law. The jurisdiction to entertain issues regarding dual
Office of the Solicitor General (OSG) claims that allegiance.
Section 2 merely declares as a state policy that
"Philippine citizens who become citizens of Moreover, Section 5, Article IV of the
another country shall be deemed not to have Constitution is a declaration of a policy
lost their Philippine citizenship." The OSG and it is not a self-executing provision.
further claims that the oath in Section 3 does The legislature still has to enact the law
not allow dual allegiance since the oath taken on dual allegiance. In Sections 2 and 3 of
by the former Filipino citizen is an effective Rep. Act No. 9225, the framers were not
renunciation and repudiation of his foreign concerned with dual citizenship per se,
citizenship. The fact that the applicant but with the status of naturalized citizens

Private International Law Page 14


who maintain their allegiance to their but also over the res, which is the
personal status of Baby Rose as well as
countries of origin even after their
that of petitioners herein.
naturalization. Congress was given a
mandate to draft a law that would set specific Art. 15 adheres to the theory that
parameters of what really constitutes dual jurisdiction over the status of a natural
person is determined by the latters'
allegiance. Until this is done, it would be
nationality. Pursuant to this theory, the Court
premature for the judicial department, has jurisdiction over the status of Baby Rose,
including this Court, to rule on issues she being a citizen of the Philippines, but not
pertaining to dual allegiance. over the status of the petitioners, who are
foreigners.
ELLIS V. REPUBLIC
7 SCRA 962 Under the Philippine’s political law, which
is patterned after the Anglo-American
Petitioner Marvin G. Ellis, a native of San legal system, the Court have, likewise,
Fransisco, California and is married to Gloria G. adopted the latter's view to the effect
Ellis in Banger, Maine, United States. Both are that personal status, in general, is
citizens of the United States. Baby Rose was determined by and/or subject to the
born on September 26, 1959. Four or five days jurisdiction of the domiciliary law
later, the mother of Rose left her with the (Restatement of the Law of Conflict of Laws, p.
Heart of Mary Villa — an institution for unwed 86; The Conflict of Laws by Beale, Vol. I, p. 305,
mothers and their babies — stating that she Vol. II, pp. 713-714).
(the mother) could not take of Rose without
bringing disgrace upon her (the mother's This, perhaps, is the reason why our Civil
family.). Code does not permit adoption by non-
resident aliens, and we have consistently
Being without issue, on November 22, 1959, refused to recognize the validity of
Mr. and Mrs. Ellis filed a petition with the Court foreign decrees of divorce — regardless of
of First Instance of Pampanga for the adoption the grounds upon which the same are
of the aforementioned baby. At the time of the based — involving citizens of the
hearing of the petition on January 14, 1960, Philippines who are not bona fide
petitioner Marvin G. Ellis and his wife had been residents of the forum, even when our laws
in the Philippines for three (3) years, he being authorized absolute divorce in the Philippines
assigned thereto as staff sergeant in the United (Ramirez v. Gmur, 42 Phil. 855; Gonayeb v.
States Air Force Base, in Angeles, Pampanga Hashim, 30 Phil. 22; Cousine Hix v. Fleumer, 55
where both lived at that time. They had been in Phil. 851; Barretto Gonzales v. Gonzales, 58
the Philippines before, or, to exact, in 1953. Phil. 67; Recto v. Harden, L-6897, Nov. 29,
1955)".
Issue:
Republic v. Maddela
Whether or not being permanent residents in
the Philippines, petitioners are qualified to 27 SCRA 702
adopt Baby Rose.
This is a petition to have the petitioners
Held: Miguela Tan Suat, and Chan Po Lan, all Chinese
Nationals, to be declared a Filipino citizens.
Inasmuch as petitioners herein are not
domiciled in the Philippines — and, hence, non- That sometime in the year 1937 Miguela was
resident aliens – the Court cannot assume and legally married to Sy Ing Seng, a Filipino
exercise jurisdiction over the status, under citizen; likewise, in the year 1961, Chan Po Lan
either the nationality theory or the domiciliary was legally married to Cu Bon Piao, a Filipino
theory. citizen; and that the petitioners have all the
qualifications and none of the disqualifications
Article 335 of the Civil Code of the Philippines, to become Filipino citizens.
provides that:
"The following cannot adopt: The court inquired from Fiscal Veluz, who
xxx xxx xxx
represents the Solicitor General, if he has any
(4) Non-resident aliens;".
xxx xxx xxx opposition to the petition to which the Fiscal
This legal provisions is too clear to require answered that he has no opposition. The Court
interpretation. had it announced to the public if there is any
opposition to the petitions of both to be
Since adoption is a proceedings in rem, declared a Filipino citizen and nobody in the
no court may entertain unless it has crowded courtroom registered his opposition.
jurisdiction, not only over the subject
matter of the case and over the parties,

Private International Law Page 15


As such, the petitioners Miguela Tan Suat and Petitioner seeks reconsideration of the decision
Chan Po Lan were declared a Filipino citizens in this case which reversed that of the Court of
by marriage and the Commissioner of First Instance of Leyte declaring her a citizen of
Immigration is hereby ordered to cancel the the Philippines, the said court have found her
necessary alien certificate of registration and to be married to a Filipino citizen and to
immigrant certificate of residence of the possess all the qualifications and none of the
petitioner and to issue the corresponding disqualifications to become Filipino citizen
identification card. enumerated in the Naturalization Law.

The Solicitor General filed the instant petitions Issue:


instead, including the Commissioner of
Immigration as co-petitioner in view of the fact Whether or not a court may grant a judicial
that the dispositive parts of the decisions of the declaration of citizenship.
lower court are addressed to him for
compliance. Held:

Issue: The Court cannot grant petitioner-appellee's


prayer for the affirmance of the trial court's
Whether or not a person claiming to be a judgment declaring her a Filipino citizen. It
citizen may get a judicial declaration of must be noted that the sole and only
citizenship. purpose of the petition is to have
petitioner declared a Filipino citizen.
Held: Under Philippine laws there can be no
judicial action or proceeding for the
Under Philippine laws, there can be no declaration of the citizenship of an
action or proceeding for the judicial individual. It is as an incident only of the
declaration of the citizenship of an adjudication of the rights of the parties to
individual. Courts of justice exist for the a controversy, that the courts may pass
settlement of justiciable controversies, which upon, and make a pronouncement
imply a given right, legally demandable and relative to, their status.
enforceable, an act or omission violative of said
right, and a remedy, granted or sanctioned by In Moy Ya Lim Yao, the Court emphasized
law, for said breach of right. the administrative procedure that needs
to be followed in the Bureau Immigration
As an incident only of the adjudication of regarding the steps to be taken by an
the right of the parties to a controversy, alien woman married to a Filipino for the
the court may pass upon, and make a cancellation of her alien certificate of
pronouncement relative to, their status. registration, and thus secure recognition
Otherwise, such a pronouncement is beyond of her status Filipino citizen. Such a
judicial power. Thus, for instance, no action or procedure could be availed of Petitioner.
proceeding may be instituted for a declaration Judicial recourse would be avoidable to
to the effect that plaintiff or petitioner is Petitioner in case of an adverse action by the
married, or single, or a legitimate child, Immigration Commissioner.
although a finding thereon may be made as a
necessary premise to justify a given relief However, if the decision of an
available only to one enjoying said status. At administrative agency on the question of
times, the law permits the acquisition of a citizenship, is affirmed by this Court on
given status, such as naturalization by judicial the ground that the same is supported by
decree. But there is no similar legislation substantial evidence on the whole record,
authorizing the institution of a judicial there appears to be no valid reason why
proceeding to declare that a given person is a such finding should have no conclusive
Filipino Citizen. (Tan v. Republic, L-14159, April effect in other cases, where the same
18, 1960). issue is involved. The same observation
holds true with respect to a decision of a court
Burca v. Republic on the matter of citizenship as a material
matter in issue in the case before it, which is
51 SCRA 248 affirmed by this Court. For the "effective
operation of courts in the social and economic
1st case: Burca not granted citizenship because
scheme requires that their decision have the
such power is granted under the executive
respect of and be observed by the parties, the
branch.
general public and the courts themselves.
Moya Lim Yao: alien wife deemed ipso facto According insufficient weight to prior decisions
Filipino citizen as long as no disqualifications, encourages disrespect and disregard of courts
no need to prove the qualifications; and their decisions and invites litigation"
(Clear, Res Judicata Reexamined, 57 Yale Law
Burca motion for reconsideration: Journal, 345).

Private International Law Page 16


Wherefore, the Court declared it to be a name except one, Rosa. She did not know the
sound rule, that where citizenship of a names of her brothers-in-law, or sisters-in-law.
party in a case is definitely resolved by a The Court of First Instance of Manila (Civil Case
court or by an administrative agency, as a 49705) denied the prayer for preliminary
material issue in controversy, after a full- injunction. Moya Lim Yao and Lau Yuen Yeung
blown hearing, with the act participation appealed.
of the Solicitor General or his authority
representative, and this finding on the Issue:
Citizenship of the party is affirmed by this
Court, the decision on the matter shows Whether Lau Yuen Yeung ipso facto became a
constitute conclusive proof of such Filipino citizen upon her marriage to a Filipino
person's citizenship, in another case or citizen.
proceeding. But it is made clear that in
Held:
instance will a decision on the question of
citizenship in such cases be considered Lau Yuen Yeung, was declared to have become
conclusive or binding in any other case a Filipino citizen from and by virtue of her
proceeding, unless obtained in accordance with marriage to Moy Ya Lim Yao al as Edilberto
the procedure herein stated. Aguinaldo Lim, a Filipino citizen of 25 January
1962.
Moy Ya Lim Yao v. Commissioner
Under Section 15 of Commonwealth Act
41 SCRA 292
473, an alien woman marrying a Filipino,
Facts: native born or naturalized, becomes ipso
facto a Filipina provided she is not
On 8 February 1961, Lau Yuen Yeung disqualified to be a citizen of the
applied for a passport visa to enter the Philippines under Section 4 of the same
Philippines as a non-immigrant. In the law. Likewise, an alien woman married to
interrogation made in connection with her an alien who is subsequently naturalized
application for a temporary visitor's visa to here follows the Philippine citizenship of
enter the Philippines, she stated that she her husband the moment he takes his
was a Chinese residing at Kowloon, oath as Filipino citizen, provided that she
Hongkong, and that she desired to take a does not suffer from any of the
pleasure trip to the Philippines to visit disqualifications under said Section 4.
her great-grand-uncle Lau Ching Ping for a Whether the alien woman requires to undergo
period of one month. She was permitted to the naturalization proceedings, Section 15 is a
come into the Philippines on 13 March parallel provision to Section 16. Thus, if the
1961, and was permitted to stay for a widow of an applicant for naturalization as
period of one month which would expire on Filipino, who dies during the proceedings, is not
13 April 1961. On the date of her arrival, Asher required to go through a naturalization
Y, Cheng filed a bond in the amount of proceedings, in order to be considered as a
P1,000.00 to undertake, among others, that Filipino citizen hereof, it should follow that the
said Lau Yuen Yeung would actually depart wife of a living Filipino cannot be denied the
from the Philippines on or before the expiration same privilege. This is plain common sense
of her authorized period of stay in this country and there is absolutely no evidence that the
or within the period as in his discretion the Legislature intended to treat them differently.
Commissioner of Immigration or his authorized As the laws of our country, both
representative might properly allow. After substantive and procedural, stand today,
repeated extensions, Lau Yuen Yeung was there is no such procedure (a substitute
allowed to stay in the Philippines up to 13 for naturalization proceeding to enable
February 1962. On 25 January 1962, she the alien wife of a Philippine citizen to
contracted marriage with Moy Ya Lim Yao have the matter of her own citizenship
alias Edilberto Aguinaldo Lim an alleged settled and established so that she may
Filipino citizen. Because of the contemplated not have to be called upon to prove it
action of the Commissioner of Immigration to everytime she has to perform an act or
confiscate her bond and order her arrest and enter into a transaction or business or
immediate deportation, after the expiration of exercise a right reserved only to
her authorized stay, she brought an action for Filipinos), but such is no proof that the
injunction with preliminary injunction. At the citizenship is not vested as of the date of
hearing which took place one and a half years marriage or the husband's acquisition of
after her arrival, it was admitted that Lau Yuen citizenship, as the case may be, for the truth is
Yeung could not write either English or that the situation obtains even as to native-
Tagalog. Except for a few words, she could not born Filipinos. Everytime the citizenship of a
speak either English or Tagalog. She could not person is material or indispensible in a judicial
name any Filipino neighbor, with a Filipino or administrative case, Whatever the
corresponding court or administrative authority

Private International Law Page 17


decides therein as to such citizenship is oath of allegiance incident
generally not considered as res adjudicata, thereto, states that he does so
hence it has to be threshed out again and only in connection with his
again as the occasion may demand. service to said foreign country:
And provided, finally, That any
Nota bene: Filipino citizen who is rendering
service to, or is commissioned
There are two laws, which govern the Loss of in, the armed forces of a
Philippine citizenship. These are foreign country under any of
Commonwealth Act No. 63 and the circumstances mentioned
Commonwealth Act No. 473. The former in paragraph (a) or (b), shall
applies to both natural-born and naturalized not be permitted to participate
citizens and the latter applies only to nor vote in any election of the
naturalized citizens. Republic of the Philippines
during the period of his service
As stated in Commonwealth Act. No. 63,
to, or commission in, the
A Filipino citizen may lose his citizenship in any armed forces of said foreign
of the following ways and/or events: country. Upon his discharge
from the service of the said
1. By naturalization in a foreign country; foreign country, he shall be
automatically entitled to the
2. By express renunciation of full enjoyment of his civil and
political rights as a Filipino
citizenship;
citizen;
3. By subscribing to an oath of
allegiance to support the constitution or
laws of a foreign country upon attaining
5. By cancellation of the of the
twenty-one years of age or more:
certificates of naturalization;
Provided, however, That a Filipino may not
divest himself of Philippine citizenship in
6. By having been declared by competent
any manner while the Republic of the
authority, a deserter of the Philippine
Philippines is at war with any country;
armed forces in time of war, unless
subsequently, a plenary pardon or
4. By rendering services to, or
amnesty has been granted; and
accepting commission in, the armed
forces of a foreign country: Provided,
7. In the case of a woman, upon her
That the rendering of service to, or the
marriage to a foreigner if, by virtue
acceptance of such commission in, the
of the laws in force in her husband's
armed forces of a foreign country, and the
country, she acquires his nationality.
taking of an oath of allegiance incident
thereto, with the consent of the Republic Commonwealth Act No. 473, Section 18 of
of the Philippines, shall not divest a said law provides that:
Filipino of his Philippine citizenship if
either of the following circumstances is A naturalization certificate may be cancelled by
present: a competent judge on any of the following
grounds:

1. If it is shown that said naturalization


a. The Republic of the certificate was obtained
Philippines has a defensive fraudulently or illegally;
and/or offensive pact of
alliance with the said foreign 2. If the person naturalized shall, within
country; or five years next following the
issuance of said naturalization
b. The said foreign country certificated, return to his native
maintains armed forces on country or to some foreign country
Philippine territory with the and establish his permanent
consent of the Republic of residence there: Provided, That the
the Philippines: Provided, fact of the person naturalized
That the Filipino citizen remaining for more than one year in his
concerned, at the time of native country or the country of his
rendering said service, or former nationality, or two years in any
acceptance of said other foreign country, shall be
commission, and taking the considered prima facie evidence of his

Private International Law Page 18


intention of taking up his permanent taking of said oath. On that same date,
residence in the same; petitioner took it and the certificate of
naturalization was issued to him.
3. If the petition was made on an
invalid declaration of intention The Government seasonably gave notice of its
intention to appeal from said order of February
4. If it is shown that the minor children 9, 1966 and filed its record on appeal among
of the person naturalized failed to the grounds that the oath was taken prior to
graduate from a public high school judgment having been final and executor.
recognized by the Office of Private
Issue:
Education of the Philippines, where
Philippine history, government and
Is the oath valid?
civics are taught as part of the school
curriculum, through the fault of their Whether or not a permission to renounce
parents either by neglecting to support citizenship is necessary from the Minister of
them or by transferring them to the Interior of Nationalist China.
another school or schools. A certified
copy of the decree of naturalization Held:
certificate shall be forwarded by the
Clerk of Court of the Department of First issue:
Interior and the Bureau of Justice.
The order of February 9, 1966 (oath-
5. If it is shown that the naturalized taking) had not — and up to the present
citizen has allowed himself to be has not — become final and executory in
used as a dummy in violation of view of the appeal duly taken by the
the constitutional provisions Government.
requiring Philippine citizenship as
2nd Issue:
a requisite for the exercise, use or
enjoyment of a right, franchise or
It is argued that the permission is not
privilege.
required by our laws and that the
naturalization of an alien, as a citizen of
The main reason why a decision in a
the Philippines, is governed exclusively
naturalization proceeding is not res
by such laws and cannot be controlled by
judicata is because such is not a judicial
any foreign law. However, the question of
adversarial proceeding. Similarly, estoppel
how a Chinese citizen may strip himself of
or laches cannot apply to the government in
that status is necessarily governed —
action for the cancellation of a certificate of
pursuant to Articles 15 and 16 of our Civil
naturalization, since it is a known principle that
Code — by the laws of China, not by those
the government is never estopped by the
of the Philippines.
mistakes on the part of its agents.
As a consequence, a Chinese national cannot
However, according to the Constitution,
be naturalized as a citizen of the Philippines,
marriage to an alien would not
unless he has complied with the laws of
automatically divest a person of his
Nationalist China requiring previous permission
citizenship, unless he or she performs
of its Minister of the Interior for the
certain acts or omission which would
renunciation of nationality.
result to the loss of his or her citizenship.
This provision, however, is not retroactive;
• Section 12 of Commonwealth Act No.
thus, does not repatriate those who lost their
473 provides, however, that before the
Philippine citizenship by marriage under the
naturalization certificate is issued, the
1935 and 1973 Constitutions.
petitioner shall "solemnly swear," inter
Oh Hek How v. Republic alia, that he renounces "absolutely and
forever all allegiance and fidelity to any
29 SCRA 94 foreign prince, potentate" and
particularly to the state "of which" he is
Petitioner Oh Hek How having been granted "a subject or citizen." The obvious
naturalization through his petition filed a purpose of this requirement is to divest
motion alleging that he had complied with the him of his former nationality, before
requirements of Republic Act No. 530 and acquiring Philippine citizenship,
praying that he be allowed to take his oath of because, otherwise, he would have two
allegiance as such citizen and issued the nationalities and owe allegiance to two
corresponding certificate of naturalization. (2) distinct sovereignties, which our
laws do not permit, except that,
The Court of First Instance of Zamboanga del pursuant to Republic Act No. 2639, "the
Norte issued forthwith an order authorizing the acquisition of citizenship by a natural-

Private International Law Page 19


born Filipino citizen from one of the sometime in the year 1870, when the
Iberian and any friendly democratic Philippines was under Spanish rule, and
Ibero-American countries shall not that San Carlos, Pangasinan, his place of
produce loss or forfeiture of his residence upon his death in 1954, in the
Philippine citizenship, if the law of that absence of any other evidence, could
country grants the same privilege to its have well been his place of residence
citizens and such had been agreed before death, such that Lorenzo Pou
upon by treaty between the Philippines would have benefited from the "en masse
and the foreign country from which Filipinization" that the Philippine Bill had
citizenship is acquired." effected in 1902. That citizenship (of
Lorenzo Pou), if acquired, would thereby
Tecson v. Comelec extend to his son, Allan F. Poe, father of
respondent FPJ. The 1935 Constitution,
424 SCRA 277 during which regime respondent FPJ has seen
first light, confers citizenship to all persons
Facts:
whose fathers are Filipino citizens regardless of
On 31 December 2003, Ronald Allan Kelly whether such children are legitimate or
Poe, also known as Fernando Poe, Jr. illegitimate.
(FPJ), filed his certificate of candidacy for
the position of President of the Republic
Board of Commissioners v. Dela Rosa
of the Philippines under the Koalisyon ng Facts:
Nagkakaisang Pilipino (KNP) Party, in the 2004
national elections. In his certificate of On July 12, 1960, Santiago Gatchalian,
candidacy, FPJ, representing himself to be a grandfather of William Gatchalian, was
natural-born citizen of the Philippines, recognized by the Bureau of Immigration as a
native born Filipino citizen following the
stated his name to be "Fernando Jr.," or
citizenship of natural mother Mariana
"Ronald Allan" Poe, his date of birth to be Gatchalian. On June 27, 1961, Willian, then
20 August 1939 and his place of birth to twelve years old, arrives in Manila from
be Manila. Victorino X. Fornier, initiated, on 9 Hongkong together with a daughter and a son
January 2004, a petition before the Commission of Santiago. They had with them certificate of
on Elections (COMELEC) to disqualify FPJ and to registration and identity issued by the
deny due course or to cancel his certificate of Philippine consulate in Hongkong based on a
cablegram bearing the signature of the
candidacy upon the thesis that FPJ made a
secretary of foreign affairs, Felixberto Serrano,
material misrepresentation in his and sought admission as Filipino citizens.
certificate of candidacy by claiming to be
a natural-born Filipino citizen when in On August 15, 1990, the Commission on
truth, according to Fornier, his parents Immigration and Deportatiion ordered the
were foreigners; his mother, Bessie Kelley arrest of William and was released upon
Poe, was an American, and his father, posting P 200,000 cash bond. Thus on the 29th
of the same month, he filed a petition for
Allan Poe, was a Spanish national, being
certiorari and prohibition before the RTC of
the son of Lorenzo Pou, a Spanish Manila. A motion to dismiss was filed but
subject. And even if Allan F. Poe was a Filipino denied.
citizen, he could not have transmitted his
Filipino citizenship to FPJ, the latter being an Petitioners, claim that respondent is an alien.
illegitimate child of an alien mother. Fornier In support of their position, petitioners point
based the allegation of the illegitimate birth of out that Santiago Gatchalian's marriage with
Chu Gim Tee in China as well as the marriage
FPJ on two assertions: (1) Allan F. Poe
of Francisco (father of William) Gatchalian to
contracted a prior marriage to a certain Paulita Ong Chiu Kiok, likewise in China, were not
Gomez before his marriage to Bessie Kelley supported by any evidence other than their
and, (2) even if no such prior marriage had own self-serving testimony nor was there any
existed, Allan F. Poe, married Bessie Kelly only showing what the laws of China were. It is the
a year after the birth of FPJ. postulate advanced by petitioners that for the
said marriages to be valid in this country, it
Issue: should have been shown that they were valid
by the laws of China wherein the same were
contracted. There being none, petitioners
Whether FPJ was a natural born citizen, so as to
conclude that the aforesaid marriages
be allowed to run for the offcie of the President cannot be considered valid. Hence,
of the Philippines. Santiago's children, including Francisco,
followed the citizenship of their mother,
Held: having been born outside of a valid
marriage. Similarly, the validity of the
Any conclusion on the Filipino citizenship of Francisco's marriage not having been
Lorenzo Pou could only be drawn from the demonstrated, William and Johnson followed
presumption that having died in 1954 at 84 the citizenship of their mother, a Chinese
years old, Lorenzo would have been born national.

Private International Law Page 20


Issue: Having declared the assailed marriages
as valid, respondent William Gatchalian
Whether or not William Gatchalian is to be follows the citizenship of his father
declared as a Filipino citizen Francisco, a Filipino, as a legitimate child
of the latter. Francisco, in turn is likewise a
Held: Filipino being the legitimate child of Santiago
Gatchalian who (the latter) is admittedly a
In Miciano vs. Brimo (50 Phil. 867 [1924]; Lim Filipino citizen whose Philippine citizenship was
and Lim vs. Collector of Customs, 36 Phil. 472; recognized by the Bureau of Immigration in an
Yam Ka Lim vs. Collector of Customs, 30 Phil. order dated July 12, 1960.
46 [1915]), this Court held that in the
absence of evidence to the contrary, Finally, respondent William Gatchalian belongs
foreign laws on a particular subject are to the class of Filipino citizens who became as
presumed to be the same as those of the such at the time of the adoption of the
Philippines. In the case at bar, there being no Constitution. . . .
proof of Chinese law relating to marriage,
there arises the presumption that it is the Nota bene: for Gatchalian
same as that of Philippine law.
In Moy Ya Lim vs. Commissioner of Immigration
The lack of proof of Chinese law on the matter (41 SCRA 292 [1971]) and in Lee vs.
cannot be blamed on Santiago Gatchalian Commissioner of Immigration (supra), this
much more on respondent William Gatchalian Court declared that:
who was then a twelve-year old minor. The fact
is, as records indicate, Santiago was not Everytime the citizenship of a person is
pressed by the Citizenship Investigation material or indispensable in a judicial or
Board to prove the laws of China relating administrative case, whatever the
to marriage, having been content with corresponding court or administrative authority
the testimony of Santiago that the decides therein as to such citizenship is
Marriage Certificate was lost or destroyed generally not considered as res
during the Japanese occupation of China. adjudicata, hence it has to be threshed out
again and again as the occasion may demand.
The testimonies of Santiago Gatchalian and
Francisco Gatchalian before the Philippine An exception to the above rule was laid by this
consular and immigration authorities regarding Court in Burca vs. Republic (51 SCRA 248
their marriages, birth and relationship to each [1973]), viz:
other are not self-serving but are admissible in
evidence as statements or declarations We declare it to be a sound rule that
regarding family reputation or tradition in where the citizenship of a party in a case
matters of pedigree (Sec. 34, Rule 130). is definitely resolved by a court or by an
administrative agency, as a material issue
Philippine law, following the lex loci in the controversy, after a full-blown
celebrationis, adheres to the rule that a hearing with the active participation of the
marriage formally valid where celebrated Solicitor General or his authorized
is valid everywhere. Referring to marriages representative, and this finding or the
contracted abroad, Art. 71 of the Civil Code citizenship of the party is affirmed by this
(now Art. 26 of the Family Code) provides that Court, the decision on the matter shall
"all marriages performed outside of the constitute conclusive proof of such party's
Philippines in accordance with the laws in force citizenship in any other case or proceeding.
in the country where they were performed, and But it is made clear that in no instance will a
valid there as such, shall also be valid in this decision on the question of citizenship in such
country . . ." cases be considered conclusive or binding in
any other case or proceeding, unless obtained
And any doubt as to the validity of the in accordance with the procedure herein
matrimonial unity and the extent as to how far stated.
the validity of such marriage may be extended
to the consequences of the coverture is Thus, in order that the doctrine of res judicata
answered by Art. 220 of the Civil Code in this may be applied in cases of citizenship, the
manner: "In case of doubt, all following must be present: 1) a person's
presumptions favor the solidarity of the citizenship must be raised as a material
family. Thus, every intendment of law or issue in a controversy where said person
facts leans toward the validity of is a party; 2) the Solicitor General or his
marriage, the indissolubility of the authorized representative took active
marriage bonds, the legitimacy of part in the resolution thereof, and 3) the
children, the community of property finding or citizenship is affirmed by this
during marriage, the authority of parents Court.
over their children, and the validity of
defense for any member of the family in NUVAL VS. GURRAY
case of unlawful aggression." (Emphasis
supplied). Bearing in mind the "processual Facts:
presumption" enunciated in Miciano and other
cases, he who asserts that the marriage is not
valid under our law bears the burden of proof
• This appeal was taken by the petitioner
to present the foreign law. Gregorio Nuval from the judgment of
the Court of First Instance of La Union,

Private International Law Page 21


upholding the defense of res judicata is Poblacion, Balaoan, La Union;"
and dismissing the quo warranto and in order to be registered in the
proceedings instituted by the said subscribed affidavit Exhibit F-1 before
Gregorio Nuval against Norbeto Guray the board of election inspectors of
and others, with costs against the precinct No. 1 of Balaoan, by virtue of
petitioner. which he was registered as an elector
of the said precinct, having made use
• Gregorio Nuval filed, in his dual of the right of suffrage in said
capacity as a voter duly qualified and municipality in the general elections of
registered in the election list of the 1925. In his cedula certificates issued
municipality of Luna and as a duly by himself as municipal treasurer of
registered candidate for the office of Balaoan from the year 1923 to 1928,
municipal president of said included, he made it appear that his
municipality, a petition against residence was the residential district of
Norberto Guray asking for the Balaoan. In the year 1926, his wife and
exclusion of his name from the children who, up to that time, had lived
election list of said municipality, in the municipality of Balaoan, went
not being a qualified voter of said back to live in the town of Luna in the
municipality and he had not resided house of his wife's parents, due to the
therein for six months as required high cost of living in that municipality.
by section 431 of the said Norberto Guray used to go home to
Administrative Code. Luna in the afternoons after office
hours, and there he passed the nights
• Norbeto Guray was elected to the with his family. His children studied in
office of municipal president of the public school of Luna. In January,
Luna by a plurality of votes, 1927, he commenced the
Gregorio Nuval obtaining second place. construction of a house of strong
materials in Luna, which has not
• Gregorio Nuval filed the present action yet been completed, and neither
of quo warranto asking that Norberto be nor his family has lived in it. On
Guray be declared ineligible had a February 1, 1928, Norberto Guray
legal residence of one year applied for and obtained vacation leave
previous to the election as required to be spent in Luna, and on the 16th of
by section 2174 of the said the same month he filed his resignation
Administrative Code in order to be by telegraph, which was accepted on
eligible to an elective municipal office. the same day, also by telegraph.
Nothwithstanding that he was already
• Norberto Guray had resided in the provided with a cedula by himself as
municipality of Luna, his municipal treasurer of Balaoan on
birthplace, where he had married January 31, 1928, declaring him
and had held the office of resident of said town, he obtained
municipal treasurer. On that date he another cedula from the municipality of
was appointed municipal treasurer of Luna on February 20, 1928, which was
Balaoan, Province of La Union. The dated January 15, 1928, in which it is
rules of the provincial treasurer of La presented that he resided in the barrio
Union, to which Norberto Guray was of Victoria, municipality of Luna,
subject as such municipal treasurer, Province of La Union. On February 23,
require that municipality 1928, Norberto Guray applied for and
treasurers live continuously in the obtained the cancellation of his name
municipality where they perform they in the election list of the municipality of
official duties, in order to be able to Balaoan, and on April 14, 1928, he
give an account of their acts as such applied for registration as a voter in
treasurers at any time. In order to Luna, alleging that he had been
qualify and be in a position to vote as residing in said municipality for thirty
an elector in Balaoan in the general years. For this purpose he made of the
election of 1925, Norberto Guray asked cedula certificate antedated.
for the cancellation of his name in the
election lists of Luna, where he had
voted in the general elections of 1922,
alleging as a ground therefore the
following: "On the ground of Issue:
transfer of any residence which
took place on the 28th day of June, Whether or not Norberto Guray had the legal
1922. My correct and new address residence of one year immediately prior to the

Private International Law Page 22


general elections of June 5, 1928, in order to be • That on September 9, 1931, an income
eligible to the office of municipal president of tax return for the fractional period from
Luna, Province of La Union? January 1, 1931 to June 30, 1931, was
also prepared by the Bureau of Internal
Revenue for the estate of the said
Ruling: deceased Arthur Graydon Moody.
• That on December 3, 1931, the
It is an established rule that "where a voter committee on claims and appraisals
abandons his residence in a state and filed with the court its report.
acquires one in another state, he cannot • That on November 4, 1931, and in
again vote in the state of his former answer to the letter mentioned in the
residence until he has qualified by a new preceding paragraph, the Bureau of
period of residence" (20 Corpus Juris, p. 71, Internal Revenue addressed to the
par. 28). "The term 'residence' as so used attorney for Ida M. Palmer another
is synonymous with 'domicile,' which letter, copy of which marked Exhibit NN
imports not only intention to reside in a is hereto attached and made a part
fixed place, but also personal presence in hereof.
that place, coupled with conduct • That the estate of the late Arthur
indicative of such intention." (People vs. Graydon Moody paid under protest the
Bender, 144 N. Y. S., 145.) sum of P50,000 on July 22, 1931, and
the other sum of P40,019.75 on
Since Norberto Guray abandoned his first January 19, 1932, making assessment
residence in the municipality of Luna and for inheritance tax and the sum of
acquired another in Balaoan, in order to vote P13,001.41 covers the assessment for
and be a candidate in the municipality of Luna, income tax against said estate.
he needed to reacquire residence in the latter • That on January 21, 1932, the Collector
municipality for the length of time prescribed of Internal Revenue overruled the
by the law, and for such purpose, he needed protest made by Ida M. Palmer through
not only the intention to do so, but his her attorney.
personal presence in said municipality. • The parties reserve their right to
introduce additional evidence at the
VELILLA VS. POSADA hearing of the present case.
• Manila, August 15, 1933.
Facts: • In addition to the foregoing agreed
statement of facts, both parties
introduced oral and documentary
• That Arthur Graydon Moody died in
evidence from which it appears that
Calcutta, India, on February 18, 1931.
Arthur G. Moody, an American citizen,
• That Arthur Graydon Moody executed came to the Philippine Islands in 1902
in the Philippine Islands a will, by or 1903 and engaged actively in
virtue of which will, he bequeathed business in these Islands up to the time
all his property to his only sister, of his death in Calcutta, India, on
Ida M. Palmer, who then was and still is February 18, 1931. He had no business
a citizen and resident of the State of elsewhere and at the time of his death
New York, United States of America. left an estate consisting principally of
• That on February 24,1931, a petition bonds and shares of stock of
for appointment of special corporations organized under the laws
administrator of the estate of the of the Philippine Islands, bank deposits
deceased Arthur Graydon Moody was and other intangibles and personal
filed by W. Maxwell Thebaut with the property valued by the commissioners
Court of First Instance of Manila. of appraisal and claims at P609,767.58
• That subsequently or on April 10, 1931, and by the Collector of Internal
a petition to the will of the deceased Revenue for the purposes of
Arthur Graydon Moody, and the same inheritance tax at P653,657.47. All of
was, after hearing, duly probated by said property at the time of his
the court in a decree dated May 5, death was located and had its
1931. situs within the Philippine Islands.
So far as this record shows, he left
• That on July 14, 1931, Ida M. Palmer
no property of any kind located
was declared to be the sole and
anywhere else. In his will,he made a
only heiress of the deceased Arthur
statement that: Arthur G. Moody, a
Graydon Moody
citizen of the United States of America,
• That the property left by the late residing in the Philippine Islands,
Arthur Graydon Moody consisted hereby publish and declare the
principally of bonds and shares of stock following as my last Will and Testament
of corporations organized under the . . ..
laws of the Philippine Islands, bank
deposits and other personal properties.
Ruling:
• That on July 22, 1931, the Bureau of
Internal Revenue prepared for the
estate of the late Arthur Graydon To effect the abandonment of one's
Moody an inheritance tax return. domicile, there must be a deliberate and
provable choice of a new domicile,
coupled with actual residence in the place

Private International Law Page 23


chosen, with a declared or provable 'residence' requirement in cases of
intent that it should be one's fixed and naturalization, has already been
permanent place of abode, one's home. interpreted to mean the actual or
There is a complete dearth of evidence in the
constructive permanent home
record that Moody ever established a new
domicile in a foreign country. otherwise known as legal residence or
domicile (Wilfredo Uytengsu vs. Republic
of the Philippines, 95 Phil. 890). A place in
Finding no merit in any of the assignments of
error of the appellant, the court affirm the a country or state where he lives and stays
judgment of the trial court, first, because the permanently, and to which he intends to
property in the estate of Arthur G. Moody at return after a temporary absence, no
the time of his death was located and had its matter how long, is his domicile. In other
situs within the Philippine Islands and, second, words domicile is characterized by
because his legal domicile up to the time
animus manendi. So an alien who has
of his death was within the Philippine
Islands. been admitted into this country as a
temporary visitor, either for business or
pleasure, or for reasons of health, though
actually present in this country cannot be
UJANO VS. REPUBLIC said to have established his domicile here
because the period of his stay is only
Facts: temporary in nature and must leave when
the purpose of his coming is accomplished.
In the present case, petitioner, who is
presently a citizen of the United
Petitioner seeks to reacquire his States of America, was admitted into
Philippine citizenship in a petition this country as a temporary visitor, a
filed before the Court of First Instance status he has maintained at the time
of Ilocos Sur. Petitioner was born 66 of the filing of the present petition for
years ago of Filipino parents in Magsingal reacquisition of Philippine citizenship
Ilocos Sur. He is married to Maxima O. and which continues up to the
Ujano with whom he has one son, Prospero, present. Such being the case, he has not
who is now of legal age. He left the complied with the specific requirement of
Philippines for the United States of law regarding six months residence before
America in 1927 where after a filing his present petition."
residence of more than 20 years he
acquired American citizenship by CAASI VS. COURT OF APPEALS
naturalization. He returned to the
Philippines on November 10, 1960 to which Facts:
he was admitted merely for a temporary
stay. He owns an agricultural land and • These two cases were consolidated
a residential house situated in because they have the same objective;
Magsingal, Ilocos Sur. He receives a the disqualification under Section 68 of
monthly pension from the Social Security the Omnibus Election Code of the
Administration of the United States of private respondent, Merito Miguel for
America. He has no record of conviction the position of municipal mayor of
and it is his intention to renounce his Bolinao, Pangasinan, to which he was
allegiance to the U.S.A. After hearing, the elected in the local elections of January
court a quo rendered decision denying the 18, 1988, on the ground that he is a
petition on the ground that petitioner did green card holder, hence, a permanent
not have the residence required by resident of the United States of
law six months before he filed his petition America, not of Bolinao.
for reacquisition of Philippine citizenship.
• In his answer to both petitions, Miguel
admitted that he holds a green card
Ruling: issued to him by the US Immigration
Service, but he denied that he is a
permanent resident of the United
States. He allegedly obtained the green
The court a quo, in denying the petition, card for convenience in order that he
made the following comment: "One of the may freely enter the United States for
qualifications for reacquiring his periodic medical examination and
Philippine citizenship is that the to visit his children there. He alleged
applicant 'shall have resided in the that he is a permanent resident of
Philippines at least six months before Bolinao, Pangasinan, that he voted in
he applies for naturalization' [Section all previous elections, including the
3(1), Commonwealth Act No. 63]. This plebiscite on February 2,1987 for the

Private International Law Page 24


ratification of the 1987 Constitution, (residence and domicile, for purposes of
and the congressional elections on May election laws are synonymous…)
18,1987.
The mischief which this provision —
• After hearing the consolidated petitions reproduced verbatim from the 1973
before it, the COMELEC dismissed the Constitution — seeks to prevent is the
petitions on the ground that: The possibility of a "stranger or newcomer
possession of a green card by the unacquainted with the conditions and needs of
respondent (Miguel) does not
a community and not identified with the latter,
sufficiently establish that he has
abandoned his residence in the from an elective office to serve that
Philippines. On the contrary, inspite community."
(sic) of his green card, Respondent has
sufficiently indicated his intention to Petitioner Imelda Romualdez-Marcos filed her
continuously reside in Bolinao as Certificate of Candidacy for the position of
shown by his having voted in Representative of the First District of Leyte
successive elections in said with the Provincial Election Supervisor on
municipality. As the respondent meets
March 8, 1995.
the basic requirements of citizenship
and residence for candidates to
elective local officials (sic) as provided Private respondent Cirilo Roy Montejo, the
for in Section 42 of the Local incumbent Representative of the First District
Government Code, there is no legal of Leyte and a candidate for the same position,
obstacle to his candidacy for mayor of filed a "Petition for Cancellation and
Bolinao, Pangasinan. (p. 12, Rollo, G.R. Disqualification" with the Commission on
No. 84508). Elections alleging that petitioner did not meet
the constitutional requirement for residency. In
his petition, private respondent contended that
Mrs. Marcos lacked the Constitution's one year
Issues: (1) whether or not a green card is proof residency requirement for candidates for the
that the holder is a permanent resident of the House of Representatives.
United States, and
Held:
(2) whether respondent Miguel had waived his
status as a permanent resident of or immigrant So settled is the concept (of domicile) in our
to the U.S.A. prior to the local elections on election law that in these and other election
January 18, 1988.
law cases, this Court has stated that the mere
absence of an individual from his
Ruling: Despite his vigorous disclaimer,
permanent residence without the
Miguel's immigration to the United States in
1984 constituted an abandonment of his intention to abandon it does not result in
domicile and residence in the Philippines. For a loss or change of domicile.
he did not go to the United States merely to
visit his children or his doctor there; he entered It stands to reason therefore, that petitioner
the limited States with the intention to have merely committed an honest mistake in jotting
there permanently as evidenced by his the word "seven" in the space provided for the
application for an immigrant's (not a visitor's or residency qualification requirement. It would
tourist's) visa. Based on that application of his,
be plainly ridiculous for a candidate to
he was issued by the U.S. Government the
requisite green card or authority to reside deliberately and knowingly make a statement
there permanently. in a certificate of candidacy which would lead
to his or her disqualification.
To be "qualified to run for elective office" in the
Philippines, the law requires that the candidate Residence in the civil law is a material fact,
who is a green card holder must have "waived referring to the physical presence of a person
his status as a permanent resident or in a place. A person can have two or more
immigrant of a foreign country." Therefore, his residences, such as a country residence and a
act of filing a certificate of candidacy for city residence. Residence is acquired by living
elective office in the Philippines, did not of
in place; on the other hand, domicile can exist
itself constitute a waiver of his status as a
permanent resident or immigrant of the United without actually living in the place. The
States. The waiver of his green card should be important thing for domicile is that, once
manifested by some act or acts independent of residence has been established in one place,
and done prior to filing his candidacy for there be an intention to stay there
elective office in this country. Without such permanently, even if residence is also
prior waiver, he was "disqualified to run for any established in some other place.
elective office" (Sec. 68, Omnibus Election
Code).
For political purposes the concepts of residence
and domicile are dictated by the peculiar
MARCOS vs COMELEC criteria of political laws. As these concepts
have evolved in our election law, what

Private International Law Page 25


has clearly and unequivocally emerged is examination by a competent lady physician
the fact that residence for election to determine her physical capacity for
purposes is used synonymously with copulation.
6. Defendant did not submit herself to the
domicile.
examination and the court entered a
decree annulling the marriage.
In Nuval vs. Guray, the Court held that
7. The City Attorney filed a Motion for
"the term residence. . . is synonymous Reconsideration, among the grounds that
with domicile which imports not only the defendant’s impotency has not been
intention to reside in a fixed place, but satisfactorily established as required by
also personal presence in that place, law; that she had not been physically
coupled with conduct indicative of such examined because she refused to be
examined.
intention." Larena vs. Teves reiterated the
same doctrine in a case involving the ISSUE:
qualifications of the respondent therein to the
post of Municipal President of Dumaguete, Whether or not the marriage may be annulled
Negros Oriental. Faypon vs. Quirino, held that on the strength only of the lone testimony of
the absence from residence to pursue studies the husband who claimed and testified that his
or practice a profession or registration as a wife is impotent.
voter other than in the place where one is
elected does not constitute loss of residence. HELD:
So settled is the concept (of domicile) in our
election law that in these and other election
law cases, this Court has stated that the mere The law specifically enumerates the legal
absence of an individual from his permanent grounds that must be proved to exist by
residence without the intention to abandon it indubitable evidence to annul a marriage. In
does not result in a loss or change of domicile. the case at bar, the annulment of the marriage
in question was decreed upon the sole
In Co vs. Electoral Tribunal of the House of
testimony of the husband who was expected to
Representatives, this Court concluded that the
give testimony tending or aiming at securing
framers of the 1987 Constitution obviously
the annulment of his marriage he sought and
adhered to the definition given to the term
seeks. Whether the wife is really impotent
residence in election law, regarding it as
cannot be deemed to have been satisfactorily
having the same meaning as domicile.
established because from the commencement
of the proceedings until the entry of the decree
A Person cannot have 2 domiciles. As long
she had abstained from taking part therein.
as the Domicile was not lost, it continues
to be the same until replaced by a new
Although her refusal to be examined or failure
one. Marcos did not overtedly abandon
to appear in court show indifference on her
her domicile since even if living in
part, yet from such attitude the presumption
Malakanyang, she constantly goes home
arising out of the suppression of evidence
to her domicile.
could not arise or be inferred because women
of this country are by nature coy, bashful and
Prior to this:
shy and would not submit to a physical
examination unless compelled to by competent
The civil code provides that the wife follows the
authority.
domicile of her husband.

A physical examination in this case is not self-


JIMENEZ V. REPUBLIC
109 PHIL 273 incriminating. She is not charged with any
offense . She is not being compelled to be a
FACTS: witness against herself.

“Impotency being an abnormal condition


1. Plaintiff Joel Jimenez filed a complaint should not be presumed. The
praying of a decree annulling his marriage presumption is in favor of potency.” The
with Remedios Canizares.
lone testimony of the husband that his wife is
2. He claimed that the orifice of her
genitals was too small to allow the physically incapable of sexual intercourse is
penetration of a male organ or penis for insufficient to tear asunder the ties that have
copulation. bound them together as husband and wife.
3. He also claimed that the condition of her
genitals existed at the time of marriage Recto v. Harden
and continues to exist. 100 Phil 427
4. The wife was summoned and served
with a copy of the complaint but she did Facts:
not file an answer.
5. The court entered an order requiring Recto and Harden entered into a contract
defendant to submit to a physical for professional services wherein the latter

Private International Law Page 26


engaged the services of the former as her (See 5 Am. Jur. 359 et seq; Ballentine, Law
counsel against her husband for a claim in their Dictionary, 2nd ed., p. 276.)
conjugal property. Mr. Harden previously filed
for divorce against the Mrs. The Court awarded
Mrs. Harden an amount totaling to almost 4 The third objection is not borne out, either by
million pesos plus litis expensae. the language of the contract between them, or
by the intent of the parties thereto. Its
Subsequently however, Mrs. Harden ordered purpose was not to secure a divorce, or to
her counsel to vacate all orders and facilitate or promote the procurement of
judgments rendered therein, and abandon a divorce. It merely sought to protect the
and nullify all her claims to the conjugal interest of Mrs. Harden in the conjugal
partnership existing between her and Mr. partnership, during the pendency of a divorce
Harden. Later, she entered into an amicable suit she intended to file in the United States.
settlement with Mr. Harden agreeing to a share What is more, inasmuch as Mr. and Mrs.
of a lesser amount. Harden are admittedly citizens of the
United States, their status and the
Appellee counsel for Mrs. Harden alleged dissolution thereof are governed —
that the purpose of the said instruments, pursuant to Article 9 of the Civil Code of
executed by Mr. and Mrs. Harden, was to Spain (which was in force in the
defeat the claim of the former for attorney’s Philippines at the time of the execution of
fees, for which reason, he prayed that the court the contract in question) and Article 15 of
grant him the necessary fees. the Civil Code of the Philippines — by the
laws of the United States, which sanction
Appellants assail the contract for professional divorce. In short, the contract of services,
services as void, mainly, upon the grounds between Mrs. Harden and herein Appellee, is
that: not contrary to law, morals, good customs,
public order or public policy.
1. that Mrs. Harden cannot bind the
It is a basic principle that status, once
conjugal partnership without her
established by the personal law of the
husband’s consent; c
party, is given universal recognition.
2. that Article 1491 of the Civil Code of Therefore, aliens can sue and be sued in
the Philippines in effect prohibits our courts subject to Philippine
contingent fees; c procedural law even on matters relating
3. that the contract in question has for its to their status and capacity. However, the
purpose to secure a decree of divorce, law to be applied by Philippine courts in
allegedly in violation of Articles 1305, determining their capacity and status is
1352 and 1409 of the Civil Code of the their personal law.
Philippines;
The last objection is based upon principles of
4. that the terms of said contract are
equity, but, pursuant thereto, one who seeks
harsh, inequitable and oppressive. equity must come with clean hands (Bastida, et
al., vs. Dy Buncio & Co., 93 Phil., 195; 30 C.J. S.
Held: 475), and Appellants have not done so, for the
circumstances surrounding the case show, to
The first objection has no foundation in our satisfaction, that their aforementioned
fact, for the contract in dispute does not agreements, ostensibly for the settlement of
seek to bind the conjugal partnership. By the differences between husband and wife,
virtue of said contract, Mrs. Harden merely were made for the purpose of circumventing or
bound herself — or assumed the personal defeating the rights of herein Appellee, under
obligation — to pay, by way of contingent fees, his above-quoted contract of services with Mrs.
20% of her share in said partnership. The Harden.
contract neither gives, nor purports to give, to
the Appellee any right whatsoever, personal or
real, in and to her aforesaid share. The amount BARNUEVO V. FUSTER
thereof is simply a basis for the computation of 29 PHIL 606
said fees.
FACTS:
For the same reason, the second objection is,
likewise, untenable. Moreover, it has already Gabriel and Constanza were married in Spain.
been held that contingent fees are not Thereafter, Gabriel went to the Philippines,
prohibited in the Philippines and are settled, and acquired real and personal
impliedly sanctioned by our Cannons (No. properties. Constanza later followed. A few
13) of Professional Ethics. (see, also, Ulanday years however, both parties executed a
vs. Manila Railroad Co., 45 Phil., 540, 554.) contract for their separation wherein the wife
Such is, likewise, the rule in the United States returned to Spain and has agreed to be
(Legal Ethics by Henry S. Drinker, p. 176). supported by the husband to be paid in Madrid,
Spain. Eventually, the wife instituted a petition
for divorce here in the Philippines against the
“in the United States, the great weight of husband. The husband opposed the petition on
authority recognizes the validity of contracts the grounds that:
for contingent fees, provided such contracts
are not in contravention of public policy, and it
is only when the attorney has taken an a. neither the trial court nor any other
unfair or unreasonable advantage of his court in the Philippine Islands has
client that such a claim is condemned.” jurisdiction over the subject matter of

Private International Law Page 27


the complaint, because, as to the
allowance for support, since neither the That by the express provision of article 80 of
plaintiff nor the defendant are the Civil Code of Spain, "jurisdiction in
residents of Manila, or of any other actions for divorce and nullification of
place in the Philippine Islands, the canonical marriages lies with
agreement upon the subject was ecclesiastical courts," while that of civil
neither celebrated, nor was it to tribunals is limited to civil marriages; that
be fulfilled, in the Philippine this being so, the action for divorce brought by
Islands; the plaintiff in the cause does not fall within
b. and as to the divorce, because the the jurisdiction of the civil courts,
action therefore ought to be tried by according to his own law of persons, because
the ecclesiastical courts. these courts ought to apply the Spanish law in
accordance with the said article 9 of the Civil
In deciding the case, the Court of First Instance Cod of Spain, and this Spanish law grants the
of the city of Manila held itself to have jurisdiction over the present cause to the
jurisdiction, decreed the suspension of life in ecclesiastical courts, in the place of which no
common between the plaintiff and defendant; tribunal of these Islands con subrogate itself.

Issue: However, husband was unable to prove by any


law or legal doctrine whatever that the
Do Philippine Courts have jurisdiction over the personal statute of a foreigner carries with it,
petition for divorce? to whether he transfers his domicile, the
What law should apply in construing the term authority established by the law of his nation to
pesetas? decree his divorce, which was what he had to
demonstrate.
Held:
As the Court upheld:
The lower court did not commit this error
attributed to him. The defendant had not "The jurisdiction of courts and other questions
proved that he had elsewhere a legal relating to procedure are considered to be of a
domicile other than that which he public nature and consequently are generally
manifestly had in the Philippines during submitted to the territorial principle. . . . All
the seventeen years preceding the date persons that have to demand justice in a case
of the complaint. On the contrary, it plainly in which foreigners intervene, since they can
appears, without proof to the contrary, that gain nothing by a simple declaration, should
during this not inconsiderable period, endeavor to apply to the tribunales of the state
extending from the year 1892 until a month which have coercive means (property situated
prior to the arrival of his wife in the Philippines in the territory) to enforce any decision they
in March, 1909, he had constantly resided in may render. Otherwise, one would expose
the said Islands, had kept open house, and had himself in the suit to making useless
acquired in the city of Manila quite a little real expenditures which, although he won his case,
property which is now the object of the division would not contribute to secure his rights
of the conjugal society. because of the court's lack of means to enforce
them." (Torres Campos, "Elementos de
It has been established that defendant is Derecho International Privado," p. 108.)
domiciled in the Philippines.
Defendant, although a Spanish subject, was a The provisions of article 80 of the Civil
resident of these Islands. Article 26 of the Civil Law of Spain is only binding within the
Code that he cites itself provides that dominions of Spain. It does not
"Spaniards who change their domicile to a accompany the persons of the Spanish
foreign country, where they may be subject wherever he may go. He could not
considered as natives without other successfully invoke it if he resided in Japan, in
conditions than that of residents therein, China, in Hongkong or in any other territory not
shall be required, in order to preserve the subject to the dominion of Spain. Foreign
Spanish nationality, to state that such is Catholics domiciled in Spain, subject to the
their wish before the Spanish diplomatic ecclesiastical courts in actions for divorce
or consular agent, who must record them according to the said article 80 of the Civil
in the registry of Spanish residents, as Code, could not allege lack of jurisdiction
well as their spouses, should they be by invoking, as the law of their personal
married, and any children they may statute, a law of their nation which gives
have." From this provision, which is the jurisdiction in such a case to territorial
exclusive and irrefutable law governing the courts, or to a certain court within or
defendant, we are to conclude that the without the territory of their nation.
domicile of the defendant and the plaintiff is
fully proven, irrespective of the Treaty of Paris.
(Benedicto vs. De la Rama, 3 Phil. Rep., 34, and
Section 377 of the Code of Civil Procedure Ibañez vs. Ortiz, 5 Phil. Rep., 325).
leaves to the election of the plaintiff the
bringing of a personal action like the one
at bar either in the place where the In the present action for divorce the Court of
defendant may reside or be found, or in First Instance of the city of Manila did not lack
that where the plaintiff resides. jurisdiction over the persons of the litigants,
for, although Spanish Catholic subjects, they
As held by the Husband: were residents of this city and had their
domicile herein.

Private International Law Page 28


The Courts of First Instance of the Philippine and petitioner herself even recognizes them as
Islands have the power and jurisdiction to try heirs of Arturo Padlan;
actions for divorce. That of the city of Manila
did not lack jurisdiction by reason of the Issue:
subject matter of the litigation.
Whether or not petitioner could inherit as the
With respect to their property regime, the surviving spouse of Arturo?
Foral Law presented by the husband in an
affidavit, with which conjugal partnership
is known to be inexistent, cannot apply Held:
since this affidavit was never presented
in proof, was never received by the trial Case remanded subject to determination of
judge, and cannot seriously be considered petitioner’s citizenship. However, Private
as an effort to establish the law of a respondent's claim to heirship was already
foreign jurisdiction. Sections 300, 301 and resolved by the trial court. She and Arturo
302 of the Code of Civil Procedure, now in force were married on 22 April 1947 while the
in these islands, indicate the method by which prior marriage of petitioner and Arturo
the law of a foreign country may be proved. was subsisting thereby resulting in a
The Court maintains that the affidavit of a bigamous marriage considered void from
person not versed in the law, which was the beginning under Arts. 80 and 83 of the
never submitted as proof, never received Civil Code. Consequently, she is not a surviving
by the trial court, and which has never spouse that can inherit from him as this status
been subjected to any cross-examination, presupposes a legitimate relationship.
is not a means of proving a foreign law on
which the defendant relies. Thus, since no Case remanded to the court a quo for further
proof has been submitted to this effect, all the proceedings since the trial court was not able
property of the marriage, says article 1407 of to completely ascertain petitioner’s citizenship.
the Civil Code, shall be considered as conjugal The trial court must have overlooked the
property until it is proven that it belongs materiality of this aspect. Once proved that
exclusively to the husband or to the wife. she was no longer a Filipino citizen at the
time of their divorce, Van Dorn would
ON PLAINTIFF’S APPEAL become applicable and petitioner could
very well lose her right to inherit from
The court did not commit it in applying the rule Arturo.
contained in article 1287 of the Civil Code.
"The usages or customs of the country
shall be taken into consideration in Nota bene:
interpreting ambiguity in contracts. . . ." If
in the contract the word " pesetas," not being Van dorn: ang bana ga laot sa wife since
specific, was ambiguous, then it was in under Philippine law, they’re still
harmony with this precept to interpret it as married.
being the peseta then in use or current when
and where the agreement was made, LLORENTE V. COURT OF APPEALS
Mexican being then the usual and current GR No. 124371, November 23, 2000
money in the Philippines.

QUITA V. PADLAN FACTS:


GR NO. 124371, DECEMBER 22, 1998

Lorenzo Llorente and petitioner Paula


FE D. QUITA and Arturo T. Padlan, both Llorente were married in 1937 in the
Filipinos, were married in the Philippines on 18 Philippines. Lorenzo was an enlisted
May 1941. They were not however blessed with serviceman of the US Navy. Soon after, he
children. Somewhere along the way their left for the US where through
relationship soured. Eventually Fe sued Arturo naturalization, he became a US Citizen.
for divorce in San Francisco, California, U.S.A. Upon his visit to his wife, he discovered that
and obtained a final judgment of divorce. She she was living with his brother and a child was
married thrice thereafter. born. The child was registered as
illegitimate but the name of the father
was left blank. Llorente filed a divorce in
On 1972 Arturo died. He left no will. California in which Paula was represented by
Respondent Blandina Padlan claiming to be the counsel, John Riley, and actively participated in
surviving spouse of Arturo Padlan, and Claro, the proceedings, which later on became final.
Alexis, Ricardo, Emmanuel, Zenaida and He married Alicia and they lived together for 25
Yolanda, all surnamed Padlan, named in the years bringing 3 children. He made his last will
children of Arturo Padlan opposed the petition. and testament stating that all his properties
Ruperto T. Padlan, claiming to be the sole will be given to his second marriage. He filed a
surviving brother of the deceased Arturo, also petition of probate that made or appointed
intervened. Alicia his special administrator of his estate.
Before the proceeding could be terminated,
The court held that no dispute exists as to the Lorenzo died. Paula filed a letter of
right of the six (6) Padlan children to inherit administration over Llorente’s estate. The trial
from the decedent because there are proofs granted the letter and denied the motion for
that they have been duly acknowledged by him reconsideration. An appeal was made to the

Private International Law Page 29


Court of Appeals, which affirmed and modified Nota bene:
the judgment of the Trial Court that she be
declared co-owner of whatever properties, she The following are the requisites of res
and the deceased, may have acquired in their judicata:
25 years of cohabitation. 1. the former judgment must be final;
2. the court that rendered it had
ISSUE: jurisdiction over the subject matter and
the parties;
3. it is a judgment on the merits; and
4. there is — between the first and the
Whether or not national law shall apply? second actions — an identity of parties,
subject matter and cause of action.
RULING:
It has been held that in order that a
“Art. 15. Laws relating to family rights and judgment in one action can be conclusive
duties, or to the status, condition and legal as to a particular matter in another action
capacity of persons are binding upon between the same parties or their privies,
citizens of the Philippines, even though it is essential that:
living abroad.
1. the issue be identical;
2. If a particular point or question is in
“Art. 16. Real property as well as personal issue in the second action,
property is subject to the law of the country 3. and the judgment will depend on the
where it is situated. determination of that particular point
or question,
First, there is no such thing as one American 4. a former judgment between the same
law. The "national law" indicated in Article 16 parties or their privies will be final and
of the Civil Code cannot possibly apply to conclusive in the second if that same
general American law. There is no such law point or question was in issue and
governing the validity of testamentary adjudicated in the first suit (Nabus v.
provisions in the United States. Each State of Court of Appeals, 193 SCRA 732
the union has its own law applicable to its [1991]).
citizens and in force only within the State. It 5. Identity of cause of action is not
can therefore refer to no other than the law of required but merely identity of issue.
the State of which the decedent was a
resident. Second, there is no showing that Art. 15. Laws relating to family rights
the application of the renvoi doctrine is and duties, or to the status,
called for or required by New York State condition and legal capacity of
law. persons are binding upon
citizens of the Philippines, even
“However, intestate and testamentary though living abroad.
succession, both with respect to the order of (FRDSCL)
succession and to the amount of successional
rights and to the intrinsic validity of Personal status – legal position of an
testamentary provisions, shall be regulated individual in a society
by the national law of the person whose Capacity – power to acquire and
succession is under consideration, exercise rights
whatever may be the nature of the property Incidental to personal status
and regardless of the country wherein said
property may be found.” (emphasis ours) Foreign laws regulating the person’s status and
capacity are to be disregarded where they are
political or penal in character;
Likewise, Lorenzo Llorente was already an Legislative jurisdiction – authority of the state
American citizen when he divorced Paula. of his nationality or domicile or where he may
Such was also the situation when he married be physically present to promulgate laws
Alicia and executed his will. As stated in affecting his status;
Article 15 of the civil code, aliens may Judicial jurisdiction – the authority of the court
obtain divorces abroad, provided that to hear and determine the cause of action
they are valid in their National Law. Thus
the divorce obtained by Llorente is valid Beginning of personality – conception provided
because the law that governs him is not subsequent birth
Philippine Law but his National Law since the End – death
divorce was contracted after he became an
American citizen. Furthermore, his National Other questions of status
Law allowed divorce.
1. Absence – a special legal status, if one
is not in his domicile and his
The case was remanded to the court of origin whereabouts is unknown;
for determination of the intrinsic validity of When can be declared: under civil code
Lorenzo Llorente’s will and determination of –
the parties’ successional rights allowing proof a. Lapse of 2 years without news or
of foreign law. since the receipt of last news

Private International Law Page 30


b. Lapse of 5 years if absentee has Art. XV, Sec. 2, 1987 Constitution
left someone in charge of the
administration of his properties Marriage, as an inviolable social institution, is
the foundation of the family and shall be
Who may declare: protected by the State.
a. Spouse
b. Heirs
NCC Art. 15. Laws relating to family rights
c. Relatives who will benefit in intestacy
d. Those who have rights over the and duties, or to the status,
properties condition and legal capacity of
persons are binding upon
2. Conflicts problems respecting an citizens of the Philippines, even
individual’s name and extent of though living abroad. (9a)
protection against abuse of his name
Art. 17. The forms and solemnities of
Right to use a title of nobility is contracts, wills, and other
determined in accordance with his public instruments shall be
national law
governed by the laws of the
3. Minority country in which they are
executed.
CAPACITY
When the acts referred to are
1. Juridical capacity – the fitness to be the executed before the diplomatic
subject of legal relations; considered or consular officials of the
inherent in every natural person and is Republic of the Philippines in a
lost only through death; foreign country, the solemnities
2. Capacity to act – the power to do acts established by Philippine laws
with legal effects; it is acquired and shall be observed in their
may be lost; execution.
Nota bene:
Prohibitive laws concerning
Under Philippine law, the capacity to dispose of persons, their acts or property,
real property is governed by the lex situs while and those which have, for their
the capacity to succeed is governed by the object, public order, public
national law of the deceased. policy and good customs shall
not be rendered ineffective by
Question: laws or judgments
promulgated, or by
A 16-year old Cambodian inherited real determinations or conventions
properties from the Philippines; he died and left agreed upon in a foreign
a will which disposes of his properties in the country. (11a)
Philippines; supposing, Cambodian law
provides that age of majority is 16, is the will Art. 66. When either or both of the
now if probated in the Philippines valid? contracting parties are citizens
or subjects of a foreign
If an 18-year old foreigner, where in his country, it shall be necessary,
country, the age of majority is 21 goes to the before a marriage license can
Philippines and gets married, is such marriage be obtained, to provide
valid? themselves with a certificate
of legal capacity to contract
Family rights, duties, status, condition marriage, to be issued by
and legal capacity of persons accompany their respective diplomatic or
a person even when he moves to a consular officials. (13a)
foreign country;
Art. 71. All marriages performed
outside the Philippines in
MARRIAGE accordance with the laws in
force in the country where they
Art. II Section 12, 1987 Constitution were performed, and valid
there as such, shall also be
The State recognizes the sanctity of family valid in this country, except
life and shall protect and strengthen the bigamous, polygamous, or
family as a basic autonomous social incestuous marriages as
institution. It shall equally protect the life of determined by Philippine law.
(19a)
the mother and the life of the unborn from
conception. The natural and primary right and
duty of parents in the rearing of the youth for Art. 75. Marriages between Filipino
citizens abroad may be
civic efficiency and the development of moral
solemnized by consuls and
character shall receive the support of the vice-consuls of the Republic of
Government. the Philippines. The duties of

Private International Law Page 31


the local civil registrar and of a Article 29 of this Code, or
judge or justice of the peace or where both of the parties
mayor with regard to the request the solemnizing officer
celebration of marriage shall be in writing in which case the
performed by such consuls and marriage may be solemnized at
vice-consuls. (n) a house or place designated by
them in a sworn statement to
FC Article 1. Marriage is a special contract that effect. (57a)
of permanent union between a
man and a woman entered into FC Art. 10. Marriages between Filipino
in accordance with law for the citizens abroad may be
establishment of conjugal and solemnized by a consul-
family life. It is the foundation general, consul or vice-consul
of the family and an inviolable of the Republic of the
social institution whose nature, Philippines. The issuance of the
consequences, and incidents marriage license and the duties
are governed by law and not of the local civil registrar and of
subject to stipulation, except the solemnizing officer with
that marriage settlements may regard to the celebration of
fix the property relations marriage shall be performed by
during the marriage within the said consular official. (75a)
limits provided by this Code.
(52a)
Art. 26. All marriages solemnized
Art. 7. Marriage may be solemnized outside the Philippines, in
by: accordance with the laws in
force in the country where they
were solemnized, and valid
1. Any incumbent member of there as such, shall also be
the judiciary within the valid in this country, except
court's jurisdiction; those prohibited under Articles
2. Any priest, rabbi, imam, or 35 (1), (4), (5) and (6), 3637
minister of any church or and 38. (17a)
religious sect duly
authorized by his church or
religious sect and Where a marriage between
registered with the civil a Filipino citizen and a
registrar general, acting foreigner is validly
within the limits of the celebrated and a divorce is
written authority granted thereafter validly obtained
by his church or religious abroad by the alien spouse
sect and provided that at capacitating him or her to
least one of the contracting remarry, the Filipino spouse
parties belongs to the shall have capacity to
solemnizing officer's church remarry under Philippine
or religious sect; law. (As amended by
3. Any ship captain or Executive Order 227)
airplane chief only in the
case mentioned in Article FC Art. 35. The following marriages shall
31; be void from the beginning:
4. Any military commander of 1. Those contracted by any
a unit to which a chaplain party below eighteen years
is assigned, in the absence of age even with the
of the latter, during a consent of parents or
military operation, likewise guardians;
only in the cases 2. Those solemnized by any
mentioned in Article 32; person not legally
5. Any consul-general, consul authorized to perform
or vice-consul in the case marriages unless such
provided in Article 10. marriages were contracted
(56a) with either or both parties
believing in good faith that
Article. 8. The marriage shall be the solemnizing officer had
solemnized publicly in the the legal authority to do so;
chambers of the judge or in 3. Those solemnized without
open court, in the church, license, except those
chapel or temple, or in the covered the preceding
office the consul-general, Chapter;
consul or vice-consul, as the 4. Those bigamous or
case may be, and not polygamous marriages not
elsewhere, except in cases of failing under Article 41;
marriages contracted on the 5. Those contracted through
point of death or in remote mistake of one contracting
places in accordance with

Private International Law Page 32


party as to the identity of apply to marriage
the other; and and divorce
6. Those subsequent wherein both
marriages that are void parties are
under Article 53. Muslims, or
wherein only the
Art. 36. A marriage contracted by any male party is a
party who, at the time of the Muslim and the
celebration, was marriage is
psychologically incapacitated solemnized in
to comply with the essential accordance with
marital obligations of marriage, Muslim law or this
shall likewise be void even if Code in any part of
such incapacity becomes the Philippines.
manifest only after its (2) In case of marriage
solemnization. (As amended by between a Muslim
Executive Order 227) and a non-Muslim,
solemnized not in
accordance with
Art. 37. Marriages between the
Muslim law or this
following are incestuous and
Code, the Civil
void from the beginning,
Code of the
whether relationship between
Philippines shall
the parties be legitimate or
apply. .chan robles
illegitimate:
virtual law library
(3) Subject to the
1. Between ascendants and provisions of the
descendants of any degree; preceding
and paragraphs, the
2. Between brothers and essential requisites
sisters, whether of the full and legal
or half blood. (81a) impediments to
marriage, divorce,
Art. 38. The following paternity and
marriages shall be void from the beginning for filiation,
reasons of public policy: guardianship and
custody of minors,
1. Between collateral support and
blood relatives whether maintenance,
legitimate or claims for
illegitimate, up to the customary dower
fourth civil degree; (mahr), betrothal,
2. Between step-parents breach of contract
and step-children; to marry,
3. Between parents-in-law solemnization and
and children-in-law; registration of
4. Between the adopting marriage and
parent and the adopted divorce, rights and
child; obligations
5. Between the surviving between husband
spouse of the adopting and wife parental
parent and the adopted authority, and the
child; properly relations
6. Between the surviving between husband
spouse of the adopted and wife shall be
child and the adopter; governed by this
7. Between an adopted Code and other
child and a legitimate applicable Muslim
child of the adopter; laws.
8. Between adopted
children of the same Art. 14. Nature. — Marriage is not only
adopter; and a civil contract but a social
9. Between parties where institution. Its nature,
one, with the intention consequences and incidents
to marry the other, are governed by this Code and
killed that other the Shari'a and not subject to
person's spouse, or his stipulation, except that the
or her own spouse. (82) marriage settlements may to a
certain extent fix the property
Muslim Code PD 1083, Art. relations of the spouses.
13. Application. —
Art. 180. Law applicable. — The
(1) The provisions of provisions of the Revised
this Title shall

Private International Law Page 33


Penal Code relative to the 3. Those solemnized without
crime of bigamy shall not license, except those
apply to a person married in covered the preceding
accordance with the provisions Chapter;
of this Code or, before its 4. Those bigamous or
effectivity, under Muslim law. polygamous marriages not
failing under Article 41;
5. Those contracted through
Essential and Formal Requisites mistake of one contracting
party as to the identity of
the other; and
FC Art. 2. No marriage shall be valid, 6. Those subsequent
unless these essential requisites are present: marriages that are void
under Article 53.
1. Legal capacity of the
contracting parties who Art. 4. The absence of any of the essential or
must be a male and a formal requisites shall render
female; and the marriage void ab initio,
2. Consent freely given in except as stated in Article 35
the presence of the (2).
solemnizing officer.
(53a)
A defect in any of the essential
requisites shall not affect the
Art. 3. The formal requisites of validity of the marriage but the
marriage are: party or parties responsible for
the irregularity shall be civilly,
1. Authority of the criminally and administratively
solemnizing officer; liable. (n)
2. A valid marriage
license except in the
cases provided for in
Chapter 2 of this Title;
and Art. 35 (2) supra
3. A marriage ceremony
which takes place with Muslim Code, Art. 15. Essential
the appearance of the requisites. —
contracting parties
before the solemnizing
officer and their No marriage contract shall be perfected
personal declaration unless the following essential requisites
that they take each are compiled with:
other as husband and
wife in the presence of a. Legal capacity of the
not less than two contracting parties;
witnesses of legal age. b. Mutual consent of the parties
(53a, 55a)
freely given;
c. Offer (ijab) and acceptance
Art. 5. Any male or female of the age of (qabul) duly witnessed by at
eighteen years or upwards not least two competent persons
under any of the impediments after the proper guardian in
mentioned in Articles 37 and marriage (wali) has given his
38, may contract marriage. consent; and
(54a) d. Stipulation of customary dower
(mahr) duly witnessed by two
Art. 35. The following competent persons.
marriages shall be void from the beginning:

1. Those contracted by any


party below eighteen years
of age even with the GOITIA VS. CAMPOS RUEDA
consent of parents or
guardians; Facts:
2. Those solemnized by
any person not legally The parties were legally married in the city of
authorized to perform Manila on January 7, 1915, and immediately
marriages unless such thereafter established their residence at 115
marriages were Calle San Marcelino, where they lived together
contracted with either for about a month, when the plaintiff returned
or both parties believing to the home of her parents.
in good faith that the
solemnizing officer had That the defendant, one month after he had
the legal authority to do contracted marriage with the plaintiff,
so; demanded of her that she perform unchaste

Private International Law Page 34


and lascivious acts on his genital organs; that resulting from a decree for separate support is
the plaintiff spurned the obscene demands of not an impeachment of that public policy by
the defendant and refused to perform any act which marriage is regarded as so sacred and
other than legal and valid cohabitation; that inviolable in its nature; it is merely a stronger
the defendant, since that date had continually policy overruling a weaker one; and except in
on other successive dates, made similar lewd so far only as such separation is tolerated as a
and indecorous demands on his wife, the means of preserving the public peace and
plaintiff, who always spurned them, which just morals may be considered, it does not in any
refusals of the plaintiff exasperated the respect whatever impair the marriage contract
defendant and induce him to maltreat her by or for any purpose place the wife in the
word and deed and inflict injuries upon her lips, situation of a feme sole.
her face and different parts of her body; and
that, as the plaintiff was unable by any means Where the wife, who is forced to leave the
to induce the defendant to desist from his conjugal abode by her husband without fault
repugnant desires and cease from maltreating on her part, may maintain an action against
her, she was obliged to leave the conjugal the husband for separate maintenance when
abode and take refuge in the home of her she has no other remedy, notwithstanding
parents. the provisions of article 149 of the Civil
Code giving the person who is obliged to
Issue: whether or not that the wife may furnish support the option to satisfy it
claim for support against her husband either by paying a fixed pension or by
outside of their conjugal abode? receiving and maintaining in his own
home the one having the right to the
Ruling: marriage partakes of the nature of an same.
ordinary contract. But it is something more
than a mere contract. It is a new relation, the
rights, duties, and obligations of which rest
not upon the agreement of the parties WONG WOO YIU VS. VIVO
but upon the general law which defines
and prescribes those rights, duties, and Facts:
obligations. Marriage is an institution, in the
maintenance of which in its purity the public is The Board of Special Inquiry No. 3 rendered a
deeply interested. It is a relation for life and decision finding petitioner to be legally
the parties cannot terminate it at any shorter married to Perfecto Blas and admitting
period by virtue of any contract they may her into the country as a non-quota
make .The reciprocal rights arising from this immigrant. This decision was affirmed by the
relation, so long as it continues, are such as Board of Commissioners of which petitioner
the law determines from time to time, and was duly informed in a letter sent on the same
none other. When the legal existence of the date by the Secretary of the Board. However,
parties is merged into one by marriage, the the same Board of Commissioners, but
new relation is regulated and controlled by the composed entirely of a new set of
state or government upon principles of public members, rendered a new decision
policy for the benefit of society as well as the reversing that of the Board of Special
parties. And when the object of a marriage is Inquiry No. 3 and ordering petitioner to
defeated by rendering its continuance be excluded from the country. Petitioner
intolerable to one of the parties and productive filed a motion for new trial requesting an
of no possible good to the community, relief in opportunity to clarify certain points taken in
some way should be obtainable. With these the decision, but the same was denied for lack
principles to guide us, we will inquire into the of merit. Whereupon, petitioner initiated the
status of the law touching and governing the instant petition for mandamus with preliminary
question under consideration. injunction before the Court of First Instance of
Manila which incidentally was considered by it
The mere act of marriage creates an as a petition for certiorari.
obligation on the part of the husband to
support his wife. This obligation is Petitioner declared that she came to the
founded not so much on the express or Philippines in 1961 for the first time to
implied terms of the contract of marriage join her husband Perfecto Blas to whom
as on the natural and legal duty of the she was married in Chingkang, China ;that
husband; an obligation, the enforcement of they had several children all of whom are not
which is of such vital concern to the state itself in the Philippines; that their marriage was
that the laws will not permit him to terminate it celebrated by one Chua Tio, a village
by his own wrongful acts in driving his wife to leader; that on June 28, 1961 the Board of
seek protection in the parental home. A Special Inquiry No. 3 rendered a decision
judgment for separate maintenance is not due finding, among others, that petitioner is legally
and payable either as damages or as a penalty; married to Perfecto Blas, a Filipino Citizen, and
nor is it a debt in the strict legal sense of the admitted her into the country as a non-quota
term, but rather a judgment calling for the immigrant; that this decision was affirmed by
performance of a duty made specific by the the Board of Commissioners of which petitioner
mandate of the sovereign. This is done from was duly notified by the Secretary of said
necessity and with a view to preserve the Board in a letter dated July 12, 1961; that in a
public peace and the purity of the wife; as motu proprio decision rendered by the Board of
where the husband makes so base demands Commissioners composed of a new set of
upon his wife and indulges in the habit of members the latter found that petitioner's
assaulting her. The pro tanto separation claim that she is the lawful wife of Perfecto

Private International Law Page 35


Blas was without basis in evidence as it was country it should be presumed that it is the
"bereft of substantial proof of husband-wife same as our own.
relationship"; that said Board further held that,
it appearing that in the entry proceedings of Since our law only recognizes a marriage
Perfecto Blas had on January 23, 1947 he celebrated before any of the officers
declared that he first visited China in 1935 and mentioned therein, and a village leader is not
married petitioner in 1936, it could not possibly one of them, it is clear that petitioner's
sustain her claim that she married Perfecto marriage, even if true, cannot be recognized in
Blas in 1929; that in an affidavit dated August this jurisdiction.
9, 1962 Perfecto Blas claimed that he went to
China in 1929, 1935 and 1941, although in his
re-entry declaration he admitted that he first ADONG VS. CHAONG SENG GEE
went to China in 1935, then in 1937, then in
1939, and lastly in 1941; and that Perfecto Blas Facts:
in the same affidavit likewise claimed that he
first went to China when he was merely four Cheong Boo, a native of China died in
years old so that computed from his date of Zamboanga, Philippine Islands on August
birth in 1908 it must have been in 1912. 5, 1919 and left property worth nearly
P100,000 which is now being claimed by
In view of the discrepancies found in the two parties - (1) Cheong Seng Gee who
statements made by petitioner and her alleged alleged that he was a legitimate child by
husband Perfecto Blas in the several marriage contracted by Cheong Boo with
investigations conducted by the immigration Tan Bit in China in 1985, and (2) Mora
authorities concerning their alleged marriage Adong who alleged that she had been
before a village leader in China in 1929, lawfully married to Cheong Boo in 1896 in
coupled with the fact that the only basis in Basilan, Philippine Islands and had two
support of petitioner's claim that she is the daughters with the deceased namely Payang
lawful wife of Perfecto Blas is "a mass of oral and Rosalia. The conflicting claims to Cheong
and documentary evidence bereft of Boo’s estate were ventilated in the lower
substantial proof of husband-wife relationship," court that ruled that Cheong Seng Gee
the Board of Commissioners motu proprio failed to sufficiently establish the Chinese
reviewed the record concerning the admission marriage through a mere letter testifying
of petitioner into the country resulting in its that Cheong Boo and Tan Bit married
finding that she was improperly admitted. each other but that because Cheong Seng
Gee had been admitted to the Philippine
Issue: whether or not that the marriage is Islands as the son of the deceased, he
valid? should share in the estate as a natural
child. With reference to the allegations of Mora
Adong and her daughters, the trial court
Ruling: there is no documentary evidence to reached the conclusion that the marriage
support the alleged marriage of petitioner to between Adong and Cheong Boo had been
Perfecto Blas but the record is punctured with adequately proved but that under the laws of
so many inconsistencies which cannot but lead the Philippine Islands it could not be held to be
one to doubt their veracity concerning the a lawful marriage and thus the daughter
pretended marriage in China in 1929. This Payang and Rosalia would inherit as natural
claim cannot also be entertained under our law children. The lower court believes that
on family relations. Thus, Article 15 of our Mohammedan marriages are not valid under
new Civil Code provides that laws relating the Philippine Island’s laws this as an Imam as
to family rights or to the status of a solemnizing officer and under Quaranic laws.
persons are binding upon citizens of the
Philippines, even though living abroad,
and it is well-known that in 1929 in order ISSUES: Whether or not the Chinese marriage
that a marriage celebrated in the between Cheong Boo and Tan Dit is valid ?
Philippines may be valid it must be
solemnized either by a judge of any court Whether or not the Mohammedan marriage
inferior to the Supreme Court, a justice of between Cheong Boo and Mora Adong is valid?
the peace, or a priest or minister of the
gospel of any denomination duly RULING: The Supreme Court found the (1)
registered in the Philippine Library and Chinese marriage not proved and
Museum (Public Act 3412, Section 2). Even if Chinaman Cheong Seng Gee has only the
we assume, therefore, that the marriage of rights of a natural child while (2) it found
petitioner to Perfecto Blas before a village the Mohammedan marriage to be proved
leader is valid in China, the same is not one of and to be valid, thus giving to the widow
those authorized in our country. Mora Adong and the legitimate children Payang
and Rosalia the rights accruing to them under
A marriage contracted outside of the the law.
Philippines which is valid under the law of
the country in which it was celebrated is (FOR STATCON) The Supreme Court held
also valid in the Philippines. But no that marriage in this jurisdiction is not only a
validity can be given to this contention civil contract but it is a new relation, an
because no proof was presented relative instruction in the maintenance of which the
to the law of marriage in China. Such being public is deeply interested. The presumption as
the case, we should apply the general rule that to marriage is that every intendment of the law
in the absence of proof of the law of a foreign leans toward legalizing matrimony. Persons

Private International Law Page 36


dwelling together in apparent matrimony are been declared an absentee nor generally
presumed, in the absence of counter- considered as dead and believed to be so by
presumption or evidence special to the case, to respondent at the time he married Maria Ching.
be in fact married. The reason is that such is
the common order of society, and if the parties Respondent, in his answer, among other
were not what they thus hold themselves out things, asserts that on June 21, 1946, he and
as being, they would be living in the constant Maria Ching alias Avelina Ching were legally
violation of decency of the law. As to married before the Justice of the Peace of
retroactive force, marriage laws is in the nature Plaridel, Bulacan, and alleges that the essential
of a curative provision intended to safeguard requisites for such marriage were complied
society by legalizing prior marriages. Public with.
policy should aid acts intended to
validate marriages and should retard acts
intended to invalidate marriages. This as Issue: whether or not petitioner still retains his
for public policy, the courts can properly incline right to the custody of his minor daughter
the scales of their decision in favor of that Maria Ching alias Avelina Ching?
solution which will most effectively promote
the public policy. That is the true construction Ruling: the Philippine marriage between
which will best carry legislative intention into said respondent and Maria Ching before
effect. the Justice of the Peace of Plaridel,
(FOR PERSONS) Sec. IV of the Marriage law Bulacan, is undisputed. It is also beyond
provides that “all marriages contracted outside question that marriage was contracted by a
the islands, which would be valid by the laws of man much over 16 years old with a girl 15
the country in which the same were years old (Act No. 3613, section 2), neither of
contracted, are valid in these islands. To whom was included in any of the exceptions
establish a valid foreign marriage mentioned in section 28 of the same Act; nor in
pursuant to this comity provision, it is those stated in section 29 thereof for the
first necessary to prove before the courts reason that the alleged prior Chinese marriage
ofthe Islands the existence of the foreign has not been established.
law as a question of fact, and it is then
necessary to prove the alleged foreign If the supposed prior Chinese marriage had
marriage by convincing evidence. A been sufficiently proven, then in order that
Philippine marriage followed by 23 years of the subsequent Philippine marriage could
uninterrupted marital life, should not be be valid, it would have been necessary
impugned and discredited, after the death of either (a) that the Chinese marriage
the husband through an alleged prior Chinese should have been previously annulled or
marriage, “save upon proof so clear, strong dissolved: or (b) that the first wife of
and unequivocal as to produce a moral respondent should have been absent for 7
conviction of the existence of such consecutive years at the time of the
impediment.” A marriage alleged to have been second marriage without the respondent
contracted in China and proven mainly by a so- having news of the absentee being alive;
called matrimonial letter held not to be valid in or (c) that the absentee should have been
the Philippines. generally considered as dead and
believed to be so by respondent at the
time of contracting the subsequent
marriage, in either of which last two
CHING HUAT VS. CO HEONG cases the subsequent marriage will be
valid until declared null and void by a
competent court, while in the first it will
Facts: be valid without this limitation. (Act No.
3613, section 29 [a] and [b]; section 30 [b].)
It is alleged in the petition, that the said But as already adverted to, the complete
minor is his legitimate daughter; that up to absence of proof of the supposed former
June 21, 1946, said minor had been living with Chinese marriage makes sections 29 and 30 of
and had under the custody of petitioner; that the Marriage Law inapplicable.
respondent, taking advantage of his
confidential and spiritual relation with Maria Ching having been validly married
Maria Ching as her godfather, persuaded on June 21, 1946, she became
and induced her by means of trick, promises emancipated on that same date (arts. 314
and cajolery, to leave the parental home and to [1] and 315, Civil Code). This emancipation
elope with him in the night of June 21, 1946, to brought about the loss by the father of the
Plaridel, Bulacan, where they were married on parental authority that he claims. On the other
the following day before the Justice of the hand, by article 48 of Chapter V of the Spanish
Peace of said municipality, said Maria Ching Marriage Law of 1870, whose articles 44 to 78
being at the time 15 years old; and that ever were, and are now partly, in force in the
since respondent has had the minor Maria Philippines (Benedicto vs. De la Rama, 3 Phil.,
Ching under his custody in Malolos, Bulacan, 34), the wife has the duty, among others, of
and has restrained her at her liberty. living in her husband's company and of
following him to wherever he transfer his
It is further alleged that respondent had domicile or residence. (Yañez de Barbuevo vs.
been previously married in China to Gue Fuster, 29 Phil., 606, 612.)
Min, said marriage being said to be subsisting
at the time respondent married Maria Ching.
Petitioner further avers that Gue Min has never

Private International Law Page 37


PEOPLE V. DUMPO witnesses. Jan Peng, a Chinaman of 52 years of
62 Phil 247 age, swore that he knew the forms of
ceremonies of marriage in China, Dee Tim also
Facts: presented a witness, Ty Cong Ting, a
Chinaman, 32 years of age and a lawyer, who
Moro Hassan and Mora Dupo have been testified concerning the laws and customs in
legally married according to the rites and China with reference to the forms of marriage
practice of the Mohammedan religion. Without ceremony. He testified that he knew and was
this marriage being dissolved, it is well acquainted with the customs and practices
alleged that Dumpo contracted another of Chinamen in China with reference to
marriage with Moro Sabdapal after which marriages and the manner and form in which
the two lived together as husband and wife. they were celebrated, and the form of proof
Dumpo was prosecuted for and convicted issued for the purpose of proving that a
of the crime of bigamy. marriage ceremony had been performed. Mr.
Ty Cong Ting was, at the time he testified as a
The accused interposed an appeal. It has been witness, the legal attorney of the Chinese
established by the defense, without the Consul General in the City of Manila.
prosecution having presented any objection
nor evidence to the contrary, that the alleged Issue:
second marriage of the accused is null and
void according to Mohammedan rites on the
ground that her father had not given his Who is the legal wife?
consent thereto.
Held:
Held:
It is perhaps true that Yap Siong did on various
Marriage among Moslems is a fact of occasions, depending upon his interest and
which no judicial notice may be taken and convenience at the particular time, state that
must be subject to proof in every Maria Lao was his querida and not his wife. It is
particular case. In the case at bar we have also perhaps true, for the same reason, that he
the uncontradicted testimony of Tahari, an stated that Dee Tim was not his wife but his
Imam or Mohammedan priest authorized to querida. Evidently he was attempting to
solemnize marriages between Mohammedans, keep the information, which he was quite
to the effect that the consent of the bride's able to do, until he had passed to that
father or in the absence thereof, that of the bourn from which none returns, and until
chief of the tribe to which she belongs is an a distribution of his large accumulated
indispensable requisite for the validity of such earnings among his heirs became
contracts. necessary.
It is an essential element of the crime of
bigamy that the alleged second marriage, Based on a preponderance of the
having all the essential requisites, would evidence the Court was convinced that
be valid were it not for the subsistence of the both Dee Tim and Maria Lao were legally
first marriage. However, accused’s subsequent married to Yap Siong in good faith,
marriage was void for lack of requisites believing that each was his sole and separate
necessary under Moslem law, she must be wife, living in absolute ignorance of the fact of
acquitted. his double marriage. They were each
married in good faith and in ignorance of
the existence of the other marriage. Yap
Siong up to the time of his death seems to
LAO AND LAO V. DEE TIM have been successful in keeping each of his
45 Phil 739 (1924) two wives ignorant of the fact that he was
married to the other.
Facts:
Under the Leyes de Partidas (Law 1, title
Yap Siong died intestate. During the 13, partida 4), where two women
distribution of his estate, Maria Lao and Jose innocently and in good faith are legally
Lao appeared claiming to be the legitimate united in holy matrimony to the same
spouse and son of the deceased. Maria claims man, their children born will be regarded
that they had been married in the Philippines as legitimate children and each family will
on June 24, 1903. On the other hand, Dee Tim be entitled to one-half of the estate of the
claims to be the legitimate widow of Yap Siong; husband upon distribution of his estate.
that she and Yap Siong were joined in holy That provision of the Leyes de Partidas is a
matrimony on the 14th day of September, very humane and wise law. It justly protects
1893, in accordance with the laws of China. those who innocently have entered into the
solemn relation of marriage and their
descendants. The good faith of all the
Maria Lao presented marriage certificates as parties will be presumed until the
proof. Dee Tim likewise presented a certificate contrary is positively proved. (Article 69,
of marriage and that it was positive proof of Civil Code; Las Leyes de Matrimonio, section
her marriage and that it complied with the 96; Gaines vs. Hennen, 65 U.S., 553.)c
custom and practice in China with reference to
marriage ceremonies. To support her YAO KEE V. GONZALES
contention she presented a number of 167 SCRA 736

Private International Law Page 38


custom on marriage not only because they
FACTS: are self serving evidence, but more
importantly, there is no showing that
they are competent to testify on the
subject matter. For failure to prove the
1. Sy Kiat, a Chinese national, died in
foreign law or custom, and consequently,
Calooocan City where he was then
the validity of the marriage in accordance
residing leaving behind real and
with said law or custom, the marriage
personal properties here in the
between Yao Kee and Sy Kiat cannot be
Philippines.
recognized in this jurisdiction.
2. Private respondents (Aida Sy-Gonzales
• However, as petitioners failed to establish
et al.,) filed a petition for the grant of
the marriage of Yao Kee with Sy Kiat
letters or administration alleging that
according to the laws of China, they cannot
they were the children of the deceased
be accorded the status of legitimate
with Asuncion Gillego.
children but only that of acknowledged
3. Petition was opposed by herein
natural children. Petitioners are natural
petitioners (Yao Kee et al.,) alleging
children, it appearing that at the time of
that they were the legitimate family.
their conception Yao Kee and Sy Kiat were
4. The probate court found that Sy Kiat
not disqualified by any impediment to
was legally married to Yao Kee and
marry one another. [See Art. 269, Civil
that their 3 offsprings were the
Code] And they are acknowledged children
legitimate children.
of the deceased because of Sy Kiat’s
5. The court likewise ruled that
recognition of Sze Sook Wah and its
respondents are the acknowledged
extension to Sze Lai Cho and Sy Chun Yen
illegitimate offspring of Sy Kiat with
who are her sisters of the full blood.
Asuncion Gillego.
6. On appeal, the lower court’s decision • Private respondents on the other hand are
was set aside declaring petitioners as also the deceased’s acknowledged natural
the acknowledge natural children of Sy children with Asuncion Gillego , a Filipina
Kiat and Asuncion Gillego. with whom he lived for 25 years without
7. Oppostiors were declared the the benefit of marriage. They have in their
acknowelged natural children of the favor their father’s acknowledgment,
deceased since the legality of the evidence by a compromise agreement
alleged marriage of Sy Kiat and Yao entered into by and between their parents
Kee in China had not been proven to be and approved by the CFI wherein Sy Kiat
valid to the laws of China. not only acknowledged them as his
children by Asuncion Gillego but likewise
ISSUE: made provisions for their support and
future inheritance.
• Was the fact of marriage of Sy Kiat and Yao REPUBLIC V. ORBECIDO
Kee in China proven as a custom? GR NO. 154380, October 5, 2005
HELD:

Facts:
• Custom is defined as “a rule of
conduct formed by repetition of acts,
uniformly observed (practiced) as a On May 24, 1981, Cipriano Orbecido III
social rule, legally binding and married Lady Myros M. Villanueva in the
obligatory.” The law requires that “a Philippines in Lam-an, Ozamis City. Their
custom must be proved as a fact, marriage was blessed with a son and a
according to the rules of evidence. daughter.
[Article 12, Civil Code] On this score the
Court had occasion to state that “a local In 1986, Cipriano’s wife left for the United
custom as a source of right cannot be States bringing along their son
considered by a court of justice unless such Kristoffer. A few years later, Cipriano
custom is properly established by discovered that his wife had been naturalized
competent evidence like any other fact. as an American citizen.
The same evidence, if not one of a higher
degree, should be required of a foreign Sometime in 2000, Cipriano learned from his
custom. son that his wife had obtained a divorce
• Construing this provision of law the Court decree and then married a certain Innocent
has held that to establish a valid foreign Stanley. She, Stanley and her child by him
marriage two things must be proven, currently live in San Gabriel, California.
namely 1) the existence of the foreign
law as a question of fact; and 2) the Cipriano thereafter filed with the trial
alleged foreign marriage by court a petition for authority to remarry
convincing evidence. invoking Paragraph 2 of Article 26 of the
• In the case at bar petitioners did not Family Code. No opposition was filed. Finding
present any competent evidence merit in the petition, the court granted the
relative to the law and custom of same. The Republic, herein petitioner,
China on marriage. The testimonies of through the Office of the Solicitor General
Yao and Gan Ching (brother) cannot be (OSG), sought reconsideration but it was
considered as proof of China’s law or denied.

Private International Law Page 39


The OSG contends that Paragraph 2 of Article and compelling reasons for the
26 of the Family Code is not applicable to exemption. However, such
the instant case because it only applies to exemption shall not apply if the
a valid mixed marriage; that is, a marriage same is not compatible with
celebrated between a Filipino citizen and an the solidarity of the family.
alien. Furthermore, the OSG argues there is no (110a)
law that governs respondent’s situation. The
OSG posits that this is a matter of legislation Art. 70. The spouses are jointly
and not of judicial determination. responsible for the support of
the family. The expenses for
Held: such support and other
conjugal obligations shall be
Taking into consideration the legislative intent paid from the community
and applying the rule of reason, we hold that property and, in the absence
Paragraph 2 of Article 26 should be thereof, from the income or
interpreted to include cases involving fruits of their separate
parties who, at the time of the properties. In case of
celebration of the marriage were Filipino insufficiency or absence of said
citizens, but later on, one of them becomes income or fruits, such
naturalized as a foreign citizen and obtains a obligations shall be satisfied
divorce decree. The Filipino spouse should from the separate properties.
likewise be allowed to remarry as if the (111a)
other party were a foreigner at the time
of the solemnization of the marriage. To Art. 73. Either spouse may exercise any
rule otherwise would be to sanction absurdity legitimate profession,
and injustice. Where the interpretation of a occupation, business or activity
statute according to its exact and literal import without the consent of the
would lead to mischievous results or other. The latter may object
contravene the clear purpose of the legislature, only on valid, serious, and
it should be construed according to its spirit moral grounds.
and reason, disregarding as far as necessary
the letter of the law. A statute may therefore In case of disagreement, the
be extended to cases not within the literal court shall decide whether or not:
meaning of its terms, so long as they come
within its spirit or intent.
1. The objection is proper;
and
In view of the foregoing, we state the twin 2. Benefit has occurred to the
elements for the application of Paragraph 2 of family prior to the objection
Article 26 as follows: or thereafter. If the benefit
accrued prior to the
1. There is a valid marriage that has objection, the resulting
been celebrated between a Filipino obligation shall be enforced
citizen and a foreigner; and against the separate
2. A valid divorce is obtained abroad property of the spouse who
by the alien spouse capacitating has not obtained consent.
him or her to remarry.
The foregoing provisions shall
The reckoning point is not the citizenship not prejudice the rights of
of the parties at the time of the creditors who acted in good
celebration of the marriage, but their faith. (117a)
citizenship at the time a valid divorce is
obtained abroad by the alien spouse Muslim Code
capacitating the latter to remarry.

VI. CONSEQUENCES OF MARRIAGE Art. 34. Mutual rights and obligations.



1. PERSONAL RELATIONS
1. The husband and the wife
Civil Code Art. 15, supra are obliged to live together,
observe mutual respect
and fidelity, and render
Family Code
mutual help and support in
accordance with this Code.
Art. 69. The husband and wife shall fix 2. When one of the spouses
the family domicile. In case of
neglects his or her duties
disagreement, the court shall
to the conjugal union or
decide.
brings danger, dishonor or
material injury upon the
The court may exempt one other, the injured party
spouse from living with the may petition the court for
other if the latter should live relief. The court may
abroad or there are other valid

Private International Law Page 40


counsel the offender to 6. The wife shall be entitled to an
comply with his or her equal and just treatment by the
duties, and take such husband.
measures as may be
proper. 2. PROPERTY RELATIONS
3. The husband and the wife
shall inherit from each
Civil Code:
other in accordance with
this Code.
4. The husband and the wife Art. 15, supra
shall have the right to
divorce in accordance with Art. 117. The wife may exercise any
this Code. profession or occupation or
engage in business. However,
the husband may object,
Art. 35. Rights and obligations of the
provided:
husband. — The husband shall
fix the residence of the family.
The court may exempt the wife 1. His income is
from living with her husband on sufficient for the family,
any of the following grounds: according to its social
standing, and
a. Her dower is not 2.His opposition is
founded on serious and
satisfied in accordance with
valid grounds.
the stipulations; or
b. The conjugal dwelling
is not in keeping with her In case of disagreement on this
social standing or is, for question, the parents and
any reason, not safe for the grandparents as well as the
members of the family or family council, if any, shall be
her property. consulted. If no agreement is
still arrived at, the court will
decide whatever may be
Art. 36. Rights and obligations of the wife. — proper and in the best interest
of the family. (n)
1. The wife shall dutifully manage
the affairs of the household. Art. 118. The property relations between
She may purchase things husband and wife shall be
necessary for the maintenance governed in the following
of the family, and the husband order:
shall be bound to reimburse a. By contract executed
the expenses, if he has not before the marriage;
delivered the proper sum. b. By the provisions of this
2. The wife cannot, without the Code; and
husband's consent, acquire any c. By custom. (1315a)
property by gratuitous title,
except from her relatives who
Art. 124. If the marriage is between a
are within the prohibited
citizen of the Philippines and a
degrees in marriage.
foreigner, whether celebrated
3. The wife may, with her in the Philippines or abroad,
husband's consent, exercise the following rules shall prevail:
any profession or occupation or
engage in lawful business
which is in keeping with Islamic 1. If the husband is a citizen
modesty and virtue. However, of the Philippines while the
if the husband refuses to give wife is a foreigner, the
his consent on the ground that provisions of this Code
his income is sufficient for the shall govern their relations;
family according to its social 2. If the husband is a
standing or his opposition is foreigner and the wife is a
based on serious and valid citizen of the Philippines,
grounds, the matter shall be the laws of the husband's
referred to the Agama country shall be followed,
Arbitration Council. without prejudice to the
4. The wife shall have the right to provisions of this Code with
demand the satisfaction of her regard to immovable
mahr. property. (1325a)
5. Unless otherwise stipulated in
the marriage settlements, the Art. 144. When a man and a woman live
wife retain ownership and together as husband and wife,
administration of her exclusive but they are not married, or
property. their marriage is void from the
beginning, the property

Private International Law Page 41


acquired by either or both of and female respectively,
them through their work or even with the consent of
industry or their wages and the parents;
salaries shall be governed by 2. Those solemnized by any
the rules on co-ownership. (n) person not legally
authorized to perform
Family Code: marriages;
3. Those solemnized without
Art. 74. A marriage in articulo mortis a marriage license, save
may also be solemnized by the marriages of exceptional
captain of a ship or chief of an character;
airplane during a voyage, or by 4. Bigamous or polygamous
the commanding officer of a marriages not falling under
military unit, in the absence of Article 83, Number 2;
a chaplain, during war. The 5. Incestuous marriages
duties mentioned in the two mentioned in Article 81;
preceding articles shall be 6. Those where one or both
complied with by the ship contracting parties have
captain, airplane chief or been found guilty of the
commanding officer. (n) killing of the spouse of
either of them;
Art. 75. Marriages between Filipino 7. Those between
citizens abroad may be stepbrothers and
solemnized by consuls and stepsisters and other
vice-consuls of the Republic of marriages specified in
the Philippines. The duties of Article 82. (n)
the local civil registrar and of a
judge or justice of the peace or Art. 147. When a man and a woman who
mayor with regard to the are capacitated to marry each
celebration of marriage shall be other, live exclusively with
performed by such consuls and each other as husband and
vice-consuls. (n) wife without the benefit of
marriage or under a void
Art. 76. No marriage license shall be marriage, their wages and
necessary when a man and a salaries shall be owned by
woman who have attained the them in equal shares and the
age of majority and who, being property acquired by both of
unmarried, have lived together them through their work or
as husband and wife for at industry shall be governed by
least five years, desire to marry the rules on co-ownership.
each other. The contracting
parties shall state the In the absence of proof to the
foregoing facts in an affidavit contrary, properties acquired
before any person authorized while they lived together shall
by law to administer oaths. The be presumed to have been
official, priest or minister who obtained by their joint efforts,
solemnized the marriage shall work or industry, and shall be
also state in an affidavit that owned by them in equal
he took steps to ascertain the shares. For purposes of this
ages and other qualifications of Article, a party who did not
the contracting parties and that participate in the acquisition by
he found no legal impediment the other party of any property
to the marriage. (n) shall be deemed to have
contributed jointly in the
Art. 77. In case two persons married in acquisition thereof if the
accordance with law desire to former's efforts consisted in
ratify their union in conformity the care and maintenance of
with the regulations, rites, or the family and of the
practices of any church, sect, household.
or religion it shall no longer be
necessary to comply with the Neither party can encumber or
requirements of Chapter 1 of dispose by acts inter vivos of
this Title and any ratification his or her share in the property
made shall merely be acquired during cohabitation
considered as a purely religious and owned in common, without
ceremony. (23) the consent of the other, until
after the termination of their
Art. 80. The following cohabitation.
marriages shall be void from the beginning:
1. Those contracted under the When only one of the parties to
ages of sixteen and a void marriage is in good faith,
fourteen years by the male the share of the party in bad

Private International Law Page 42


faith in the co-ownership shall contract, shall be governed by
be forfeited in favor of their the regime of complete
common children. In case of separation of property in
default of or waiver by any or accordance with this Code and,
all of the common children or in a suppletory manner, by the
their descendants, each vacant general principles of Islamic
share shall belong to the law and the Civil Code of the
respective surviving Philippines.
descendants. In the absence of
descendants, such share shall VII. Dissolution of Marriage
belong to the innocent party. In
all cases, the forfeiture shall
take place upon termination of 1. Annulment
the cohabitation. (144a)
Civil Code:
Art. 148. In cases of cohabitation not
falling under the preceding Art. 66. When either or both of the
Article, only the properties contracting parties are citizens
acquired by both of the parties or subjects of a foreign
through their actual joint country, it shall be necessary,
contribution of money, before a marriage license can
property, or industry shall be be obtained, to provide
owned by them in common in themselves with a certificate of
proportion to their respective legal capacity to contract
contributions. In the absence of marriage, to be issued by their
proof to the contrary, their respective diplomatic or
contributions and consular officials. (13a)
corresponding shares are
presumed to be equal. The Art. 71. All marriages performed
same rule and presumption outside the Philippines in
shall apply to joint deposits of accordance with the laws in
money and evidences of credit. force in the country where they
were performed, and valid
If one of the parties is validly there as such, shall also be
married to another, his or her valid in this country, except
share in the co-ownership shall bigamous, polygamous, or
accrue to the absolute incestuous marriages as
community or conjugal determined by Philippine law.
partnership existing in such (19a)
valid marriage. If the party who
acted in bad faith is not validly Family Code
married to another, his or her
shall be forfeited in the manner
provided in the last paragraph Art. 45. A marriage may be annulled for
of the preceding Article. any of the following causes,
existing at the time of the
marriage:
The foregoing rules on
forfeiture shall likewise apply
even if both parties are in bad 1. That the party in whose
faith. (144a) behalf it is sought to have
the marriage annulled was
eighteen years of age or
Muslim Code: over but below twenty-one,
and the marriage was
Art. 37. How governed. — The property solemnized without the
relations between husband and consent of the parents,
wife shall be governed in the guardian or person having
following order: substitute parental
authority over the party, in
that order, unless after
a. By contract before or at the attaining the age of
time of the celebration of
twenty-one, such party
marriage;
freely cohabited with the
b. By the provisions of this other and both lived
Code; and together as husband and
c. By custom. wife;
2. That either party was of
Art. 38. Regime of property relations. unsound mind, unless such
— The property relations party after coming to
between the spouses, in the reason, freely cohabited
absence of any stipulation to with the other as husband
the contrary in the marriage and wife;
settlements or any other 3. That the consent of either
party was obtained by

Private International Law Page 43


fraud, unless such party age of twenty-one, or by
afterwards, with full the parent or guardian or
knowledge of the facts person having legal charge
constituting the fraud, of the minor, at any time
freely cohabited with the before such party has
other as husband and wife; reached the age of twenty-
4. That the consent of either one;
party was obtained by 2. For causes mentioned in
force, intimidation or undue number 2 of Article 45, by
influence, unless the same the same spouse, who had
having disappeared or no knowledge of the
ceased, such party other's insanity; or by any
thereafter freely cohabited relative or guardian or
with the other as husband person having legal charge
and wife; of the insane, at any time
5. That either party was before the death of either
physically incapable of party, or by the insane
consummating the spouse during a lucid
marriage with the other, interval or after regaining
and such incapacity sanity;
continues and appears to 3. For causes mentioned in
be incurable; or number 3 of Article 45, by
6. That either party was the injured party, within
afflicted with a sexually- five years after the
transmissible disease found discovery of the fraud;
to be serious and appears 4. For causes mentioned in
to be incurable. (85a) number 4 of Article 45, by
the injured party, within
Art. 46. Any of the following five years from the time
circumstances shall constitute the force, intimidation or
fraud referred to in Number 3 undue influence
of the preceding Article: disappeared or ceased;
5. For causes mentioned in
1. Non-disclosure of a number 5 and 6 of Article
previous conviction by final 45, by the injured party,
judgment of the other party within five years after the
of a crime involving moral marriage. (87a)
turpitude;
2. Concealment by the wife of Art. 48. In all cases of annulment or
the fact that at the time of declaration of absolute nullity
the marriage, she was of marriage, the Court shall
pregnant by a man other order the prosecuting attorney
than her husband; or fiscal assigned to it to
3. Concealment of sexually appear on behalf of the State
transmissible disease, to take steps to prevent
regardless of its nature, collusion between the parties
existing at the time of the and to take care that evidence
marriage; or is not fabricated or suppressed.
4. Concealment of drug
addiction, habitual In the cases referred to in the
alcoholism or preceding paragraph, no
homosexuality or judgment shall be based upon
lesbianism existing at the a stipulation of facts or
time of the marriage. confession of judgment. (88a)

No other misrepresentation or Art. 49. During the pendency of the


deceit as to character, health, action and in the absence of
rank, fortune or chastity shall adequate provisions in a
constitute such fraud as will written agreement between the
give grounds for action for the spouses, the Court shall
annulment of marriage. (86a) provide for the support of the
spouses and the custody and
Art. 47. The action for annulment of support of their common
marriage must be filed by the children. The Court shall give
following persons and within paramount consideration to the
the periods indicated herein: moral and material welfare of
said children and their choice
1. For causes mentioned in of the parent with whom they
number 1 of Article 45 by wish to remain as provided to
the party whose parent or in Title IX. It shall also provide
guardian did not give his or for appropriate visitation rights
her consent, within five of the other parent. (n)
years after attaining the

Private International Law Page 44


Art. 50. The effects provided for by the appropriate civil registry
paragraphs (2), (3), (4) and (5) and registries of property;
of Article 43 and by Article 44 otherwise, the same shall not
shall also apply in the proper affect third persons. (n)
cases to marriages which are
declared ab initio or annulled Art. 53. Either of the former spouses
by final judgment under may marry again after
Articles 40 and 45. compliance with the
requirements of the
The final judgment in such immediately preceding Article;
cases shall provide for the otherwise, the subsequent
liquidation, partition and marriage shall be null and
distribution of the properties of void.chan robles virtual law
the spouses, the custody and library
support of the common
children, and the delivery of Art. 54. Children conceived or born
third presumptive legitimes, before the judgment of
unless such matters had been annulment or absolute nullity
adjudicated in previous judicial of the marriage under Article
proceedings. 36 has become final and
executory shall be considered
All creditors of the spouses as legitimate. Children conceived
well as of the absolute or born of the subsequent
community or the conjugal marriage under Article 53 shall
partnership shall be notified of likewise be legitimate.
the proceedings for liquidation.
Art. 55. A petition for legal
In the partition, the conjugal separation may be filed on any of the following
dwelling and the lot on which it grounds:
is situated, shall be adjudicated
in accordance with the 1. Repeated physical
provisions of Articles 102 and violence or grossly
129. abusive conduct
directed against the
Art. 51. In said partition, the value of petitioner, a common
the presumptive legitimes of all child, or a child of the
common children, computed as petitioner;
of the date of the final 2. Physical violence or
judgment of the trial court, moral pressure to
shall be delivered in cash, compel the petitioner
property or sound securities, to change religious or
unless the parties, by mutual political affiliation;
agreement judicially approved, 3. Attempt of respondent
had already provided for such to corrupt or induce the
matters. petitioner, a common
child, or a child of the
petitioner, to engage in
The children or their guardian prostitution, or
or the trustee of their property connivance in such
may ask for the enforcement of corruption or
the judgment. inducement;
4. Final judgment
The delivery of the sentencing the
presumptive legitimes herein respondent to
prescribed shall in no way imprisonment of more
prejudice the ultimate than six years, even if
successional rights of the pardoned;
children accruing upon the 5. Drug addiction or
death of either of both of the habitual alcoholism of
parents; but the value of the the respondent;
properties already received 6. Lesbianism or
under the decree of annulment homosexuality of the
or absolute nullity shall be respondent;
considered as advances on 7. Contracting by the
their legitime. (n) respondent of a
subsequent bigamous
Art. 52. The judgment of annulment or marriage, whether in
of absolute nullity of the the Philippines or
marriage, the partition and abroad;
distribution of the properties of 8. Sexual infidelity or
the spouses and the delivery of perversion;
the children's presumptive
legitimes shall be recorded in

Private International Law Page 45


9. Attempt by the ROEHR VS. RODRIGUEZ
respondent against the
life of the petitioner; or
10. Abandonment of Facts:
petitioner by
respondent without Petitioner Wolfgang Roehr, a German citizen,
justifiable cause for married a Filipina, Carmen Rodriguez in
more than one year. Germany. The marriage was ratified in
Tayasan, Negros Oriental. Private respondent
For purposes of this Article, the filed a petition for the declaration of nullity of
term "child" shall include a marriage before the RTC of Makati. Petitioner
child by nature or by adoption. filed a motion to dismiss but was denied by the
(9a) trial court. The petitioner obtained a decree of
divorce from the Court of First Instance of
Hamburg - Blankenese and granting the
2. Absolute Divorce custody of the children to the father.

Civil Code: Issue:

Art. 15, supra Whether or not the legal effects of a divorce


obtained from a foreign country such as
support and custody of the children can be
Art. 17, Supra determined in our courts?

Muslim Code: Held:

45-55 supra Yes. In order to take effect, a foreign


judgement must clearly show that the
opposing party has been given ample
Art. 27. By a husband. — Notwithstanding the opportunity to do so under the Rules of
rule of Islamic law permitting a Civil Procedure. Accordingly, the respondent
Muslim to have more than one was not given the opportunity to challenge the
wife but one wife unless he can judgement of the German Court, therefore,
deal with them with equal legal effects of divorce must be determined in
companionship and just our courts. The court held that the trial court
treatment as enjoined by has jurisdiction over the issue between the
Islamic law and only in parties as to who has the parental custody.
exceptional cases.

Art. 34. Mutual rights and obligations. RECTO VS. HARDEN (supra)

1. The husband and the wife


are obliged to live together, GONZALES VS. GONZALES
observe mutual respect
and fidelity, and render
mutual help and support in
accordance with this Code. Facts:
2. When one of the spouses
neglects his or her duties Plaintiff and defendant are citizens of the
to the conjugal union or Philippine Islands and at present residents of
brings danger, dishonor or the City of Manila. They were married in the
material injury upon the City of Manila on January 19, 1919, and lived
other, the injured party together as man and wife in the Philippine
may petition the court for Islands until the spring of 1926. They
relief. The court may voluntarily separated and since that time have
counsel the offender to not lived together as man and wife. Of this
comply with his or her union four children were born. Negotiations
duties, and take such between the parties, both being represented
measures as may be by attorneys, whereupon it was mutually
proper. .chan robles virtual agreed to allow the plaintiff for her support and
law library that of her children, five hundred pesos (P500)
3. The husband and the wife monthly; this amount to be increased in case of
shall inherit from each illness or necessity, and the title of certain
other in accordance with properties to be put in her name. Shortly after
this Code. this agreement the husband left the Islands,
4. The husband and the betook himself to Reno, Nevada, and secured
wife shall have the right in that jurisdiction an absolute divorce on the
to divorce in accordance ground of desertion. Shortly thereafter the
with this Code. defendant moved to California and returned to
these Islands in August 1928, where he has
since remained. On the same date that he
secured a divorce in Nevada he went through
the forms of marriage with another citizen of

Private International Law Page 46


these Islands and now has three children as a jurisdiction, except it be for a cause, and under
result of that marriage. Defendant, after his conditions for which the courts of Philippine
departure from these Islands, reduced the Islands would grant a divorce. The lower court
amount he had agreed to pay monthly for the in granting relief as prayed for frankly stated
support of his wife and four minor children and that the securing of the divorce, the
has not made the payments fixed in the Reno contracting of another marriage and the
divorce as alimony. bringing into the world of innocent children
brings about such a condition that the court
Issue: must grant relief. The hardships of the existing
divorce laws of the Philippine Islands are well
Whether or not that the divorced acquired in known to the members of the Legislature. It is
Nevada is valid here in the Philippines? of no moment in this litigation what he
personal views of the writer on the subject of
Held divorce may be. It is the duty of the courts to
enforce the laws of divorce as written by the
While the parties in this action are in dispute Legislature if they are constitutional. Courts
over financial matters they are in unity in have no right to say that such laws are too
trying to secure the courts of this jurisdiction to strict or too liberal.
recognize and approve of the Reno divorce. On
the record here presented this can not be TENCHAVEZ V. ESCANO 15 SCRA 355
done. The public policy in this jurisdiction on
the question of divorce is clearly set forth in
Act No. 2710, and the decisions of this court. Facts:

• Pastor Tenchavez married Vicenta Escano


The entire conduct of the parties from the time on Feb. 24, 1948 in Cebu City. As of June
of their separation until the case was 1948, the newly-weds were already
submitted to this court, in which they all estranged. Vicenta left for the US and filed
prayed that the Reno divorce be ratified and a verified complaint for divorce against the
confirmed, clearly indicates a purpose to Pastor in the State of Nevada on the
circumvent the laws of the Philippine Islands ground of "extreme cruelty, entirely mental
regarding divorce and to secure for themselves in character."
a change of status for reasons and under • A decree of divorce was issued by the
conditions not authorized by our law. At all Nevada Court. Later on, Vicenta married an
times the matrimonial domicile of this couple American, Russell Leo Moran in Nevada.
has been within the Philippine Islands and the She now lives with him in California and
residence acquired in the State of Nevada by has begotten children. She acquired
the husband of the purpose of securing a American citizenship on August 8, 1958. On
divorce was not a bona fide residence and did July 30, 1955, Pastor filed a complaint for
not confer jurisdiction upon the Court of that legal separation and damages against
State to dissolve the bonds if matrimony in Vicenta and her parents in the CFI-Cebu.
which he had entered in 1919. While the
decisions of this court heretofore in refusing to HELD:
recognize the validity of foreign divorce has
usually been expressed in the negative and
have been based upon lack of matrimonial • At the time the divorce decree was
domicile or fraud or collusion, we have not issued, Vicenta, like her husband, was
overlooked the provisions of the Civil Code now still a Filipino citizen. She was then
in force in these Islands. Article 9 thereof reads subject to Philippine laws under Art.
as follows: 15 of the New Civil Code. Philippine law,
under the NCC then now in force, does not
admit absolute divorce but only provides
The laws relating to family rights
for legal separation.
and duties, or to the status,
condition and legal capacity or • For Philippine courts to recognize
persons, are binding upon foreign divorce decrees between
Spaniards even though they reside Filipino citizens would be a patent
in a foreign country. violation of the declared policy of the
State, especially in view of the 3rd
And article 11, the last part of which reads: par. of Art. 17, of the New Civil Code
which reads: Prohibitive laws
concerning persons, their acts or
. . . the prohibitive laws concerning property, and those which have, for
persons, their acts and their their object, public order, public policy
property, and those intended to and good customs shall not be
promote public order and good rendered ineffective by laws or
morals, shall nor be rendered judgments promulgated, or by
without effect by any foreign laws determinations or conventions agreed
or judgments or by anything done upon in a foreign country. Moreover,
or any agreements entered into a recognition would give rise to scandalous
foreign country. discrimination in favor of wealthy citizens
to the detriment of those members of our
It is therefore a serious question whether any society whose means do not permit them
foreign divorce relating to citizens of the to sojourn abroad and obtain absolute
Philippine Islands, will be recognized in this divorce outside the Philippines.

Private International Law Page 47


• Therefore, a foreign divorce between enforced outside of the territorial
Filipino citizens, sought and decreed after jurisdiction of the court.
the effectivity of the NCC, is not entitled to • Plaintiff-appellant had made her choice of
recognition as valid in this jurisdiction. two inconsistent remedies afforded her by
law: (1) to impugn the divorce and file
SIKAT V. CANSON an action for support, or (2) uphold
67 PHIL 207 the validity of the divorce and sue for
a liquidation of conjugal partnership.
• Hilaria Sikat and John Canson contracted She chose the first remedy when she
marriage and lived together as husband filed her action for support. She lost the
and wife until 1911 when they separated. case and should take the consequences.
• During the same year the wife commenced • The courts in the Philippines can grant
divorce proceedings against her spouses, a divorce only on the ground of
but the case was dismissed without the "adultery on the part of the wife or
court passing upon the merits thereof. concubinage on the part of the
• At the time of their marriage in 1904, John husband" as provided for under
Canson was an Italian citizen but on section 1 of Act No. 2710. The divorce
February 27, 1922, he became a decree in question was granted on the
naturalized Filipino citizen. ground of desertion, clearly not a cause for
• In 1929, he went to Reno, Nevada, United divorce under our laws.
States of America, and on October 8, of • That our divorce law, Act No. 2710, is too
that year, he obtained an absolute decree strict or too liberal is not for this court to
of divorce on the ground of desertion. decide. (Barretto Gonzalez vs. Gonzalez,
• Hilaria, in 1933, filed another action, civil supra.)
case No. 5398 of the Court of First Instance • The allotment of powers between the
of Rizal, wherein she sought to compel the different governmental agencies
defendant to pay her a monthly pension of restricts the judiciary within the
P500 as alimony or support. confines of interpretation, not of
• Canson interposed three defenses: (1) legislation. The legislative policy on the
adultery on the part of the plaintiff: (2) matter of divorce in this jurisdiction is
absolute divorce obtained by the defendant clearly set forth in Act No. 2710 and has
as decreed by the court in Reno, Nevada, been upheld by this court;
United States of America; and (3) that the
defendant did not have the means to pay ARCA V. JAVIER
the allowance sought. The lower court 95 PHIL 579
dismissed the complaint and declined to
accord validity to the divorce obtained in • Dissatisfied with the decision of the
Reno but found that Hilaria Sikat had Court of First Instance of Cavite ordering
forfeited her right to support because she him to give a monthly allowance of P60 to
had committed adultery. plaintiffs beginning March 31, 1953, and to
• This judgment was not appealed and it pay them attorney's fees in the amount of
became final. P150 defendant took the case directly to
• On June 1, 1934, the present action was this Court attributing five errors to the
instituted by the plaintiff-appellant to court below. The facts are not disputed.
obtain the liquidation of the conjugal
partnership. The action is predicated on • Javier and Arca got married in Manila.
the existence of a final decree of absolute Javier, an enlisted US Navy personnel left
divorce rendered by the court of Reno, for the States 7 years after the birth of
Nevada, since 1929. their first born. At such time, Arca lived
with Javier’s parents. However, due to
Held: strained relations with the latter, she left
and transferred to her hometown.
• It is not, however, the citizenship of Thereafter, Javier filed a case for divorce in
the plaintiff for divorce which confers Alabama against Arca alleging
jurisdiction upon a court, but his legal abandonment. Arca made her reply
residence within the State (Cousins Hix claiming among other things that she
vs. Fluemer, 55 Phil. 851). And assuming never abandoned her husband and that
that John Canson acquired legal residence their separation was due to a physical
in the State of Nevada through the impossibility which justifies her separation
approval of his citizenship papers, this did if the husband moves to ultra-marine
not confer jurisdiction on the Nevada court colonies. The divorce however was
to grant a divorce that would be valid in granted.
this jurisdiction nor jurisdiction that could • Javier got married the 2nd time but was
determine their matrimonial status, subsequently divorced. After a few years,
because the wife was still domiciled in the he went back to the Philippines, and
Philippines. The Nevada court never believing that the first two divorces were
acquired jurisdiction over her person. valid, remarried the 3rd time.
(Gorayeb vs. Hashim, 50 Phil. 26, and
Cousins Hix vs. Fluemer, supra.) This was Issue:
not a proceeding in rem to justify a
court in entering a decree as to the Did the Circuit Court of Mobile County acquire
res or marriage relation entitled to be jurisdiction of both spouses and effectively

Private International Law Page 48


rendered a judgment in rem when it granted Federico, and Ernesto. Later, Kauffman brought
divorce to Javier? Leona to Switzerland to recuperate her health.
A few years later, she fell for a Dr. Mory to
Held: whom she had a child, Leontina. She informed
Kauffman that she no longer wished to stay
• No, one of the essential conditions for with him to which the latter obtained a divorce
the validity of a decree of divorce is in France where Leona was in default. Leona,
that the court must have jurisdiction over after the divorce got married in London, and
the subject matter and in order that this after which bore two children from which the
may be acquired, plaintiff must be last childbirth caused Leona her life. The heirs
domiciled in good faith in the State in of Leona from the first marriage and the
which it is granted (Cousins Hix vs. second now claims the Estate of Samuel to
Fluemer, 55 Phil., 851, 856). which Ana Ramirez opposed since Leona is not
a recognized natural child.
• While it is true that Salud R. Arca filed an
answer in the divorce case instituted at the Held:
Mobile County in view of the summons
served upon her in this jurisdiction, but this The status of Leona Castro as recognized
action cannot be interpreted as placing her natural daughter of Samuel Bischoff is fully and
under the jurisdiction of the court because satisfactorily shown.
its only purpose was to impugn the
claim of appellant that his domicile or With reference to the right of the von
legal residence at that time was Kauffman children, it is enough to say
Mobile County, and to show that the that they are legitimate children, born to
ground of desertion imputed to her was their parents in lawful wedlock; and they
baseless and false. Such answer should are therefore entitled to participate in the
be considered as a special appearance inheritance which would have devolved upon
the purpose of which is to impugn the their mother, if she had survived the testator.
jurisdiction of the court over the case.
• It is established by the great weight The Court is of the opinion that the
of authority that the court of a decree of divorce upon which reliance is
country in which neither of the placed by the representation of the Mory
spouses is domiciled and to which one children cannot be recognized as valid in
or both of them may resort merely for the courts of the Philippine Islands. The
the purpose of obtaining a divorce has French tribunal has no jurisdiction to entertain
no jurisdiction to determine their an action for the dissolution of a marriage
matrimonial status; and a divorce contracted in these Islands by person
granted by such a court is not entitled to domiciled here, such marriage being
recognition elsewhere. (See Note to indissoluble under the laws then prevailing in
Succession of Benton, 59 L. R. A., 143) The this country.
voluntary appearance of the defendant
before such a tribunal does not invest the The evidence shows that the decree was
court with jurisdiction. (Andrews vs. entered against the defendant in default, for
Andrews, 188 U. S., 14; 47 L. ed., failure to answer, and there is nothing to show
366.) that she had acquired, or had attempted to
• It follows that, to give a court acquire, a permanent domicile in the City of
jurisdiction on the ground of the Paris. It is evident of course that the presence
plaintiff's residence in the State or of both the spouses in that city was due merely
country of the judicial forum, his to the mutual desire to procure a divorce from
residence must be bona fide. If a each other.
spouse leaves the family domicile and goes
to another State for the sole purpose of It is established by the great weight of
obtaining a divorce, and with no intention authority that the court of a country in
of remaining, his residence there is not which neither of the spouses is domiciled
sufficient to confer jurisdiction on the and to which one or both of them may
courts of the State. (Ramirez vs. Gmur, 82 resort merely for the purpose of obtaining
Phil., 855.) a divorce has no jurisdiction to determine
• But even if his residence had been taken their matrimonial status; and a divorce
up is good faith, and the court had granted by such a court is not entitled to
acquired jurisdiction to take cognizance of recognition elsewhere.
the divorce suit, the decree issued in his
favor is not binding upon the appellant; for It follows that, to give a court jurisdiction
the matrimonial domicile of the spouses on the ground of the plaintiff's residence
being the City of Manila; in the State or country of the judicial
forum, his residence must be bona fide. If
a spouse leaves the family domicile and goes
Ramirez v. Gmur to another State for the sole purpose of
42 Phil 855 obtaining a divorce, and with no intention of
remaining, his residence there is not sufficient
Leona Castro was the natural daughter of to confer jurisdiction on the courts of that
decedent Samuel Bischoff. Whereas Ana State. This is especially true where the cause
Ramirez was the latter's widow to which they of divorce is one not recognized by the laws of
had no children. Leona was married to the State of his own domicile.
Kauffman. They had 3 children, Elena,

Private International Law Page 49


reach of execution to satisfy the judgment debt
of Corominas.

VAN DORN V. ROMILLO


MANILA SURETY & FIDELITY VS. TEODORO 139 SCRA 139

FACTS: Facts:

Jose Corominas, Jr. and Sonia Lizares were • Petitioner Alice Reyes Van Dorn is a citizen
married in Iloilo on January 5, 1935. On of the Philippines while private respondent
November 29,1954, a decree of divorce was Richard Upton is a US citizen; they were
granted by the Court of the State of Nevada married in Hong Kong in 1972; after the
dissolving the bonds of matrimony between marriage, they established their residence
Sonia Lizares and Jose Corominas, Jr. . . . in the Philippines and begot 2 children;
Alicia filed for divorce in Nevada; the
Trinidad Teodoro met Jose Corominas, Jr. in parties were divorced in Nevada, US, in
Hongkong on October 30, 1955. . . . On March 1982; and petitioner has remarried also in
26, 1956, they went through a Buddhist Nevada, this time to Theodore Van Dorn.
wedding ceremony in Hongkong. Upon their • On June 18, 1983 Upton filed a suit against
return to the Philippines they took up residence petitioner in the RTC-Pasay, stating that
in a rented house at No. 2305 Agno Street . . . petitioner's business in Ermita, Manila (the
Manila. On September 5, 1961, plaintiff and Galleon Shop), is conjugal property and
Jose Corominas, Jr. were married for a second asking that petitioner be ordered to render
time on Washoe County, Nevada. U.S.A. an accounting of that business, and that
Upton be declared as having the right to
Additional Pertinent facts, also mentioned in manage the conjugal property.
the decision under review and controverted by
the parties, are that Sonia Lizares is still living Held:
and that the conjugal partnership formed by
her marriage to Corominas was dissolved by
the Juvenile and Domestic Relations Court of • Owing to the nationality principle
Manila upon their joint petition, the decree of embodied in Art. 13, NCC, only
dissolution having been issued on October 21, Philippine nationals are covered by
1957. Trinidad questioned the levy on the the policy against absolute divorce
property since the property in question was her the same being considered contrary to
paraphernal property. our concept of public policy and
morality. However, aliens may obtain
divorce abroad, which may be recognized
ISSUE:
in the Philippines provided they are valid
according to their national law.
Whether or not the properties in question are • In this case, the divorce in Nevada
conjugal? released private respondents from the
marriage from the standards of
RULING: American law, under which divorce
dissolves the marriage.
There is no doubt that the decree of • Court said that “Ours is not only a court
divorce granted by the Court of Nevada in of law but also a court of equity.” The
1954 is not valid under Philippine law, Court could not turn its back on its citizen
which has outlawed divorce altogether; when the foreign national itself benefited
that the matrimonial bonds between Jose from such divorce decree;
Corominas, Jr. and Sonia Lizares have not been
dissolved, although their conjugal partnership • Thus, pursuant to his national law,
was terminated in 1957; and that the former's Upton is no longer the husband of
subsequent marriage in Hongkong to Trinidad petitioner. He would have no standing to
Teodoro is bigamous and void. sue in the case below as petitioner's
husband who is entitled to exercise control
In the present case, however, we find no need over conjugal assets.
to pass on this question. The particular • To maintain, as Upton does, that under our
properties involved here which were admittedly laws, petitioner has to be considered still
acquired by respondent Teodoro, cannot be married to him and still subject to a wife's
deemed to belong to such co-ownership obligations under the NCC cannot be just.
because, as found by the trial court and Petitioner should not be obliged to live
confirmed by the Court of Appeals, the funds together with, observe respect and fidelity,
used in acquiring said properties were and render support to private respondent.
fruits of respondent's paraphernal The latter should not continue to be one of
investments which accrued before her her heirs w/ possible rights to conjugal
"marriage" to Corominas. In other words properties. She should not be discriminated
they were not acquired by either or both of the against in her own country if the ends of
partners in the void marriage through their justice are to be observed.
work or industry or their wages and salaries,
and hence cannot be the subject of co-
ownership under Article 144. They remain
respondent's exclusive properties, beyond the PILAPIL V. IBAY-SOMERA

Private International Law Page 50


174 SCRA 653 established and, as already demonstrated,
such status or capacity must indubitably
• Petitioner Imelda Manalaysay Pilapil, a exist as of the time he initiates the action.
Filipino citizen, and private respondent • In the present case, the fact that
Erich Ekkehard Geiling, a German national, private respondent obtained a valid
were married in the Federal Republic of divorce in his country, the Federal
Germany. The marriage started Republic of Germany, it is deemed
auspiciously enough, and the couple lived admitted. Said divorce and its legal
together for some time in Malate, Manila effects may be recognized in the
where their only child, Isabella Pilapil Philippines insofar as private respondent is
Geiling, was born on April 20, 1980. concerned in view of the nationality
• Thereafter, marital discord set in, with principle in our civil law on the matter of
mutual recriminations between the status of persons.
spouses, followed by a separation de facto • Therefore, private respondent, being
between them. no longer the husband of petitioner,
• After about three and a half years of had no legal standing to commence
marriage, private respondent initiated a the adultery case under the imposture
divorce proceeding against petitioner in that he was the offended spouse at
Germany. He claimed that there was failure the time he filed suit.
of their marriage and that they had been
living apart since April, 1982. Petitioner, on
the other hand, filed an action for legal
separation, support and separation of LLORENTE V. COURT OF APPEALS
property before the Regional Trial Court of GR No. 124371, November 23, 2000
Manila.
• Thereafter a decree of divorce was
FACTS:
promulgated. The records show that under
German law said court was locally and
internationally competent for the divorce
proceeding and that the dissolution of said Lorenzo Llorente and petitioner Paula
marriage was legally founded on and Llorente were married in 1937 in the
authorized by the applicable law of that Philippines. Lorenzo was an enlisted
foreign jurisdiction. serviceman of the US Navy. Soon after, he
left for the US where through
• More than five months after the naturalization, he became a US Citizen.
issuance of the divorce decree, private Upon his visit to his wife, he discovered that
respondent filed two complaints for she was living with his brother and a child was
adultery before the City Fiscal of Manila born. The child was registered as
alleging that, while still married to said illegitimate but the name of the father
respondent, petitioner "had an affair with a was left blank. Llorente filed a divorce in
certain William Chua as early as 1982 and California in which Paula was represented by
with yet another man named Jesus Chua counsel, John Riley, and actively participated in
sometime in 1983". the proceedings, which later on became final.
He married Alicia and they lived together for 25
Issue: years bringing 3 children. He made his last will
and testament stating that all his properties
WON the adultery case be sustained even will be given to his second marriage. He filed a
though there has already been a finality of a petition of probate that made or appointed
divorce decree. Alicia his special administrator of his estate.
Before the proceeding could be terminated,
Lorenzo died. Paula filed a letter of
Held: administration over Llorente’s estate. The trial
granted the letter and denied the motion for
• The law specifically provides that in reconsideration. An appeal was made to the
prosecutions for adultery and concubinage Court of Appeals, which affirmed and modified
the person who can legally file the the judgment of the Trial Court that she be
complaint should be the offended spouse, declared co-owner of whatever properties, she
and nobody else. and the deceased, may have acquired in their
• Corollary to such exclusive grant of 25 years of cohabitation.
power to the offended spouse to
institute the action, it necessarily ISSUE:
follows that such initiator must have
the status, capacity or legal
representation to do so at the time of Whether or not national law shall apply?
the filing of the criminal action. This is
a familiar and express rule in civil actions;
in fact, lack of legal capacity to sue, as a RULING:
ground for a motion to dismiss in civil
cases, is determined as of the filing of the “Art. 15. Laws relating to family rights and
complaint or petition. In these cases, duties, or to the status, condition and legal
therefore, it is indispensable that the status capacity of persons are binding upon
and capacity of the complainant to citizens of the Philippines, even though
commence the action be definitely living abroad.

Private International Law Page 51


“Art. 16. Real property as well as personal
property is subject to the law of the country Whether the divorce between respondent and
where it is situated. Editha Samson was proven;

First, there is no such thing as one American Whether respondent was proven to be legally
law. The "national law" indicated in Article 16 capacitated to marry petitioner;
of the Civil Code cannot possibly apply to
general American law. There is no such law Held:
governing the validity of testamentary
provisions in the United States. Each State of A divorce obtained abroad by an alien may be
the union has its own law applicable to its recognized in our jurisdiction, provided such
citizens and in force only within the State. It decree is valid according to the national law of
can therefore refer to no other than the law of the foreigner. However, the divorce decree
the State of which the decedent was a and the governing personal law of the
resident. Second, there is no showing that alien spouse who obtained the divorce
the application of the renvoi doctrine is must be proven. Our courts do not take
called for or required by New York State judicial notice of foreign laws and judgment;
law. hence, like any other facts, both the divorce
decree and the national law of the alien must
be alleged and proven according to our law on
“However, intestate and testamentary evidence.
succession, both with respect to the order of
succession and to the amount of successional Was the first divorce validly obtained and
rights and to the intrinsic validity of binding?
testamentary provisions, shall be regulated
by the national law of the person whose At the outset, the Court lays the following basic
succession is under consideration, legal principles; Philippine law does not provide
whatever may be the nature of the property for absolute divorce; hence, Philippine courts
and regardless of the country wherein said cannot grant it. A marriage between two
property may be found.” (emphasis ours) Filipinos cannot be dissolved even by a divorce
obtained abroad, because of Articles 1522 and
1723 of the Civil Code. In mixed marriages
Likewise, Lorenzo Llorente was already an involving a Filipino and a foreigner,
American citizen when he divorced Paula. Article 26 of the Family Code allows the
Such was also the situation when he married former to contract a subsequent marriage
Alicia and executed his will. As stated in in case the divorce is "validly obtained
Article 15 of the civil code, aliens may abroad by the alien spouse capacitating
obtain divorces abroad, provided that him or her to remarry." A divorce obtained
they are valid in their National Law. Thus abroad by a couple, who are both aliens, may
the divorce obtained by Llorente is valid be recognized in the Philippines, provided it is
because the law that governs him is not consistent with their respective national laws.
Philippine Law but his National Law since the
divorce was contracted after he became an Before a foreign divorce decree can be
American citizen. Furthermore, his National recognized by our courts, the party
Law allowed divorce. pleading it must prove the divorce as a
fact and demonstrate its conformity to
the foreign law allowing it. Presentation
The case was remanded to the court of origin solely of the divorce decree is insufficient. In
for determination of the intrinsic validity of the case at bar, Respondent only presented the
Lorenzo Llorente’s will and determination of divorce decree;
the parties’ successional rights allowing proof
of foreign law. Likewise, before a foreign judgment is
given presumptive evidentiary value, the
GARCIA V. RECIO document must first be presented and
October 2, 2001 admitted in evidence. A divorce obtained
abroad is proven by the divorce decree itself.
• Rederick Recio, a Filipino, married Indeed the best evidence of a judgment is the
Editha Samson, an Australian in Malabon judgment itself. The decree purports to be a
Rizal. However, on 1989, they got divorced written act or record of an act of an officially
in an Australian family court. body or tribunal of a foreign country.
• On 1992, Rederick became an
However, under Sections 24 and 25 of Rule
Australian Citizen. He later married
132, a writing or document may be proven as a
Petitioner in 1994 in Cabanatuan City.
public or official record of a foreign country by
• Thereafter, the two separated and either (1) an official publication or (2) a
petitioner filed a complaint for Declaration copy thereof attested by the officer
of Nullity of Marriage on the ground of having legal custody of the document. If
bigamy. the record is not kept in the Philippines,
• While the suit was pending, Rederick such copy must be (a) accompanied by a
was able to obtain a divorce decree in certificate issued by the proper
Australia. Trial Court declared the marriage diplomatic or consular officer in the
dissolved based on the subsequent divorce Philippine Foreign Service stationed in
decree obtained by the respondent. the foreign country in which the record is
Issues:

Private International Law Page 52


kept and (b) authenticated by the seal of concerned. The certificate mentioned in Article
his office. 21 of the Family Code would have been
sufficient to establish the legal capacity of
The divorce decree between respondent and respondent, had he duly presented it in court.
Editha Samson appears to be an authentic one A duly authenticated and admitted
issued by an Australian family court. However, certificate is prima facie evidence of legal
appearance is not sufficient; compliance with capacity to marry on the part of the alien
the aforementioned rules on evidence must be applicant for a marriage license.
demonstrated.
In the case at bar, there is absolutely no
Fortunately for respondent, this matter evidence that proves respondent's legal
was not objected to by the petitioner, capacity to marry petitioner.
thus by virtue of such waiver, is deemed
admitted as evidence. Based on the above records, the Court cannot
conclude that respondent, who was then a
Who has the burden of proving a foreign naturalized Australian citizen, was legally
law? capacitated to marry petitioner on January 12,
1994. The court a quo erred in finding that
Respondent has the burden of proof; The the divorce decree ipso facto clothed
burden of proof lies with "the party who respondent with the legal capacity to
alleges the existence of a fact or thing remarry without requiring him to adduce
necessary in the prosecution or defense sufficient evidence to show the Australian
of an action." It is a well-settled that courts personal law governing his status; or at the
cannot take judicial notice of foreign laws. Like very least, to prove his legal capacity to
any other facts, they must be alleged and contract the second marriage.
proved. Australian marital laws are not among
those matters that judges are supposed to Neither can the Court grant petitioner's
know by reason of their judicial function. The prayer to declare her marriage to
power of judicial notice must be exercised with respondent null and void on the ground of
caution, and every reasonable doubt upon the bigamy. After all, it may turn out that under
subject should be resolved in the negative. Australian law, he was really capacitated to
marry petitioner as a direct result of the
2nd Issue: Is Respondent legally divorce decree.
capacitated to remarry?
Hence, case was remanded to the court a quo
Divorce means the legal dissolution of a lawful for further determination of legal capacity and
union for a cause arising after marriage. But to receive evidence to determine if bigamy has
divorces are of different types. The two basic been committed;
ones are (1) absolute divorce or a vinculo
matrimonii and (2) limited divorce or a REPUBLIC V. ORBECIDO
mensa et thoro. The first kind terminates the GR NO. 154380, October 5, 2005
marriage, while the second suspends it and
leaves the bond in full force. There is no Facts:
showing in the case at bar which type of
divorce was procured by respondent.
On May 24, 1981, Cipriano Orbecido III
Respondent presented a decree nisi or an married Lady Myros M. Villanueva in the
interlocutory decree – a conditional or Philippines in Lam-an, Ozamis City. Their
provisional judgment of divorce. It is in effect marriage was blessed with a son and a
the same as a separation from bed and daughter.
board, although an absolute divorce may
follow after the lapse of the prescribed In 1986, Cipriano’s wife left for the United
period during which no reconciliation is States bringing along their son
effected. Kristoffer. A few years later, Cipriano
discovered that his wife had been naturalized
On its face, the herein Australian divorce as an American citizen.
decree contains a restriction that reads:
Sometime in 2000, Cipriano learned from his
"1. A party to a marriage who marries son that his wife had obtained a divorce
again before this decree becomes decree and then married a certain Innocent
absolute (unless the other party has died) Stanley. She, Stanley and her child by him
commits the offence of bigamy." currently live in San Gabriel, California.
This quotation bolsters the Court’s contention
that the divorce obtained by respondent may Cipriano thereafter filed with the trial
have been restricted. It did not absolutely court a petition for authority to remarry
establish his legal capacity to remarry invoking Paragraph 2 of Article 26 of the
according to his national law. Family Code. No opposition was filed. Finding
merit in the petition, the court granted the
Significance of the Certificate of Legal same. The Republic, herein petitioner,
Capacity through the Office of the Solicitor General
(OSG), sought reconsideration but it was
Legal capacity to contract marriage is denied.
determined by the national law of the party

Private International Law Page 53


The OSG contends that Paragraph 2 of Article Art. 99. No person shall be entitled to a
26 of the Family Code is not applicable to legal separation who has not
the instant case because it only applies to resided in the Philippines for
a valid mixed marriage; that is, a marriage one year prior to the filing of
celebrated between a Filipino citizen and an the petition, unless the cause
alien. Furthermore, the OSG argues there is no for the legal separation has
law that governs respondent’s situation. The taken place within the territory
OSG posits that this is a matter of legislation of this Republic. (Sec. 2a, Act
and not of judicial determination. No. 2710)

Held: TITLE II

LEGAL SEPARATION
Taking into consideration the legislative intent
and applying the rule of reason, we hold that Art. 55. A petition for legal separation may be
Paragraph 2 of Article 26 should be filed on any of the following grounds:
interpreted to include cases involving (1) Repeated physical violence or
parties who, at the time of the grossly abusive conduct directed
celebration of the marriage were Filipino against the petitioner, a common child,
citizens, but later on, one of them becomes or a child of the petitioner;
naturalized as a foreign citizen and obtains a
divorce decree. The Filipino spouse should
likewise be allowed to remarry as if the (2) Physical violence or moral pressure
other party were a foreigner at the time to compel the petitioner to change
of the solemnization of the marriage. To religious or political affiliation;
rule otherwise would be to sanction absurdity
and injustice. Where the interpretation of a (3) Attempt of respondent to corrupt or
statute according to its exact and literal import induce the petitioner, a common child,
would lead to mischievous results or or a child of the petitioner, to engage in
contravene the clear purpose of the legislature, prostitution, or connivance in such
it should be construed according to its spirit corruption or inducement;
and reason, disregarding as far as necessary
the letter of the law. A statute may therefore (4) Final judgment sentencing the
be extended to cases not within the literal respondent to imprisonment of more
meaning of its terms, so long as they come than six years, even if pardoned;
within its spirit or intent.
(5) Drug addiction or habitual
In view of the foregoing, we state the twin alcoholism of the respondent;
elements for the application of Paragraph 2 of
Article 26 as follows:
(6) Lesbianism or homosexuality of the
respondent;
3. There is a valid marriage that has
been celebrated between a Filipino
citizen and a foreigner; and (7) Contracting by the respondent of a
4. A valid divorce is obtained abroad subsequent bigamous marriage,
by the alien spouse capacitating whether in the Philippines or abroad;
him or her to remarry.
(8) Sexual infidelity or perversion;
The reckoning point is not the citizenship
of the parties at the time of the (9) Attempt by the respondent against
celebration of the marriage, but their the life of the petitioner; or
citizenship at the time a valid divorce is
obtained abroad by the alien spouse (10) Abandonment of petitioner by
capacitating the latter to remarry. respondent without justifiable cause for
more than one year.
5. Legal Separation
For purposes of this Article, the term "child"
Civil Code: shall include a child by nature or by adoption.
(9a)
Art. 97. A petition for legal
separation may be filed:
Art. 56. The petition for legal separation shall
1. For adultery on the part of be denied on any of the following grounds:
the wife and for
concubinage on the part of (1) Where the aggrieved party has
the husband as defined in condoned the offense or act
the Penal Code; or complained of;

2. An attempt by one spouse (2) Where the aggrieved party has


against the life of the consented to the commission of the
other. (n) offense or act complained of;

Private International Law Page 54


(3) Where there is connivance between absolute community or the conjugal
the parties in the commission of the partnership, which shall be forfeited in
offense or act constituting the ground accordance with the provisions of
for legal separation; Article 43(2);

(4) Where both parties have given (3) The custody of the minor children
ground for legal separation; shall be awarded to the innocent
spouse, subject to the provisions of
(5) Where there is collusion between Article 213 of this Code; and
the parties to obtain decree of legal
separation; or (4) The offending spouse shall be
disqualified from inheriting from the
(6) Where the action is barred by innocent spouse by intestate
prescription. (100a) succession. Moreover, provisions in
favor of the offending spouse made in
the will of the innocent spouse shall be
Art. 57. An action for legal separation shall be revoked by operation of law. (106a)
filed within five years from the time of the
occurrence of the cause. (102)
Art. 64. After the finality of the decree of legal
separation, the innocent spouse may revoke
Art. 58. An action for legal separation shall in the donations made by him or by her in favor
no case be tried before six months shall have of the offending spouse, as well as the
elapsed since the filing of the petition. (103) designation of the latter as beneficiary in any
insurance policy, even if such designation be
Art. 59. No legal separation may be decreed stipulated as irrevocable. The revocation of the
unless the Court has taken steps toward the donations shall be recorded in the registries of
reconciliation of the spouses and is fully property in the places where the properties are
satisfied, despite such efforts, that located. Alienations, liens and encumbrances
reconciliation is highly improbable. (n) registered in good faith before the recording of
the complaint for revocation in the registries of
Art. 60. No decree of legal separation shall be property shall be respected. The revocation of
based upon a stipulation of facts or a or change in the designation of the insurance
confession of judgment. beneficiary shall take effect upon written
notification thereof to the insured.
In any case, the Court shall order the
prosecuting attorney or fiscal assigned to it to The action to revoke the donation under this
take steps to prevent collusion between the Article must be brought within five years from
parties and to take care that the evidence is the time the decree of legal separation become
not fabricated or suppressed. (101a) final. (107a)

Art. 61. After the filing of the petition for legal Art. 65. If the spouses should reconcile, a
separation, the spouses shall be entitled to live corresponding joint manifestation under oath
separately from each other. duly signed by them shall be filed with the
court in the same proceeding for legal
separation. (n)
The court, in the absence of a written
agreement between the spouses, shall
designate either of them or a third person to Art. 66. The reconciliation referred to in the
administer the absolute community or conjugal preceding Articles shall have the following
partnership property. The administrator consequences:
appointed by the court shall have the same
powers and duties as those of a guardian under (1) The legal separation proceedings, if
the Rules of Court. (104a) still pending, shall thereby be
terminated at whatever stage; and
Art. 62. During the pendency of the action for
legal separation, the provisions of Article 49 (2) The final decree of legal separation
shall likewise apply to the support of the shall be set aside, but the separation of
spouses and the custody and support of the property and any forfeiture of the
common children. (105a) share of the guilty spouse already
effected shall subsist, unless the
Art. 63. The decree of legal separation shall spouses agree to revive their former
have the following effects: property regime.

(1) The spouses shall be entitled to live The court's order containing the foregoing shall
separately from each other, but the be recorded in the proper civil registries.
marriage bonds shall not be severed; (108a)

(2) The absolute community or the Art. 67. The agreement to revive the former
conjugal partnership shall be dissolved property regime referred to in the preceding
and liquidated but the offending Article shall be executed under oath and shall
spouse shall have no right to any share specify:
of the net profits earned by the

Private International Law Page 55


(1) The properties to be contributed
anew to the restored regime; Civil Code:

(2) Those to be retained as separated Art. 15, supra


properties of each spouse; and
Art. 335. The following cannot adopt:
(3) The names of all their known 1. Those who have
creditors, their addresses and the legitimate, legitimated,
amounts owing to each. acknowledged natural
children, or natural children
The agreement of revival and the motion for its by legal fiction;
approval shall be filed with the court in the 2. The guardian, with
same proceeding for legal separation, with respect to the ward, before
copies of both furnished to the creditors named the final approval of his
therein. After due hearing, the court shall, in its accounts;
order, take measure to protect the interest of 3. A married person,
creditors and such order shall be recorded in without the consent of the
the proper registries of properties. other spouse;
4. Non-resident aliens;
The recording of the ordering in the registries 5. Resident aliens with
of property shall not prejudice any creditor not whose government the
listed or not notified, unless the debtor-spouse Republic of the Philippines
has sufficient separate properties to satisfy the has broken diplomatic
creditor's claim. (195a, 108a) relations;
6. Any person who has
been convicted of a crime
Capacity to Remarry involving moral turpitude,
when the penalty imposed
Art. 26. All marriages solemnized was six months'
outside the Philippines, in imprisonment or more.
accordance with the laws in (174a)
force in the country where they
were solemnized, and valid Art. 339. The following cannot be
there as such, shall also be adopted:
valid in this country, except
those prohibited under Articles 1. A married person,
35 (1), (4), (5) and (6), 3637 without the written consent
and 38. (17a) of the other spouse;
2. An alien with whose
Where a marriage between government the Republic of
a Filipino citizen and a the Philippines has broken
foreigner is validly diplomatic relations;
celebrated and a divorce is 3. A person who has
thereafter validly obtained already been adopted. (n)
abroad by the alien spouse
capacitating him or her to Muslim Code:
remarry, the Filipino spouse
shall have capacity to TITLE V
remarry under Philippine Parental Authority
law. (As amended by
Executive Order 227) CHAPTER I
Nature and Effects
Nota bene:
Art. 71. Who exercises. —
Formal validity – law of the place of
celebration; lex loci contractus rule (1) The father and the
Substantial validity – governed by: mother shall jointly
exercise just and
1. Personal law: status/legal capacity – reasonable parental
National law of the parties authority and fulfill their
2. lex loci celebrationis: responsibility over their
 exceptions: legitimate and
• Family Code Art. 26, paragraph acknowledged children. In
1 case of disagreement, the
• Consular Marriages – consul father's decision shall
granted by the accepting prevail unless there is a
country where such consular judicial order to the
office was located to solemnize contrary.
marriage
(2) The mother shall
exercise parental authority
VIII. Parents and Children (Parental over her children born out
Relationship) of wedlock, but the court

Private International Law Page 56


may, when the best renounced nor transferred
interests of the children so except as otherwise provided
require, appoint a general in this Code and the general
guardian. .chan robles principles of Islamic law.
virtual law library
Art. 77. Extinguishment of parental
Art. 72. Duty to parents. — authority. —

(1) Children shall respect, (1) Parental authority


revere, and obey their terminates upon the death
parents always unless the of the parents or the child,
latter cast them into or upon emancipation.
disbelief. (2) Subject to Article 78,
the widowed mother who
(2) Grandparents are contracts a subsequent
likewise entitled to respect marriage shall lose
and reverence, and shall be parental authority and
consulted whenever custody over all children by
practicable by all members the deceased husband,
of the family on all unless the second husband
important questions. is related to them within
the prohibited degrees of
Art. 73. Duty to children. — Every consanguinity.
parent and every person (3) The court may deprive
exercising parental authority a person of parental
shall see to it that the rights of authority or suspend the
the children are respected, and exercise thereof if he treats
their duties complied with, and his children with excessive
shall particularly by precept harshness, gives then
and example, imbue them with corrupting or immoral
religious and civic attachment orders and counsel, or
to the ideal of permanent world abandons them.
peace.
CHAPTER II
Art. 74. Effects upon person of children. Custody and Guardianship
— The parents have, with
respect to their unemancipated Art. 78. Care and custody. —
children:
(1) The care and custody
(a) The duty to support of children below seven
them, have them in their years of age whose parents
company, educate and are divorced shall belong to
instruct them in keeping the mother or, in her
with their means and absence, to the maternal
represent them in all grandmother, the paternal
actions which shall redound grandmother, the sister
to their benefits; and and aunts. In their default,
it shall devolve upon the
(b) The power to correct, father and the nearest
discipline, and punish them paternal relatives. The
moderately. minor above seven years of
age but below the age of
Art. 75. Effects upon property of puberty may choose the
children. — parent with whom he wants
to stay.
(1) The father, or in his
absence the mother, shall (2) The unmarried
be the legal administrator daughter who has reached
of the property of the child the age of puberty shall
under parental authority. If stay with the father; the
the property is worth more son, under the same
than five thousand pesos, circumstances, shall stay
the father or the mother with the mother.
shall give a bond to be
approved by the court. Art. 79. Guardian for marriage (wali). —
The following persons shall
(2) The court may appoint have authority to act as
a guardian (wasi) in the guardian for marriage (wali) in
absence of one who is the order of precedence:
natural or testamentary.
(a) Father
Art. 76. Parental authority non- (b) Paternal grandfather;
transferable. — Parental (c) Brother and other
authority can neither be paternal relatives;

Private International Law Page 57


(d) Paternal grandfather's provided in the second paragraph of
executor or nominee; or Article 164; or
.chan robles virtual law
library (3) That in case of children conceived
(e) The court. through artificial insemination, the
written authorization or ratification of
Art. 80. Guardian of minor's property. either parent was obtained through
— The following persons shall mistake, fraud, violence, intimidation,
exercise guardianship over the or undue influence. (255a)
property of minors in the order
of precedence:
Art. 167. The child shall be considered
(a) Father; legitimate although the mother may have
(b) Father's executor or declared against its legitimacy or may have
nominee; been sentenced as an adulteress. (256a)
(c) Paternal grandfather;
(d) Paternal grandfather's Art. 168. If the marriage is terminated and the
nominee; or mother contracted another marriage within
(e) The court. three hundred days after such termination of
the former marriage, these rules shall govern
in the absence of proof to the contrary:
Art. 163. The filiation of children may be by
nature or by adoption. Natural filiation may be (1) A child born before one hundred
legitimate or illegitimate. (n) eighty days after the solemnization of
the subsequent marriage is considered
Art. 164. Children conceived or born during the to have been conceived during the
marriage of the parents are legitimate. former marriage, provided it be born
within three hundred days after the
Children conceived as a result of artificial termination of the former marriage;
insemination of the wife with the sperm of the
husband or that of a donor or both are likewise (2) A child born after one hundred
legitimate children of the husband and his wife, eighty days following the celebration of
provided, that both of them authorized or the subsequent marriage is considered
ratified such insemination in a written to have been conceived during such
instrument executed and signed by them marriage, even though it be born
before the birth of the child. The instrument within the three hundred days after the
shall be recorded in the civil registry together termination of the former marriage.
with the birth certificate of the child. (55a, (259a)
258a)
Art. 169. The legitimacy or illegitimacy of a
Art. 165. Children conceived and born outside a child born after three hundred days following
valid marriage are illegitimate, unless the termination of the marriage shall be proved
otherwise provided in this Code. (n) by whoever alleges such legitimacy or
illegitimacy. (261a)
Art. 166. Legitimacy of a child may be
impugned only on the following grounds: Art. 170. The action to impugn the legitimacy
of the child shall be brought within one year
(1) That it was physically impossible for from the knowledge of the birth or its recording
the husband to have sexual intercourse in the civil register, if the husband or, in a
with his wife within the first 120 days proper case, any of his heirs, should reside in
of the 300 days which immediately the city or municipality where the birth took
preceded the birth of the child because place or was recorded.
of:
If the husband or, in his default, all of his heirs
(a) the physical incapacity of do not reside at the place of birth as defined in
the husband to have sexual the first paragraph or where it was recorded,
intercourse with his wife; the period shall be two years if they should
reside in the Philippines; and three years if
(b) the fact that the husband abroad. If the birth of the child has been
and wife were living separately concealed from or was unknown to the
in such a way that sexual husband or his heirs, the period shall be
intercourse was not possible; or counted from the discovery or knowledge of
the birth of the child or of the fact of
registration of said birth, whichever is earlier.
(c) serious illness of the (263a)
husband, which absolutely
prevented sexual intercourse;
Art. 171. The heirs of the husband may impugn
the filiation of the child within the period
(2) That it is proved that for biological prescribed in the preceding article only in the
or other scientific reasons, the child following cases:
could not have been that of the
husband, except in the instance

Private International Law Page 58


(1) If the husband should died before The action must be brought within the same
the expiration of the period fixed for period specified in Article 173, except when the
bringing his action; action is based on the second paragraph of
Article 172, in which case the action may be
(2) If he should die after the filing of brought during the lifetime of the alleged
the complaint without having desisted parent. (289a)
therefrom; or
Art. 176. Illegitimate children shall use the
(3) If the child was born after the death surname and shall be under the parental
of the husband. (262a) authority of their mother, and shall be entitled
to support in conformity with this Code. The
legitime of each illegitimate child shall consist
Chapter 2. Proof of Filiation of one-half of the legitime of a legitimate child.
Except for this modification, all other provisions
Art. 172. The filiation of legitimate children is in the Civil Code governing successional rights
established by any of the following: shall remain in force. (287a)
(1) The record of birth appearing in the
civil register or a final judgment; or
Chapter 4. Legitimated Children
(2) An admission of legitimate filiation
in a public document or a private Art. 177. Only children conceived and born
handwritten instrument and signed by outside of wedlock of parents who, at the time
the parent concerned. of the conception of the former, were not
disqualified by any impediment to marry each
In the absence of the foregoing evidence, the other may be legitimated. (269a)
legitimate filiation shall be
Art. 178. Legitimation shall take place by a
proved by: subsequent valid marriage between parents.
The annulment of a voidable marriage shall not
(1) The open and continuous affect the legitimation. (270a)chan robles
possession of the status of a legitimate virtual law library
child; or
Art. 179. Legitimated children shall enjoy the
(2) Any other means allowed by the same rights as legitimate children. (272a)
Rules of Court and special laws. (265a,
266a, 267a) Art. 180. The effects of legitimation shall
retroact to the time of the child's birth. (273a)
Art. 173. The action to claim legitimacy may be
brought by the child during his or her lifetime Art. 181. The legitimation of children who died
and shall be transmitted to the heirs should the before the celebration of the marriage shall
child die during minority or in a state of benefit their descendants. (274)
insanity. In these cases, the heirs shall have a
period of five years within which to institute Art. 182. Legitimation may be impugned only
the action. by those who are prejudiced in their rights,
within five years from the time their cause of
Art. 174. Legitimate children shall have the action accrues. (275a)
right:
Art. 209. Pursuant to the natural right and duty
(1) To bear the surnames of the father of parents over the person and property of
and the mother, in conformity with the their unemancipated children, parental
provisions of the Civil Code on authority and responsibility shall include the
Surnames; caring for and rearing them for civic
consciousness and efficiency and the
(2) To receive support from their development of their moral, mental and
parents, their ascendants, and in physical character and well-being. (n)
proper cases, their brothers and
sisters, in conformity with the Art. 210. Parental authority and responsibility
provisions of this Code on Support; and may not be renounced or transferred except in
the cases authorized by law. (313a)
(3) To be entitled to the legitimate and
other successional rights granted to Art. 211. The father and the mother shall jointly
them by the Civil Code. (264a) exercise parental authority over the persons of
their common children. In case of
Chapter 3. Illegitimate Children disagreement, the father's decision shall
prevail, unless there is a judicial order to the
Art. 175. Illegitimate children may establish contrary.
their illegitimate filiation in the same way and
on the same evidence as legitimate children. Children shall always observe respect and
reverence towards their parents and are
obliged to obey them as long as the children

Private International Law Page 59


are under parental authority. (311a) chan Art. 218. The school, its administrators and
robles virtual law library teachers, or the individual, entity or institution
engaged in child are shall have special parental
Art. 212. In case of absence or death of either authority and responsibility over the minor
parent, the parent present shall continue child while under their supervision, instruction
exercising parental authority. The remarriage or custody.
of the surviving parent shall not affect the
parental authority over the children, unless the Authority and responsibility shall apply to all
court appoints another person to be the authorized activities whether inside or outside
guardian of the person or property of the the premises of the school, entity or institution.
children. (n) (349a)

Art. 213. In case of separation of the parents, Art. 129. Those given the authority and
parental authority shall be exercised by the responsibility under the preceding Article shall
parent designated by the Court. The Court shall be principally and solidarily liable for damages
take into account all relevant considerations, caused by the acts or omissions of the
especially the choice of the child over seven unemancipated minor. The parents, judicial
years of age, unless the parent chosen is unfit. guardians or the persons exercising substitute
(n) parental authority over said minor shall be
subsidiarily liable.
Art. 214. In case of death, absence or
unsuitability of the parents, substitute parental The respective liabilities of those referred to in
authority shall be exercised by the surviving the preceding paragraph shall not apply if it is
grandparent. In case several survive, the one proved that they exercised the proper diligence
designated by the court, taking into account required under the particular circumstances.
the same consideration mentioned in the
preceding article, shall exercise the authority. All other cases not covered by this and the
(355a) preceding articles shall be governed by the
provisions of the Civil Code on quasi-delicts. (n)
Art. 215. No descendant shall be compelled, in
a criminal case, to testify against his parents
and grandparents, except when such testimony
is indispensable in a crime against the Chapter 3. Effect of Parental Authority
descendant or by one parent against the other. Upon the Persons of the Children
(315a)
Art. 220. The parents and those exercising
parental authority shall have with the respect
Chapter 2. Substitute and Special Parental to their unemancipated children on wards the
Authority following rights and duties:
(1) To keep them in their company, to
Art. 216. In default of parents or a judicially support, educate and instruct them by
appointed guardian, the following person shall right precept and good example, and
exercise substitute parental authority over the to provide for their upbringing in
child in the order indicated: keeping with their means;
(1) The surviving grandparent, as
provided in Art. 214; (2) To give them love and affection,
advice and counsel, companionship
(2) The oldest brother or sister, over and understanding;
twenty-one years of age, unless unfit or
disqualified; and (3) To provide them with moral and
spiritual guidance, inculcate in them
(3) The child's actual custodian, over honesty, integrity, self-discipline, self-
twenty-one years of age, unless unfit or reliance, industry and thrift, stimulate
disqualified. their interest in civic affairs, and inspire
in them compliance with the duties of
Whenever the appointment or a judicial citizenship;
guardian over the property of the child
becomes necessary, the same order of (4) To furnish them with good and
preference shall be observed. (349a, 351a, wholesome educational materials,
354a) supervise their activities, recreation
and association with others, protect
Art. 217. In case of foundlings, abandoned them from bad company, and prevent
neglected or abused children and other them from acquiring habits detrimental
children similarly situated, parental authority to their health, studies and morals;
shall be entrusted in summary judicial
proceedings to heads of children's homes, (5) To represent them in all matters
orphanages and similar institutions duly affecting their interests;
accredited by the proper government agency.
(314a) (6) To demand from them respect and
obedience;

Private International Law Page 60


(7) To impose discipline on them as may determine, but not less than ten per
may be required under the centum (10%) of the value of the property or
circumstances; and annual income, to guarantee the performance
of the obligations prescribed for general
(8) To perform such other duties as are guardians.
imposed by law upon parents and
guardians. (316a) A verified petition for approval of the bond
shall be filed in the proper court of the place
Art. 221. Parents and other persons exercising where the child resides, or, if the child resides
parental authority shall be civilly liable for the in a foreign country, in the proper court of the
injuries and damages caused by the acts or place where the property or any part thereof is
omissions of their unemancipated children situated.
living in their company and under their
parental authority subject to the appropriate The petition shall be docketed as a summary
defenses provided by law. (2180(2)a and (4)a ) special proceeding in which all incidents and
issues regarding the performance of the
Art. 222. The courts may appoint a guardian of obligations referred to in the second paragraph
the child's property or a guardian ad litem of this Article shall be heard and resolved.
when the best interests of the child so requires.
(317) The ordinary rules on guardianship shall be
merely suppletory except when the child is
Art. 223. The parents or, in their absence or under substitute parental authority, or the
incapacity, the individual, entity or institution guardian is a stranger, or a parent has
exercising parental authority, may petition the remarried, in which case the ordinary rules on
proper court of the place where the child guardianship shall apply. (320a)
resides, for an order providing for disciplinary
measures over the child. The child shall be Art. 226. The property of the unemancipated
entitled to the assistance of counsel, either of child earned or acquired with his work or
his choice or appointed by the court, and a industry or by onerous or gratuitous title shall
summary hearing shall be conducted wherein belong to the child in ownership and shall be
the petitioner and the child shall be heard. devoted exclusively to the latter's support and
education, unless the title or transfer provides
However, if in the same proceeding the court otherwise.
finds the petitioner at fault, irrespective of the
merits of the petition, or when the The right of the parents over the fruits and
circumstances so warrant, the court may also income of the child's property shall be limited
order the deprivation or suspension of parental primarily to the child's support and secondarily
authority or adopt such other measures as it to the collective daily needs of the family.
may deem just and proper. (318a) (321a, 323a)

Art. 224. The measures referred to in the Art. 227. If the parents entrust the
preceding article may include the commitment management or administration of any of their
of the child for not more than thirty days in properties to an unemancipated child, the net
entities or institutions engaged in child care or proceeds of such property shall belong to the
in children's homes duly accredited by the owner. The child shall be given a reasonable
proper government agency. monthly allowance in an amount not less than
that which the owner would have paid if the
The parent exercising parental authority shall administrator were a stranger, unless the
not interfere with the care of the child owner, grants the entire proceeds to the child.
whenever committed but shall provide for his In any case, the proceeds thus give in whole or
support. Upon proper petition or at its own in part shall not be charged to the child's
instance, the court may terminate the legitime. (322a)
commitment of the child whenever just and
proper. (391a)
Chapter 5. Suspension or Termination of
Parental Authority
Chapter 4. Effect of Parental Authority Upon
the Property of the Children Art. 228. Parental authority terminates
permanently:
Art. 225. The father and the mother shall jointly (1) Upon the death of the parents;
exercise legal guardianship over the property
of the unemancipated common child without (2) Upon the death of the child; or
the necessity of a court appointment. In case of
disagreement, the father's decision shall (3) Upon emancipation of the child.
prevail, unless there is a judicial order to the (327a)
contrary.
Art. 229. Unless subsequently revived by a final
Where the market value of the property or the judgment, parental authority also terminates:
annual income of the child exceeds P50,000, (1) Upon adoption of the child;
the parent concerned shall be required to
furnish a bond in such amount as the court

Private International Law Page 61


(2) Upon appointment of a general exercising special parental authority inflict
guardian; corporal punishment upon the child. (n)

(3) Upon judicial declaration of Ching Leng v. Galang


abandonment of the child in a case 57 OG 2312
filed for the purpose;

(4) Upon final judgment of a competent “In Ching Leng the provision in the 1935
court divesting the party concerned of Constitution stating "those whose fathers are
parental authority; or citizens of the Philippines" refers only to
legitimate children. When the 1973 and 1987
(5) Upon judicial declaration of absence Constitutions were drafted, the framers did not
or incapacity of the person exercising attempt to change the intent of this provision,
parental authority. (327a) even as they were presumably aware of the
Ching Leng doctrine.
Art. 230. Parental authority is suspended upon
conviction of the parent or the person Nevertheless, I believe that it is now time to
exercising the same of a crime which carries abandon the Ching Leng doctrine. (Tecson V.
with it the penalty of civil interdiction. The Comelec)”
authority is automatically reinstated upon
service of the penalty or upon pardon or The prevailing doctrine today is that an
amnesty of the offender. (330a) illegitimate child of a Filipino father and an
alien mother follows the citizenship of the alien
Art. 231. The court in an action filed for the mother as the only legally known parent. The
purpose in a related case may also suspend illegitimate child, even if acknowledged and
parental authority if the parent or the person legally adopted by the Filipino father, cannot
exercising the same: acquire the citizenship of the father. The Court
made this definitive doctrinal ruling in Ching
Leng v. Galang which involved the illegitimate
(1) Treats the child with excessive minor children of a naturalized Filipino of
harshness or cruelty; Chinese descent with a Chinese woman, Sy An.
The illegitimate children were later on jointly
(2) Gives the child corrupting orders, adopted by the naturalized Filipino and his
counsel or example; legal wife, So Buan Ty.

(3) Compels the child to beg; or The facts in Ching Leng as quoted by the Court
from the trial court’s decision are as follows:
(4) Subjects the child or allows him to
be subjected to acts of lasciviousness. After the petitioner Ching Leng Alias
Ching Ban Lee obtained judgment in this
The grounds enumerated above are deemed to Court dated May 2, 1950 granting his
include cases which have resulted from petition for naturalization, he together with
culpable negligence of the parent or the person his wife So Buan Ty filed another petition also
exercising parental authority. in this Court in Special Proc. No. 1216 for the
adoption of Ching Tiong Seng, Ching Liong
Ding, Victoria Ching Liong Yam, Sydney Ching
If the degree of seriousness so warrants, or the and Ching Tiong An, all minors and
welfare of the child so demands, the court shall admittedly the illegitimate children of
deprive the guilty party of parental authority or petitioner Ching Leng with one Sy An, a
adopt such other measures as may be proper Chinese citizen. Finding the petition for
under the circumstances. adoption proper, this Court granted the same
in a decision dated September 12, 1950,
The suspension or deprivation may be revoked declaring the said minors free from all legal
and the parental authority revived in a case obligations of obedience and maintenance with
filed for the purpose or in the same proceeding respect to their mother Sy An and to all legal
if the court finds that the cause therefor has intents and purposes the children of the
ceased and will not be repeated. (33a) adopter Ching Leng alias Ching Ban Lee
and So Buan Ty with all the legal rights
Art. 232. If the person exercising parental and obligations provided by law.
authority has subjected the child or allowed
him to be subjected to sexual abuse, such On September 29, 1955, Ching Leng took his
person shall be permanently deprived by the oath of allegiance and became therefore
court of such authority. (n) a full pledge (sic) Filipino citizen. Believing
now that his adopted illegitimate children
Art. 233. The person exercising substitute became Filipino citizens by virtue of his
parental authority shall have the same naturalization, petitioner Ching Leng addressed
authority over the person of the child as the a communication to the respondent
parents. Commissioner of Immigration requesting that
the alien certificate of registration of the said
minors be cancelled. (Bold underscoring
In no case shall the school administrator, supplied)
teacher of individual engaged in child care

Private International Law Page 62


In Ching Leng, the Court made a definitive
ruling on the meaning of "minor child or • On 30th of October, 1915 the steamship
children" in Section 15 of the Naturalization Tian arrived at the port of Manila.
Law, as well as the meaning of children "whose • A woman, Marcosa S. Dy Jiongco, together
parents are citizens of the Philippines" under with two children, Ng Tio a female of the
the Constitution. The Court categorically age of 9 years, and Ng Hian a boy of 16
ruled that these children refer to years of age (the petitioner herein) were on
legitimate children only, and not to board.
illegitimate children. Thus, the Court held: • Marcosa S. Dy Jiongco, a Filipina born of a
Filipina mother and a Chinese father was
It is claimed that the phrases "minor children" married to a Chinaman by the name of
and "minor child", used in these provisions, (Filipino name) Juan Uy Tue, (Chinese
include adopted children. The argument is name) Ng Chion Tue.
predicated upon the theory that an • That Juan Uy Tue (Ng Chion Tue), before
adopted child is, for all intents and his marriage with Marcosa S. Dy Jiongco,
purposes, a legitimate child. Whenever, had been married to a Chinese woman with
the word "children" or "child" is used in whom he had some children, the petitioner
statutes, it is generally understood, herein and also one called Ng Guan.
however, to refer to legitimate children, • It appears that Ng Guan was residing in the
unless the context of the law and its spirit Philippine Islands at the time of the
indicate clearly the contrary. Thus, for presentation of the present petition;
instance, when the Constitution provides that • The Chinese wife of Juan Uy Tue died while
"those whose parents are citizens of the the petitioner herein, Ng Hian, was a very
Philippines, "and "those whose mothers are small child;
citizens of the Philippines," who shall elect • Juan Uy Tue, after the death of his Chinese
Philippine citizenship "upon reaching the age of wife, married Marcosa S. Dy Jiongco.
majority", are citizens of the Philippines (Article • Ng Tio was the daughter of the brother of
IV, Section 1, subdivisions 3 and 4), our the said Juan Uy Tue, born of a Chinese
fundamental law clearly refers to legitimate father and mother; that the father of the
children (Chiong Bian vs. De Leon, 46 Off. Gaz., little girl had given her to Marcosa S. Dy
3652-3654; Serra v. Republic, L-4223, May 12, Jiongco;
1952).
• Marcosa S. Dy Jiongco, being the
stepmother of the said Ng Hian, adopted
Similarly, the children alluded to in said him and was bringing him to the Philippine
section 15 are those begotten in lawful Islands to study.
wedlock, when the adopter, at least is the • The Board of Special Inquiry refused the
father. In fact, illegitimate children are under right of each child to enter the Philippine
the parental authority of the mother and follow Islands;
her nationality, not that of the illegitimate • However, after a rehearing, Ng Tio was
father (U.S. vs. Ong Tianse, 29 Phil. 332, 335- admitted but Ng Hian was not.
336; Santos Co vs. Gov’t of the Philippines, 52 • Petitioner petitioned for habeas corpus in
Phil. 543, 544; Serra v. Republic, supra; the CFI, CFI granted entry to petitioner,
Gallofin v. Ordoñez, 70 Phil. 287; Quimsuan vs. Collector of Customs appealed; Thus the
Republic, L-4693, Feb. 16, 1953). Although, present petition;
adoption gives "to the adopted person the
same rights and duties as if he were a Issue:
legitimate child of the adopter", pursuant to
said Article 341 of our Civil Code, we have Whether or not the minor children of a
already seen that the rights therein deceased resident Chinese merchant have a
alluded to are merely those enumerated right to enter the territory of the Philippine
in Article 264, and do not include the Islands;
acquisition of the nationality of the
adopter. Held:

Moreover, as used in said section 15 of It is true that the petitioner, Ng Hian, had never
the Naturalization Law, the term been in the Philippine Islands before. It is also
"children" could not possibly refer to true that the said Marcosa S. Dy Jingco
those whose relation to the naturalized was his stepmother. She swore positively
person is one created by legal fiction, as, that she had adopted him. That fact is not
for instance, by adoption, for, otherwise, denied of record. Until the fact is denied we
the place and time of birth of the child must accept it. There is nothing in the
would be immaterial. The fact that the record which shows or tends to show that
adopted persons involved in the case at bar she had not adopted him in good faith.
are illegitimate children of appellant Ching The question whether or not Marcosa S. Dy
Leng does not affect substantially the legal Jiongco could bring Ng Hian into the territory of
situation before us, for, by legal fiction, they the Philippine Islands as her adopted son has
are now being sought to be given the status of been discussed by the Federal Courts of the
legitimate children of said appellant, despite United States. In the case of Ex parte Fong
the circumstance that the Civil Code of the Yim (134 Fed. Rep., 938), the court held
Philippine does not permit their legitimation. that:

Ng Hian v. Collector A Chinese merchant domiciled in the


34 Phil 248 United States has the right to bring into

Private International Law Page 63


this country with his wife minor children such decree is valid according to the
legally adopted by him in China, where it national law of the foreigner. Relevant to
is shown that the adoption was bona fide, the present case is Pilapil v. Ibay-Somera,
and that the children have lived as where the Court specifically recognized the
members of his family and have been validity of a divorce obtained by a German
supported by him for several years. citizen in his country, the Federal Republic of
Germany. The Court held in Pilapil that a
The court further said: foreign divorce and its legal effects may be
recognized in the Philippines insofar as
Of course, the question whether the adoption is respondent is concerned in view of the
a genuine one is a question of fact, open to nationality principle in our civil law on the
investigation . . . . The evidence shows that the status of persons.
practice of adopting children in China is very
common, that it takes place substantially In this case, the divorce decree issued by
without legal formalities, but that the rights the German court dated December 16,
and obligations of children adopted and 1997 has not been challenged by either of
recognized as such are similar to those of the parties. In fact, save for the issue of
natural children. Under these parental custody, even the trial court
circumstances I can see no difference recognized said decree to be valid and binding,
between the legal status of adopted thereby endowing private respondent the
children and of natural children. The capacity to remarry. Thus, the present
Supreme Court (of the United States) having controversy mainly relates to the award of the
decided that a Chinese merchant domiciled custody of their two children, Carolynne and
in this country has the right to bring into Alexandra Kristine, to petitioner.
it his natural children, I think that the
same decision is authority for the
proposition that he has the right to As a general rule, divorce decrees
introduce his adopted children; obtained by foreigners in other countries
are recognizable in our jurisdiction, but
ROEHR VS. RODRIGUEZ the legal effects thereof, e.g. on custody,
care and support of the children, must
still be determined by our courts. Before
Facts: our courts can give the effect of res judicata to
a foreign judgment, such as the award of
Petitioner Wolfgang Roehr, a German citizen, custody to petitioner by the German court, it
married a Filipina, Respondent Carmen must be shown that the parties opposed
Rodriguez in Germany. The marriage was to the judgment had been given ample
ratified in Tayasan, Negros Oriental. They had opportunity to do so on grounds allowed
two children. Private respondent filed a petition under Rule 39, Section 50 of the Rules of
for the declaration of nullity of marriage before Court (now Rule 39, Section 48, 1997
the RTC of Makati. Petitioner filed a motion to Rules of Civil Procedure), to wit:
dismiss but was denied by the trial court.
However, The petitioner obtained a decree of SEC. 50. Effect of foreign
divorce from the Court of First Instance of judgments. - The effect of a judgment
Hamburg - Blankenese with the custody of the of a tribunal of a foreign country,
children granted to the father. having jurisdiction to pronounce the
judgment is as follows:
Issue:
(a) In case of a judgment upon a
Whether or not the legal effects of a divorce specific thing, the judgment is
obtained from a foreign country such as conclusive upon the title to the thing;
support and custody of the children can be
determined in our courts?
(b) In case of a judgment against a
Held: person, the judgment is
presumptive evidence of a right as
Yes. In order to take effect, a foreign between the parties and their
judgement must clearly show that the successors in interest by a
opposing party has been given ample subsequent title; but the judgment
opportunity to do so under the Rules of may be repelled by evidence of a
Civil Procedure. Accordingly, the respondent want of jurisdiction, want of notice
was not given the opportunity to challenge the to the party, collusion, fraud, or
judgment of the German Court, therefore, legal clear mistake of law or fact.
effects of divorce must be determined in our
courts. The court held that the trial court has It is essential that there should be an
jurisdiction over the issue between the parties opportunity to challenge the foreign judgment,
as to who has the parental custody. in order for the court in this jurisdiction to
properly determine its efficacy. In this
jurisdiction, our Rules of Court clearly
In Garcia v. Recio,19 Van Dorn v. Romillo, provide that with respect to actions in
Jr.,20 and Llorente v. Court of Appeals, the personam, as distinguished from actions
Court consistently held that a divorce in rem, a foreign judgment merely
obtained abroad by an alien may be constitutes prima facie evidence of the
recognized in our jurisdiction, provided

Private International Law Page 64


justness of the claim of a party and, as
such, is subject to proof to the contrary.

In the present case, it cannot be said that


private respondent was given the opportunity
to challenge the judgment of the German court
so that there is basis for declaring that
judgment as res judicata with regard to the
rights of petitioner to have parental custody of
their two children. The proceedings in the
German court were summary.

Exam is on Wednesday
630 to 830
Coverage is from start to parents and
children

Private International Law Page 65

S-ar putea să vă placă și